File:  [Local Repository] / db / baza / okmar18.txt
Revision 1.12: download - view: text, annotated - select for diffs - revision graph
Tue Jan 8 17:17:54 2019 UTC (5 years, 4 months ago) by rubashkin
Branches: MAIN
CVS tags: HEAD
*** empty log message ***

Чемпионат:
Окский марафон - 2018 (Пущино)

Дата:
27-Jan-2018

Тур:
1 тур. "Hangover" (Архангельск - Северодвинск)

Вопрос 1:
Герой одного сериала - обиженный жизнью неудачник, который неожиданно
смог добиться успеха. Его имя совпадает с названием населенного пункта,
полиция которого 3 мая 2016 года попросила горожан вести себя хорошо и
быть осторожными. Назовите этот населенный пункт.

Ответ:
Лестер.

Комментарий:
Как и футбольный "Лестер", Лестер Найгаард, герой сериала "Фарго",
превращается из неудачника в победителя. Официальный твиттер полиции
графства Лестершир также поздравил фанатов с исторической победой, но не
забыл и о безопасности.

Источник:
   1. Телесериал "Фарго", 1-й сезон.
   2. https://www.eurosport.ru/football/premier-league/2015-2016/story_sto5558324.shtml

Автор:
Алексей Амосов

Вопрос 2:
Футбольный клуб "Истанбул Башакшехир" наделал много шума в Турецкой
суперлиге, заняв в сезоне 2016/17 второе место. Говоря об этом, Влад
Шабловский подмечает, что раньше в районе Башакшехир ДЕЛАЛИ ЭТО. В
России ЭТО ДЕЛАЮТ, например, в Казани и Тамбове. Какие два слова,
начинающиеся на одну букву, мы заменили словами "ДЕЛАТЬ ЭТО"?

Ответ:
Производить порох.

Зачет:
По смыслу с удержанием формы.

Комментарий:
"Истанбул Башакшехир" выстрелил в минувшем сезоне, обогнав в турнирной
таблице многих грандов турецкого футбола; Казань и Тамбов известны
своими пороховыми заводами.

Источник:
   1. https://www.sports.ru/tribuna/blogs/pylny4erdak/1332380.html
   2. https://ru.wikipedia.org/wiki/Казанский_пороховой_завод
   3. https://www.fkptpz.ru/ru/

Автор:
Сергей Коновалов

Вопрос 3:
На карикатуре XVI века, среди прочих, можно увидеть, например, лица
монаха и демона. Карикатура называется "ПРОПУСК Мартина Лютера". В одной
из ранних трактовок известной книги ПРОПУСК - это римские императоры от
Нерона до Траяна, в гораздо более поздней - руководители СССР. Какие два
слова мы дважды пропустили в тексте вопроса?

Ответ:
Семь голов.

Комментарий:
Мартин Лютер представлялся своим современникам-католикам "зверем о семи
головах" - предвозвестником конца света из "Апокалипсиса".

Источник:
https://syg.ma/@zotow/liutyi-liutier-ikonoghrafichieskii-biespriediel-rieformatsii

Автор:
Денис Никитенко

Вопрос 4:
В романе Виктора Пелевина описывается картина, на которой изображена
девочка с головой известного писателя. Автор пишет, что соединительный
шов закрывает ИКС. Назовите ИКС словом с дефисом.

Ответ:
Галстук-бабочка.

Комментарий:
Портрет представляет собой голову Владимира Набокова, который был
большим любителем бабочек, пришитую к телу девочки.

Источник:
В.О. Пелевин. Empire V.
https://books.google.ru/books?id=UcTMAAAAQBAJ&pg=PT35#v=onepage&q&f=false

Автор:
Денис Никитенко

Вопрос 5:
В своей статье Станислав Купцов пишет, что в стране был популярен
пушкарт - спуск с горы на самодельных колесных устройствах. Статья
посвящена появлению ЕЕ. Назовите ЕЕ четырьмя словами.

Ответ:
Сборная Ямайки по бобслею.

Комментарий:
Сборная Ямайки по бобслею впервые приняла участие в Зимних Олимпийских
Играх в Калгари, а теперь и вовсе не пропускает крупные международные
старты.

Источник:
https://www.sports.ru/tribuna/blogs/heroesofsport/1535255.html

Автор:
Сергей Коновалов

Вопрос 6:
Последний представитель династии Сансонов жил в бедности, но ему удалось
поправить свое материальное положение, после того как он продал
дедушкину гильотину братьям. Назовите фамилию братьев.

Ответ:
Тюссо.

Комментарий:
После смерти Марии Тюссо в 1850 году знаменитый музей перешел в руки ее
сыновей; последний представитель династии Сансонов продал им дедушкину
гильотину для экспозиции, посвященной Французской революции.

Источник:
https://diletant.media/articles/35802288/

Автор:
Сергей Коновалов

Вопрос 7:
Распределение ролей между Вячеславом Невинным и Виктором Сергачёвым в
экранизации произведения конца XIX века некоторые критики сочли очень
оригинальным и удачным. Назовите автора экранизируемого произведения.

Ответ:
[Антон Павлович] Чехов.

Комментарий:
Грузный Невинный играет роль Тонкого, в то время как стройный Сергачёв -
роль Толстого.

Источник:
Х/ф "Смешные люди!" (1977), реж. Михаил Швейцер.

Автор:
Денис Никитенко

Вопрос 8:
Описывая Эрзерумскую наступательную операцию 1916 года, Евгений Белаш
отмечает, что русские войска были очень хорошо подготовлены к данной
битве. Бойцы были обеспечены теплыми портянками, валенками, полушубками,
рукавицами и даже консервами. Что мы пропустили в предыдущем
предложении?

Ответ:
Очками-.

Зачет:
Очками.

Комментарий:
Операция происходила зимой в горной местности, так что бойцам нужна была
защита от снежной слепоты.

Источник:
https://warspot.ru/4961-erzerumskaya-operatsiya-russkiy-triumf-v-turtsii

Автор:
Сергей Коновалов

Вопрос 9:
В первые дни учений "Millenium Challenge" условный противник одержал
неожиданные победы, после чего учения были перезапущены, и обеим
сторонам был навязан жесткий сценарий. Автор вопроса назвал эти учения
АЛЬФОЙ. Настоящая операция "АЛЬФА" уже проводилась почти за шестьдесят
лет до этого. Какие два слова мы заменили словом "АЛЬФА"?

Ответ:
Рельсовая война.

Комментарий:
Жесткий линейный сценарий, не допускающий отклонений, часто называется
рельсовым. Партизанская операция "Рельсовая война" проводилась в 1943
году.

Источник:
   1. ЛОАВ.
   2. https://ru.wikipedia.org/wiki/Рельсовая_война

Автор:
Сергей Дмитриев

Вопрос 10:
По одной из версий, ОН - это результат перемещения людей между
параллельными мирами. В одном из ЕГО примеров упоминается ботинок.
Назовите ЕГО двумя словами, одно из которых - имя собственное.

Ответ:
Эффект Манделы.

Комментарий:
Эффект Манделы связан с ложными общими воспоминаниями, например, с тем,
что Никита Хрущёв стучал ботинком во время речи на трибуне.

Источник:
https://lurkmore.to/Эффект_Манделы

Автор:
Алексей Амосов

Вопрос 11:
Внимание, в вопросе словом "ИКС" заменены два слова, начинающиеся на
соседние буквы.
   Сотрудникам полиции Подольска удалось установить личность грабителя
при помощи ИКСА. Косвенной причиной гибели героя англоязычного романа
также стал ИКС. Назовите этого героя.

Ответ:
Горлум.

Зачет:
Смеагорл.

Комментарий:
ИКС - откушенный палец.

Источник:
   1. https://www.newsru.com/crime/14apr2017/biteoffrobfingrpdm.html
   2. Дж.Р.Р. Толкин. Властелин колец: Возвращение короля.

Автор:
Алексей Амосов

Вопрос 12:
"Всё есть раса, другой истины нет", "Невозможно что-либо сделать, пока
арийские расы не высвободятся из пут семитизма". Парадоксально, но эти
фразы принадлежат известному европейскому политику и писателю. Возможно,
в том числе поэтому литературоведы считают, что из двух именно ИКС
является карикатурой на этого человека, ведь "ИКС" - слово двухкоренное.
Назовите ИКС.

Ответ:
Единорог.

Комментарий:
Премьер-министр Великобритании Бенджамин Дизраэли часто позволял себе
откровенно расистские и даже антисемитские высказывания, несмотря на
свое еврейское происхождение; битва Единорога и Льва в "Алисе в
Зазеркалье" - прямая аллюзия на политическую борьбу Дизраэли и
Гладстона.

Источник:
   1. http://scepsis.net/library/id_2694.html
   2. https://ru.wikipedia.org/wiki/Лев_и_единорог

Автор:
Денис Никитенко

Вопрос 13:
Давид Алрой, предводитель восстания курдских евреев XII века, известен
благодаря двум произведениям: первое - воспоминания раввина Беньямина
Тудельского, а второе - исторический роман, написанный в 1833 году.
Назовите автора этого романа.

Ответ:
[Бенджамин] Дизраэли.

Комментарий:
Как вы узнали из предыдущего вопроса, Дизраэли был не только политиком,
но и писателем, поэтому понятен его интерес к еврейской теме. Интересно,
что авторы, писавшие о Давиде Алрое, были тезками.

Источник:
   1. http://www.shabashkevich.com/bio.php?id=201
   2. Б. Дизраэли. Алрой. http://flibusta.is/b/345866/read
   3. https://ru.wikipedia.org/wiki/Алрой,_Давид

Автор:
Денис Никитенко

Вопрос 14:
Дуплет.
   1. ИКС ТАКОГО буйвола не является особенно желанным трофеем. Буйволы
совсем не водятся на ТАКОМ ИКСЕ. Какое короткое слово мы заменили словом
"ИКС"?
   2. ИКСЫ ТАКОГО буйвола образуют что-то вроде ИГРЕКА. ИГРЕКИ делают из
шкуры буйвола. Какое короткое слово мы заменили словом "ИГРЕК"?

Ответ:
   1. Рог.
   2. Щит.

Комментарий:
   1. Рог африканского буйвола не пользуется особенным интересом у
браконьеров; сам африканский буйвол не водится на полуострове Сомали,
который еще называется "Африканским Рогом".
   2. У взрослых быков основания рогов на лбу срастаются, образуя нечто
вроде сплошного костного щита; масаи делают свои щиты из шкуры
африканского буйвола.

Источник:
https://ru.wikipedia.org/wiki/Африканский_буйвол

Автор:
Денис Никитенко

Вопрос 15:
Мануэль Годой, маркиз Альварес де Фариа, герцог Алькудия, гранд 1-го
класса, которого называли злым гением Испании, много сделал для выхода
своей страны из войны с Францией и получил титул ИКС. Чьей
характеристикой является ИКС, употребляемый вместе с местоимением?

Ответ:
Дьявола.

Зачет:
Сатаны; прочие синонимы.

Комментарий:
Несмотря на обилие наград, Мануэль Годой после Базельского мира 1795
года получил еще и титул "князь мира".

Источник:
   1. Г.И. Ревзин. Риэго. http://flibusta.is/b/400647/read
   2. https://dic.academic.ru/dic.nsf/brokgauz_efron/121963/

Автор:
Денис Никитенко

Вопрос 16:
Рассуждая о том, что запасов органического топлива в мире осталось
совсем мало, автор вопроса пришел к выводу, что АЛЬФА все-таки НЕ ДЕЛАЕТ
ЭТО. Назовите автора произведения 1989 года, в котором АЛЬФА ДЕЛАЕТ ЭТО.

Ответ:
[Егор] Летов.

Комментарий:
АЛЬФА ДЕЛАЕТ ЭТО - вечность пахнет нефтью.

Источник:
   1. ЛОАВ.
   2. Егор Летов, "Русское поле экспериментов".

Автор:
Алексей Амосов

Вопрос 17:
Статья о губернаторе, состоящем в КПРФ, но стремящемся снизить
антиправительственную риторику партии, называется "Сеньор Помидор".
Какие три буквы мы заменили в тексте вопроса?

Ответ:
Тер.

Комментарий:
Автор статьи о губернаторе Орловской области Вадиме Потомском проводит
аналогии с Термидорианским переворотом, когда к власти пришли люди,
резко снизившие революционный накал.

Источник:
http://www.club-rf.ru/57/theme/449

Автор:
Денис Никитенко

Вопрос 18:
(pic: 20180278.jpg)
   Через секунду на диване появится и Лиза, а в АЛЬФЕ появится БЕТА.
Назовите АЛЬФУ двумя словами, начинающимися на одну и ту же букву.

Ответ:
Голова Гомера.

Комментарий:
Лиза в образе богини Афины появится, конечно же, из головы Зевса, в
которой образуется из-за этого дырка.

Источник:
Мультсериал "Симпсоны", s27e10.

Автор:
Денис Никитенко

Вопрос 19:
В статье о пиратских русификациях компьютерных игр Михаил Карпов
называет версию машинного перевода популярной компьютерной игры 2005
года "порченой". В предыдущем предложении мы слегка изменили одно слово.
Восстановите это слово в исходном виде.

Ответ:
Потраченной.

Комментарий:
Речь идет об игре "GTA: San Andreas". Если главный герой умирал, то на
экране появлялась надпись "WASTED" ("уделали"), что автопереводчик
перевел как "ПОТРАЧЕНО", - с тех пор за этой версией и закрепилось
название "Потраченной". Персонажи Big Smoke, Ryder и Sweet стали Большим
Дымом, Ездоком и Сладким, а главный герой из Сиджея (CJ) преобразился в
загадочного СИДОДЖИ.

Источник:
https://lenta.ru/articles/2016/09/24/potracheno/

Автор:
Дмитрий Анциферов

Вопрос 20:
В одном из эпизодов мультсериала "My Little Pony" одна из героинь
пытается построить машину времени и просит помощи у персонажа по имени
Копыто. Какие восемь символов мы пропустили в тексте вопроса?

Ответ:
Доктор т.

Комментарий:
Персонажа зовут Doctor Whooves, что на русский перевели как "Доктор
Ктопыто". Аллюзия на путешественника во времени из одноименного
британского сериала сохранилась.

Источник:
Мультсериал "My Little Pony", s02e20.

Автор:
Алексей Амосов

Вопрос 21:
Роланд Эстерсон, герой серии романов Александра Зорича, - конструктор
космических истребителей. Интересно, что один из его кораблей получил
название "ИКС". Французская Википедия сообщает, что слово "ИКС" -
женского рода, чем и объясняется истинная привязанность и любовь.
Назовите ИКС.

Ответ:
Дюрандаль.

Комментарий:
Дюрандаль - легендарный меч Роланда, которым рыцарь очень дорожил. Имя
конструктора является дополнительной подсказкой.

Источник:
   1. А. Зорич. Завтра война. http://flibusta.is/b/162311/read
   2. https://fr.wikipedia.org/wiki/Durandal

Автор:
Алексей Амосов

Вопрос 22:
Сюжет программы "Сегодня", посвященный реставрационным работам в музее
деревянного зодчества "Кижи" называется "ПРОПУСК". ПРОПУСК двумя первыми
буквами отличается от названия произведения 1995 года. Заполните
пропуск, и да поможет вам богиня Анис!

Ответ:
Рубить по-русски.

Комментарий:
Богиня Анис - так называет свою жену Никита Джигурда; произведение 1995
года - фильм "Любить по-русски".

Источник:
   1. https://www.ntv.ru/video/259121/
   2. https://ru.wikipedia.org/wiki/Любить_по-русски

Автор:
Владимир Ермолин

Вопрос 23:
Дуплет.
   1. Дмитрий Циликин описывает фаталистичную атмосферу спектакля Театра
имени Ленсовета: сцена "обрамлена белыми шарообразными лампами, а над
ней реют шары, но уже черные, жутковатыми бесформенными гроздьями".
После этого критик упоминает гигантское насекомое и ПОСЛЕДНИЕ ДНИ".
Какие два слова мы заменили в этом вопросе?
   2. Во время одного из подвигов Геракл посетил БАТУМ, очевидно,
названный так потому, что там заканчивал свой путь Гелиос. Какие два
слова мы заменили одним?

Ответ:
   1. Роковые яйца.
   2. Багровый остров.

Комментарий:
"Роковые яйца", "Последние дни", "Батум" и "Багровый остров" -
произведения Михаила Булгакова; вероятно, название острова древние греки
придумали в связи с закатом.

Источник:
   1. https://www.dp.ru/a/2016/02/11/Uspehi_somnologii
   2. http://myths.kulichki.net/lostcivil/greece/myth0006/st33.shtml

Автор:
Денис Никитенко

Вопрос 24:
В фильме "Хранители" очень много отсылок. Так, Ночной Филин спасает от
вооруженного грабителя семейную пару. На одной из афиш на стене здания,
из которого они выходили, можно увидеть название произведения, русское
название которого состоит из двух слов, начинающихся на соседние буквы.
Напишите это название.

Ответ:
"Летучая мышь".

Комментарий:
Во вселенной "Хранителей" многие вещи не случились. Например, были
спасены родители Брюса Уэйна, которые шли с оперетты. "Летучая мышь" -
оперетта Штрауса.

Источник:
Х/ф "Хранители" (2009), реж. Зак Снайдер.

Автор:
Денис Никитенко

Тур:
2 тур. "Палец в носу застрял" (Москва)

Редактор:
Серафим Шибанов (Москва)

Вопрос 1:
В настоящее время ИссЭй СагАва, пройдя курс лечения, проживает в Токио и
иногда пишет ресторанные обозрения. Когда в начале 1990-х он увидел
известный фильм, то заявил, что персонаж, сыгранный этим актером,
совершенно нереалистичный. Назовите этого актера.

Ответ:
[Энтони] Хопкинс.

Комментарий:
Сагава прославился тем, что убил и съел свою однокурсницу. Выдуманный
маньяк-каннибал Ганнибал Лектер показался настоящему людоеду
недостаточно реалистичным.

Источник:
   1. А. Парфрей. Культура времен апокалипсиса.
http://flibusta.is/b/123515/read
   2. https://ru.wikipedia.org/wiki/Сагава,_Иссэй

Автор:
Серафим Шибанов (Москва)

Вопрос 2:
Йон РымАру, в начале 1970-х по ночам наводивший ужас на столицу, получил
прозвище из двух слов, начинающихся на соседние буквы. Напишите это
прозвище.

Ответ:
Бухарестский вампир.

Зачет:
Вампир Бухареста.

Комментарий:
Рымару - известный румынский серийный убийца.

Источник:
https://en.wikipedia.org/wiki/Ion_R%C3%AEmaru

Автор:
Серафим Шибанов (Москва)

Вопрос 3:
Фриц ХаармАнн владел мясной лавкой. По совету своего друга Ханса Гранса
ХаармАнн стал действовать почти как жительница Лондона. Напишите фамилию
ее мужа.

Ответ:
Ловетт.

Комментарий:
Хаарманн был серийным убийцей и избавлялся от тел жертв, добавляя их в
сосиски. Примерно таким же образом миссис Ловетт помогала избавляться от
трупов Суини Тодду.

Источник:
   1. https://ru.wikipedia.org/wiki/Хаарманн,_Фриц
   2. https://en.wikipedia.org/wiki/Mrs._Lovett

Автор:
Серафим Шибанов (Москва)

Вопрос 4:
В молодости Юро КАра написал сценарий и сыграл главную роль в фильме
"Оскверненные ангелы" - о том, как 18-летний паренек после посещения
порнокинотеатра в первой попавшейся ему по дороге из кино больнице
насилует, потрошит и убивает всех медсестер. 14 лет спустя после выхода
фильма Кара получил письмо из Франции. От кого?

Ответ:
От [ИссЭя] СагАвы.

Комментарий:
Знаменитый каннибал вдохновлялся, в частности, и таким вот арт-хаусом,
созданным соотечественником.

Источник:
https://daily.afisha.ru/archive/gorod/archive/gannibal/

Автор:
Серафим Шибанов (Москва)

Вопрос 5:
Дуплет.
   1. 17-летний Антон, как и его кумир Боб, с большим почтением
относится к группе "KMFDM" [ка-эм-эф-дэ-эм]. Какое имя мы заменили в
этом вопросе?
   2. Считается, что у 16-летнего Александра есть страница во
"ВКонтакте" с ником Эд Харрис. Какое имя мы заменили в этом вопросе?

Ответ:
   1. Дилан.
   2. Эрик.

Комментарий:
Речь идет о Дилане Клиболде и Эрике Харрисе, учениках школы "КолумбАйн",
устроивших в ней бойню. Антон БичИвин из Улан-Удэ и Александр БуслИдзе
из Перми недавно отметились схожими действиями.

Источник:
   1. https://t.me/breakingmash/3614
   2. https://vk.com/id468910658
   3. https://vk.com/id468721354
   4. https://sobesednik.ru/obshchestvo/20180123-pochemu-podrostki-idut-vojnoj-na-svoi-shkoly

Автор:
Серафим Шибанов (Москва)

Вопрос 6:
ЕГО личность смогли установить после того, как в сентябре ОН решил
обналичить в одном из нью-йоркских банков золотой сертификат с истекшим
сроком годности. Назовите фамилию Чарльза, который упоминается в статье
Википедии о НЕМ.

Ответ:
Линдберг.

Комментарий:
Именно этими золотыми сертификатами и был внесен выкуп за ребенка
знаменитого летчика. Бруно Гауптман то ли просто опоздал, то ли
специально ждал, пока шумиха вокруг "преступления века" поутихнет.

Источник:
   1. https://en.wikipedia.org/wiki/Lindbergh_kidnapping
   2. https://en.wikipedia.org/wiki/Gold_certificate
   3. https://en.wikipedia.org/wiki/Richard_Hauptmann

Автор:
Серафим Шибанов (Москва)

Вопрос 7:
В видеоигре "Manhunt" [мэнхАнт] отрицательных героев зовут Лайонел
СтарквЕзер и Джеймс В. ГЕйси, что роднит их с участниками известного
коллектива. Назовите лидера этого коллектива.

Ответ:
Мэрилин Мэнсон.

Комментарий:
Упомянутые герои носят фамилии знаменитых серийных убийц (Чарльза
Старквезера и Джона Гейси). Участники группы "Marylin Manson" тоже взяли
себе псевдонимами фамилии серийных убийц - в частности, Джорди Осборн
Уайт стал Твигги Рамиресом.

Источник:
   1. https://ru.wikipedia.org/wiki/Manhunt
   2. https://ru.wikipedia.org/wiki/Marilyn_Manson

Автор:
Серафим Шибанов (Москва)

Вопрос 8:
   <раздатка>
   S = f (V, D),
   где S - вероятность,
   f - функция,
   V - ранимость (vulnerability),
   D - общественные лишения (deprivation)
   </раздатка>
   Перед вами закон ФАрбера, описывающий вероятность ЕГО. Каким ОН был в
заглавии песни, записанной в 1990 году?

Ответ:
Непрерывным.

Комментарий:
По закону Фарбера, когда какой-либо из показателей (V, D) на максимуме,
суицид вероятнее всего. В 1990 году Егор Летов записал альбом
"Инструкция по выживанию", в который вошли песни одноименной группы за
авторством Романа Неумоева, среди которых был и "Непрерывный суицид".

Источник:
   1. Г.Ш. Чхартишвили. Писатель и самоубийство.
http://flibusta.is/b/82210/read
   2. http://www.gr-oborona.ru/pub/discography/instrukcija_po_vyzhivaniju.html

Автор:
Серафим Шибанов (Москва)

Вопрос 9:
Диктатор Панамы Мануэль НорьЕга был упрятан властями США за решетку
сразу после своего свержения. В фильме 2013 года однофамилец диктатора
по имени Эдуардо сыграл ЕГО. Назовите ЕГО десятибуквенным словом.

Ответ:
Наркобарон.

Комментарий:
Актер, сыгравший роль наркобарона, является однофамильцем реально
существующего торговца наркотиками Мануэля Норьеги, бывшего диктатора
Панамы.

Источник:
   1. https://ru.wikipedia.org/wiki/Норьега,_Мануэль
   2. https://ru.wikipedia.org/wiki/Возвращение_героя

Автор:
Серафим Шибанов (Москва)

Вопрос 10:
ЕГО также называют "черным золотом". Из-за НЕГО собирались ограничить
доступ... К чему?

Ответ:
К русскому сегменту Википедии.

Зачет:
По упоминанию Википедии.

Комментарий:
ОН - это наркотик чАрас. В 2015 году Роскомнадзор собирался
заблокировать Википедию из-за отказа редактировать содержание статьи.

Источник:
   1. https://ru.wikipedia.org/wiki/Чарас_(наркотическое_вещество)
   2. https://ru.wikipedia.org/wiki/Блокирование_Википедии_в_России

Автор:
Серафим Шибанов (Москва)

Вопрос 11:
В этом вопросе ИКС - это замена.
   Робин Уильямс однажды заметил, что ИКС - это способ Господа показать
человеку, что тот зарабатывает слишком много. В фильме "ИКС" Клифф
Кёртис сыграл уроженца Рио-Негро. Назовите этого уроженца.

Ответ:
[Пабло] Эскобар.

Комментарий:
ИКС - это кокаин.

Источник:
   1. https://www.goodreads.com/quotes/519813-cocaine-is-god-s-way-of-telling-you-you-are-making
   2. https://en.wikipedia.org/wiki/Blow_(film)
   3. https://en.wikipedia.org/wiki/Pablo_Escobar

Автор:
Серафим Шибанов (Москва)

Вопрос 12:
Один интернет-пользователь пошутил, что в своем стремлении бороться с
заразой некое ведомство назвало концерт известной группы в России
нежелательным. О какой группе идет речь?

Ответ:
"Spice Girls" [чтецу: спайс гёрлз].

Комментарий:
ФСКН очень не любит спайсы.

Источник:
https://samir-befish.livejournal.com/1539.html

Автор:
Серафим Шибанов (Москва)

Вопрос 13:
Какое длинное прозвище получил от коллег герой романа Георгия Вайнера
Константин Константинович Кузьмичёв?

Ответ:
КуКлуксКлан.

Комментарий:
Его инициалы были ККК, что и привело к появлению прозвища.

Источник:
Г.А. Вайнер. Райский сад дьявола. http://flibusta.is/b/200512/read

Автор:
Серафим Шибанов (Москва)

Вопрос 14:
   <раздатка>
   ignored, captured, murdered, tortured
   </раздатка>
   Перед вами слова, встречающиеся в статье англоязычной Википедии.
Назовите тех, кому она посвящена, двумя словами, начинающимися на одну и
ту же букву.

Ответ:
Красные кхмеры.

Комментарий:
В каждом из этих слов есть буквосочетание "red".

Источник:
https://en.wikipedia.org/wiki/Khmer_Rouge

Автор:
Серафим Шибанов (Москва)

Вопрос 15:
В этом вопросе ИКС - это замена.
   Во время резни в доме Шарон Тейт один из убийц написал на стене
кровью слово "Свинья". Интересно, что фамилия Линды КасейбиАн, входившей
в секту Чарльза Мэнсона, происходит от арабского слова со значением
"ИКС". В известном произведении ИКС и сам стал жертвой. Чьей жертвой?

Ответ:
Робина Бобина Барабека.

Комментарий:
ИКС - это мясник. Робин Бобин Барабек, как известно, съел корову, и
быка, и кривого мясника.

Источник:
   1. https://en.wikipedia.org/wiki/Kasabian
   2. https://en.wikipedia.org/wiki/Tate_murders
   3. https://en.wikipedia.org/wiki/Linda_Kasabian
   4. https://45parallel.net/korney_chukovskiy/barabek.html

Автор:
Серафим Шибанов (Москва)

Вопрос 16:
В этом вопросе АЛЬФА - это замена.
   На конкурсе "Мисс РФПЛ - 2015" один из призов выиграла Ольга
КузькОва. Вскоре после этого разразился скандал, поскольку Ольга
оказалась АЛЬФОЙ. Один из пользователей даже отметил: "С Прядкина за это
спросим". Какой химический элемент можно найти в АЛЬФЕ?

Ответ:
Неон.

Комментарий:
"С Прядкина за это спросим - один четыре восемь восемь". Кузькова
оказалась неонацисткой. В слове "неонацистка" можно найти "неон".

Источник:
https://www.sports.ru/tribuna/blogs/buzzing/806917.html

Автор:
Серафим Шибанов (Москва)

Вопрос 17:
Можно сказать, что для Николая II, вернувшегося из Японии с, так
сказать, "сувениром", ИМ был ХорИте из Кобе. ОН идет третьим на альбоме
1993 года. Назовите ЕГО.

Ответ:
Кольщик.

Комментарий:
Кольщик - это тюремное наименование татуировщика. Песня с таким
названием есть на альбоме "ЖигАн-лимон", который принадлежит авторству
Михаила Круга.

Источник:
   1. Ф. Османова, Д. Стахов. Истории простых вещей.
http://flibusta.is/b/349623/read
   2. https://ru.wikipedia.org/wiki/Жиган-лимон
   3. https://ru.wiktionary.org/wiki/кольщик

Автор:
Серафим Шибанов (Москва)

Вопрос 18:
Героиня романа Питера Хёга так опасается самоубийства, что в ее доме
пустяк. Какие три слова, начинающиеся на одну и ту же букву, мы заменили
одним?

Ответ:
Не наточены ножи.

Комментарий:
Перерезать вены или горло таким ножом тоже очень трудно. Песня Олега
Митяева "Ночной гость", в которой есть строки "И пустяк, что не наточены
ножи", известна в исполнении Михаила Шуфутинского.

Источник:
   1. П. Хёг. Фрекен Смилла и ее чувство снега.
http://flibusta.is/b/174020/read
   2. http://www.karaoke.ru/song/6536.htm

Автор:
Серафим Шибанов (Москва)

Вопрос 19:
В этом вопросе АЛЬФА и ИКС заменяют другие слова.
   Петр ЛюкимсОн пишет, что еще в 1935 году была куплена АЛЬФА, на
которой ИКС работал свыше 50 лет. Совсем иная АЛЬФА ИКСА упоминается в
известной песне 2001 года. Назовите группу, которая эту песню записала.

Ответ:
"Воровайки".

Комментарий:
АЛЬФА ИКСА - это машинка Зингера. В первом случае речь идет о
нобелевском лауреате по литературе Исааке Башевисе Зингере. Во втором -
о швейной машинке. "Машинка Зингера иголочку сломала" - это цитата из
известной песни "Хоп, мусорок" группы "Воровайки".

Источник:
   1. П.Е. Люкимсон. Последний Бес. Жизнь и творчество Исаака
Башевиса-Зингера.
https://books.google.ru/books?id=1EN3DgAAQBAJ&pg=PT220#v=onepage&q&f=false
   2. http://vorovaiki.ru/vorovaiki/music/
   3. https://ru.wikipedia.org/wiki/Воровайки

Автор:
Серафим Шибанов (Москва)

Вопрос 20:
Живший одно время на Таганской площади интернет-пользователь
рассказывает, как однажды поселился в гостинице на втором этаже. Сообщая
об этом друзьям, чтобы те зашли в гости, он процитировал строчку из
песни 1995 года. Напишите эту строчку.

Ответ:
Мой номер 245.

Комментарий:
Известная песня Гарика Кричевского. Есть в ней и слова "А раньше жил я
на Таганке".

Источник:
   1. https://samir-befish.livejournal.com/2263.html
   2. http://krichevsky.ws/discografia/privokzalnaya/

Автор:
Серафим Шибанов (Москва)

Вопрос 21:
Документальный фильм, посвященный кинокартине Майкла Пауэлла, называется
"Чисто британский психопат". Назовите эту кинокартину.

Ответ:
"Подглядывающий".

Зачет:
"Peeping Tom"; даже "Подсматривающий Том" и схожие ответы.

Комментарий:
Фильм Майкла Пауэлла, кроме прообраза современных слэшеров, также
считают британским ответом на "Психо" Альфреда Хичкока. В некоторых из
отечественных переводов название фильма Хичкока переводится как
"Психопат".

Источник:
Rodley, Chris (dir.) et al. (1997). A Very British Psycho. Peeping Tom
(DVD). The Criterion Collection.

Автор:
Дмитрий Сальников (Москва)

Вопрос 22:
В известной игре ОНА называется "Усердный бобер". Гуннар Хансен,
снявшийся в фильме, в названии которого упоминается ОНА, в 11 лет
переехал вместе с семьей. В какой штат?

Ответ:
Техас.

Комментарий:
Гуннар Хансен сыграл роль маньяка Кожаное лицо в знаменитом фильме
ужасов "Техасская резня бензопилой".

Источник:
   1. https://ru.wikipedia.org/wiki/Doom
   2. https://en.wikipedia.org/wiki/Gunnar_Hansen

Автор:
Серафим Шибанов (Москва)

Вопрос 23:
В одном из недавних фильмов группа грабителей договаривается
использовать для ограбления маски известного кинозлодея. Однако на
встречу перед преступлением все грабители объявляются в масках
известного комика. Назовите кинозлодея.

Ответ:
Майк Майерс.

Комментарий:
Полный тезка главного персонажа "Хэллоуина" - комик Майк Майерс,
известный по киносерии "Остин Пауэрс".

Источник:
Х/ф "Малыш на драйве" (2017), реж. Эдгар Райт.

Автор:
Дмитрий Сальников (Москва)

Вопрос 24:
В пародийной инструкции фигурируют одна собачка, одна корзина, сорок две
бабочки, один лосьон и один парик. Имея на руках это всё, вы сможете
получить ЕГО. Назовите ЕГО.

Ответ:
Баффало Билл.

Зачет:
Джейм Гамб.

Комментарий:
Речь идет о маньяке из фильма "Молчание ягнят".

Источник:
https://pikabu.ru/story/instruktsii_ot_ikea_4639751

Автор:
Андрей Новиков (Москва)

Тур:
3 тур. "Канкан-Макуты" (Тверь - Клин - Москва)

Редактор:
Станислав Попов и Олег Холодов

Вопрос 1:
Известно, что македонская царица Олимпиада была очень ревнива и из-за
ревности постоянно портила жизнь царю Филиппу. Герой романа Мэри Рено,
перс по происхождению, утверждал, что в Македонии не было ни одного
человека, обученного правильно управлять войском. Какое слово мы
заменили в предыдущем предложении?

Ответ:
Гаремом.

Источник:
М. Рено. Персидский мальчик. http://flibusta.is/b/122775/read

Автор:
Софья Вискова

Вопрос 2:
В 2017 году Русская Служба Би-Би-Си опубликовала статью, в которой
эксперты по средневековому праву обсуждали законность притязаний на
престол нескольких героев саги "Песнь Льда и пламени" Джорджа Мартина.
Одним из экспертов выступил Ричард Уильям. В предыдущем предложении мы
пропустили несколько букв. Восстановите их.

Ответ:
Фитц.

Зачет:
Фиц.

Комментарий:
В частности, в статье обсуждались права бастардов на престол. Английские
фамилии бастардов традиционно начинались с приставки Фитц-. А фамилия
эксперта - Фитцуильям.

Источник:
https://www.bbc.com/russian/features-41065717

Автор:
Станислав Попов

Вопрос 3:
В вопросе есть замена.
   Биолог Александр Марков рассказывает, что в эволюции птиц четко
прослеживается адаптация к полету путем уменьшения массы тела и
оптимизации обмена веществ. Например, даже СПЕКТР птиц в полтора-два
раза короче, чем у млекопитающих. Какое слово мы заменили в предыдущем
предложении?

Ответ:
Геном.

Комментарий:
Короткий геном чисто физически уменьшает вес отдельной клетки, хотя,
конечно, всё намного сложнее. "Геном" и "Спектр" - романы Лукьяненко,
если это кому-то вдруг помогло.

Источник:
А.В. Марков. Рождение сложности. Эволюционная биология сегодня:
неожиданные открытия и новые вопросы. http://flibusta.is/b/400942/read

Автор:
Станислав Попов

Вопрос 4:
В 2017 году московскую кампанию по реновации быстро прозвали
"хреновацией", поскольку обещание обновления для многих обернулось
потерей. Ксения ТуркОва замечает, что подобное словообразование с
использованием буквы "Х" уже было. Какое слово она приводит в качестве
примера?

Ответ:
Прихватизация.

Источник:
https://snob.ru/selected/entry/132745

Автор:
Софья Вискова

Вопрос 5:
В этом фильме 1953 года снялись Мэрилин Монро и Джейн Рассел. Любопытно,
что продюсеры заплатили Рассел в несколько раз больше, чем Монро, и это
особенно возмутило Мэрилин. Как назывался этот фильм?

Ответ:
"Джентльмены предпочитают блондинок".

Комментарий:
Брюнетка Джейн Рассел получила гонорар в 18 раз больше. На тот момент
она была мегазвездой, а Монро только начала подъем.

Источник:
https://ru.wikipedia.org/wiki/Джентльмены_предпочитают_блондинок

Автор:
Софья Вискова

Вопрос 6:
В конце XIX века в Польше женщин не допускали в высшие учебные
заведения, и несколько профессоров организовали для них подпольные
курсы. Место проведения занятий постоянно менялось, поэтому организация
получила прозвище "ТАКОЙ университет". ТАКАЯ ОНА по-польски -
"nietoperz" [нетОпеш]. Какие слова мы заменили словами "ТАКАЯ ОНА"?

Ответ:
Летучая мышь.

Источник:
   1. https://ru.wikipedia.org/wiki/Летучий_университет
   2. https://translate.google.ru/#pl/ru/nietoperz

Автор:
Софья Вискова

Вопрос 7:
Мультфильм "Большой Тылль", вышедший в СССР в 1980 году, был исследован
цензорами на предмет наличия в нем националистических мотивов. В
частности, цензоры обратили внимание, что глаза главного героя содержали
ПРОПУСК. Заполните пропуск тремя словами, которые начинаются с букв,
каждая из которых в игре "Скрэббл" стоит не менее пяти очков.

Ответ:
Цвета эстонского флага.

Комментарий:
Большой Тылль - великан из эстонского эпоса. В советские времена
эстонский флаг был запрещен как националистический символ. Хотя иногда
голубые глаза - просто голубые глаза.

Источник:
https://ru.wikipedia.org/wiki/Большой_Тылль_(мультфильм)

Автор:
Станислав Попов

Вопрос 8:
Неправительственная североамериканская организация помогает лицам
нетрадиционной сексуальной ориентации тайно эмигрировать из стран, где
они подвергаются дискриминации и насилию. Эта организация называется
"Радужная ОНА". Назовите ЕЕ двумя словами.

Ответ:
Железная дорога.

Комментарий:
По аналогии с "Подпольной железной дорогой", помогавшей рабам бежать в
северные штаты.

Источник:
https://www.rainbowrailroad.ca/

Автор:
Софья Вискова

Вопрос 9:
   <раздатка>
   Floreat Etona
   </раздатка>
   В пьесе Джеймса Барри капитан Крюк гибнет со словами "Floreat Etona".
Переводчик Борис Заходер заменил эту фразу другой, но отсылка к прошлому
капитана Крюка сохранилась. Какую строчку он использовал?

Ответ:
"Gaudeamus igitur".

Комментарий:
"Floreat Etona" переводится как "Да здравствует Итон". Капитан Крюк, как
можно понять из этой фразы, был выпускником этого колледжа. Заходер в
своем переводе оставил намек на высшее образование капитана.

Источник:
https://arzamas.academy/mag/466-peter

Автор:
Софья Вискова

Вопрос 10:
   <раздатка>
   "Il Clom Dallas Muntognas"
   </раздатка>
   Песня "The Call of the Mountains" фолк-метал-группы "Eluveitie" стала
хитом, и группа впоследствии выпустила еще четыре ее версии. На каком
языке спета та версия, название которой мы раздали?

Ответ:
Ретороманский.

Зачет:
Романш; романшский; граубюндер.

Комментарий:
"Eluveitie" - швейцарцы и, в основном, пишут песни на мертвом галльском
языке. После успеха англоязычной песни они выпустили ее кавер-версии на
четырех государственных языках Швейцарии.

Источник:
Дискография "Eluveitie", бонус-диск "Call of the Mountains" (2014).

Автор:
Станислав Попов

Вопрос 11:
   <раздатка>
   ______ history drama
   </раздатка>
   Пьеса Майка Бартлетта "Король Карл III" написана в редком жанре,
название которого вы видите на раздаточном материале. Пропущенное слово
в названии этого жанра было последним в названии известной трилогии XX
века. Воспроизведите название трилогии по-русски или по-английски.

Ответ:
"Назад в будущее".

Зачет:
"Back to the Future".

Комментарий:
Необычный жанр - future history drama. "Король Карл III" - история
принца Чарльза, восшедшего на престол после смерти королевы Елизаветы
II. Пьеса имеет вид исторической хроники, но время ее действия -
будущее.

Источник:
   1. https://en.wikipedia.org/wiki/King_Charles_III_(film)
   2. https://en.wikipedia.org/wiki/Back_to_the_Future_(franchise)

Автор:
Софья Вискова

Вопрос 12:
   <раздатка>
   Decameron
   </раздатка>
   Фильм "Территория девственниц" - молодежная комедия, основанная на
"Декамероне" Боккаччо. Одно из прокатных названий этого фильма состояло
из двух слов, первое из которых - "Decameron". Напишите второе слово.

Ответ:
Pie.

Комментарий:
Аллюзия на "American Pie".

Источник:
https://en.wikipedia.org/wiki/Virgin_Territory

Автор:
Софья Вискова

Вопрос 13:
Подраздел "Создатели сериала" одной статьи русскоязычной Википедии дает
ссылки на уроженцев Портленда, Бруклина, Лос-Анджелеса, Детройта,
Бристоля, Филадельфии и еще одного города. Назовите этот город.

Ответ:
Спрингфилд.

Комментарий:
Это создатели сериала "Симпсоны". Выходит, что за право быть родиной
Гомера Симпсона спорят семь городов, в том числе и Спрингфилд.

Источник:
https://ru.wikipedia.org/wiki/Симпсоны

Автор:
Павел Кадочников

Вопрос 14:
Индиец Ом Сингх Ратор погиб в аварии на своем мотоцикле. Полиция
эвакуировала мотоцикл в участок, но, как гласит легенда, он мистическим
образом неоднократно исчезал из участка. Мы не спрашиваем, что
происходило с мотоциклом после исчезновений. Ответьте, с кем сравнил его
Джонатан Легг.

Ответ:
Хатико.

Комментарий:
Мотоцикл возвращался на место гибели хозяина. Сейчас на этом месте
построили храм, а мотоцикл стоит на алтаре.

Источник:
Сериал "Road less Traveled with Jonathan Legg", 1-я серия.

Автор:
Павел Кадочников

Вопрос 15:
Герой Сола Беллоу был тяжело травмирован психически, поэтому утверждает,
что живет в ИКСЕ. ИКС есть и в этом вопросе. Назовите ИКС двумя словами,
начинающимися на парные согласные.

Ответ:
Страдательный залог.

Источник:
С. Беллоу. Хендерсон - король дождя. http://flibusta.is/b/334636/read

Автор:
Павел Кадочников

Вопрос 16:
Лейл Лаундес, рассказывая о кокетливом прикрывании лица женщинами в
разных эпохах, сравнивает ПЕРВОЕ со ВТОРЫМ. Некоторые java-скрипты
предназначены для того, чтобы элементы ПЕРВОГО располагались в виде
ВТОРОГО. Назовите ПЕРВОЕ и ВТОРОЙ.

Ответ:
Меню, веер.

Источник:
Л. Лаундес. Скрытые сексуальные сигналы.
https://books.google.ru/books?id=kEHWBQAAQBAJ&pg=PT44#v=onepage&q&f=false

Автор:
Павел Кадочников

Вопрос 17:
У музыкального инструмента морИн хУур даже струны изготовлены из ИКСА.
Другой ИКС, вопреки поверьям, для человека безопасен. Назовите ИКС двумя
словами.

Ответ:
Конский волос.

Комментарий:
Морин хуур - монгольский инструмент, и не только его смычок, но и струны
делаются из конских хвостов. Черви волосатики, которых в народе называют
"конский волос", паразитируют только в насекомых, и залезть под кожу
человека при купании неспособны.

Источник:
   1. https://ru.wikipedia.org/wiki/Моринхур
   2. https://ru.wikipedia.org/wiki/Волосатики

Автор:
Станислав Попов

Вопрос 18:
В вопросе есть замены.
   Мишель Пастуро в книге, посвященной истории цвета в культуре, пишет,
что в Средние века в знак любви дама могла подарить рыцарю один из своих
ИКСОВ, который тот прикреплял к шлему. Вероятно, так и появились "ТАКИЕ
ИКСЫ". Назовите ТАКИЕ ИКСЫ по-русски или по-английски.

Ответ:
Зеленые рукава.

Зачет:
Greensleeves.

Источник:
М. Пастуро. Зеленый. История цвета. http://flibusta.is/b/512461/read

Автор:
Софья Вискова

Вопрос 19:
Внимание, в вопросе мы заменили два двухкоренных слова словами "ТАКОЙ" и
"СЯКОЙ".
   Один из распространенных сортов винограда называют то сира, то шираз.
Как правило, сира - это ТАКОЙ виноград, а шираз - СЯКОЙ. "ТАКИЕ ОНИ"
были написаны в 1835 году. Назовите "ТАКИХ ИХ".

Ответ:
Старосветские помещики.

Комментарий:
Название "Сира" закрепилось за французскими винами, а "Шираз" - за
австралийскими, американскими и южноафриканскими.

Источник:
   1. https://www.somelie.ru/events/sira-ono-zhe-shiraz-kak-zvuchit-v-bokale-vino-iz-raznykh-stran/
   2. https://ru.wikipedia.org/wiki/Старосветские_помещики

Автор:
Станислав Попов

Вопрос 20:
Художница из Новосибирска, работающая, как ни странно, не с деревом, а с
папье-маше, называет себя АЛЬФОЙ. АЛЬФА погибает в четвертой серии
третьего сезона. Назовите АЛЬФУ двумя словами. Двумя словами!

Ответ:
Мама Карла.

Источник:
   1. http://www.diary.ru/~vertebra-p
   2. https://en.wikipedia.org/wiki/Killer_Within

Автор:
Софья Вискова

Вопрос 21:
В книге Доди Смит в семье героини живет кот по имени Аб. Из текста
неясно, кастрирован кот или нет, хотя собаку в семье зовут... Как?

Ответ:
Элоиза.

Комментарий:
А полное имя кота - Абеляр.

Источник:
Д. Смит. Я захватываю замок. http://flibusta.is/b/378236/read

Автор:
???

Вопрос 22:
Барри Бакнелл завоевал популярность миллионов британцев в конце 1950-х
годов, однако недруги утверждали, что Бакнелл "уничтожил больше домов,
чем Люфтваффе". Назовите фамилию Андрея, который, по сути, делал то же,
что и Бакнелл, в России с 1992 года.

Ответ:
Бахметьев.

Комментарий:
Бакнелл вел на Би-Би-Си программу из разряда "как сделать что-то дома
своими руками". И много англичан самолично переделали памятники
викторианской архитектуры в черт знает что.

Источник:
   1. Hidden Killers of Post-War Homes.
http://www.bbc.co.uk/programmes/b03n2yxq
   2. https://ru.wikipedia.org/wiki/Бахметьев,_Андрей_Александрович

Автор:
Станислав Попов

Вопрос 23:
В книге Доди Смит герои обсуждают роман под названием "Борьба Иакова",
одна из глав которого была написана ТАК. Кто из наших соотечественников
нередко писал ТАК?

Ответ:
Владимир Маяковский.

Комментарий:
Одна из глав была написана "лесенкой" - видимо, так описывалась
"Лестница Иакова".

Источник:
   1. Д. Смит. Я захватываю замок. http://flibusta.is/b/378236/read
   2. https://dic.academic.ru/dic.nsf/ruwiki/356568/

Автор:
Софья Вискова

Вопрос 24:
Бактериальный мат - сложно устроенное сообщество нескольких видов
микроорганизмов с различными типами обмена веществ. Биолог Александр
Марков утверждает, что настоящий бактериальный мат должен быть именно
ТАКИМ. Ответьте сложным словом: каким - ТАКИМ?

Ответ:
Трехэтажным.

Зачет:
Многоэтажным.

Источник:
А.В. Марков. Рождение сложности. Эволюционная биология сегодня:
неожиданные открытия и новые вопросы. http://flibusta.is/b/400942/read

Автор:
Станислав Попов

Тур:
4 тур. "Полосатый кусь" (Зеленоград)

Редактор:
Константин Крюков

Вопрос 1:
Внимание, в вопросе есть замены.
   На самом деле американские ОНИ действительно ДЕЛАЮТ ЭТО. Поэтому
автор известного произведения вкладывал в его название просто игру слов,
но не оксюморон, о котором часто думали европейские читатели, видя
название книги. Назовите этого автора.

Ответ:
[Кен] Кизи.

Комментарий:
Американские кукушки вьют гнезда, в отличие от европейских, которые
подкладывают яйца в гнезда других птиц. Кен Кизи - автор книги "Пролетая
над гнездом кукушки" - вкладывал в название игру слов "кукушка" и
"сумасшедший", обозначающихся в английском языке одним словом.

Источник:
https://ru.wikipedia.org/wiki/Пролетая_над_гнездом_кукушки

Автор:
Илья Корнийчук

Вопрос 2:
Астанинский монумент Байтерек символизирует Дерево жизни. По легенде,
каждый год в кроне Дерева священная птица Самрук откладывает яйцо.
Продолжение легенды схоже с произведением, которое многим из нас
известно с детства. А кто является автором этого произведения?

Ответ:
[Корней Иванович] Чуковский.

Комментарий:
Яйцо символизирует Солнце, которое, по легенде, проглатывает дракон
Айдахар, живущий у подножия дерева. Этот цикл символизирует смену времен
года, дня и ночи и борьбу добра и зла.

Источник:
   1. https://ru.wikipedia.org/wiki/Байтерек_(монумент)
   2. https://ru.wikipedia.org/wiki/Краденое_солнце_(книга)

Автор:
Константин Крюков

Вопрос 3:
Илья Кабанов в рецензии на сериал "Корона", рассказывая о начале
правления Елизаветы II, замечает, что королевской семье не очень приятно
видеть достаточно откровенное изображение самих себя. Чуть ранее в этой
же рецензии упоминается известный политик. Назовите этого политика.

Ответ:
[Уинстон] Черчилль.

Комментарий:
Автор рецензии сравнил сериал с известным портретом Уинстона Черчилля. В
1954 году британский художник Грэхем Сазерленд написал портрет политика,
который позже был подарен тому на 80-летие. Подарок не пришелся по душе
имениннику. Картина была увезена семейством Черчиллей сразу после
торжества в их загородный дом, и с того момента эту работу больше никто
не видел.

Источник:
   1. Журнал "Дилетант", 2017, N 2. - С. 77.
   2. http://art-news.com.ua/5-kartin-s-pechalnoj-uchastyu-kotorye-my-bolshe-nikogda-ne-uvidim-6098.html

Автор:
Константин Крюков

Вопрос 4:
Основатели одной тайваньской компании решили дать ей мифологическое
название. Однако в итоге отбросили три первые буквы и зарегистрировали
компанию под получившимся именем, чтобы она располагалась выше в
телефонных справочниках. Какое название дали тайваньцы своей фирме?

Ответ:
"Asus".

Зачет:
"Асус".

Комментарий:
Изначально название должно было звучать как "Pegasus".

Источник:
https://en.wikipedia.org/wiki/Asus#Name

Автор:
Наталья Самсонова

Вопрос 5:
Внимание, в вопросе есть замены.
   После стыковки Союза Т-14 со станцией "Салют-7" космонавт Георгий
Гречко в шутку произнес: "Говорят, что [ПРОПУСК], а вот СЕВЕР к ПОДНОЖЬЮ
МИТРЫ пришел". Мы не спрашиваем, что мы заменили словами "СЕВЕР" и
"ПОДНОЖЬЕ МИТРЫ". Заполните пропуск первой частью известной поговорки.

Ответ:
Гора с горой не сходится.

Комментарий:
Чегет (Север, Находящийся в тени) и Памир (Подножье Митры) - это не
только горы, но и позывные космонавтов.

Источник:
В.П. Савиных. Записки с мертвой станции.
http://militera.lib.ru/explo/savinyh_vp/01.html

Автор:
Константин Крюков

Вопрос 6:
Согласно шутке финского автора комикса, его персонаж, выйдя полюбоваться
ночным небом, разочарованно был вынужден поставить ему такую оценку.
Назовите эту оценку двумя словами.

Ответ:
Ноль звезд.

Комментарий:
Для финского неба отсутствие звезд из-за пасмурной погоды - увы,
привычная реальность.

Источник:
http://mustaheppa.sarjakuvablogit.com/2017/08/25/kriittinen-taivaankatsomus/

Автор:
Екатерина Батракова

Вопрос 7:
Словами "АЛЬФА" и "БЕТА" в этом вопросе заменены двукоренные слова.
   Если искать АЛЬФУ в Википедии, то в первую очередь попадешь на
страницу БЕТЫ. Одна из моделей немецких военно-транспортных самолетов
1940-х годов имела второе название "АЛЬФА". А на создание какой БЕТЫ
вдохновили одного режиссера опыты доктора Менгеле?

Ответ:
Человеческой.

Комментарий:
Если искать в Википедии сороконожку, она перенаправляет на страницу
многоножки. Второе название "сороконожка" имел самолет Арадо Ar 232.
"Человеческая многоножка" - фильм, снятый Томом Сиксом в 2009 году.

Источник:
   1. Википедия, поисковый запрос "сороконожка".
   2. https://ru.wikipedia.org/wiki/Arado_Ar_232
   3. https://ru.wikipedia.org/wiki/Человеческая_многоножка

Автор:
Илья Корнийчук

Вопрос 8:
Вышедшая в 2016 году песня русского рэпера Федука посвящена столкновению
российских и британских фанатов в Марселе на чемпионате Европы во
Франции. Воспроизведите название песни, совпадающее с названием
спортивного соревнования.

Ответ:
"Тур де Франс".

Зачет:
"Tour de France".

Комментарий:
Выезд на матч своей команды на сленге называется туром.

Источник:
https://text-lyrics.ru/f/feduk/5358-feduk-tour-de-france-text-pesni.html

Автор:
Максим Скрипниченко

Вопрос 9:
Внимание, в вопросе есть замены.
   Прослушайте цитату из статьи, посвященной событию 2017 года: "Самая
засекреченная часть условий связана с условиями хранения файла. Всего
несколько человек знают, где лежат ОНИ. И это не стандартный накопитель,
где хранятся все продукты..., а специальные ХРАНИЛИЩА - с ограниченным
входом. Вынуть оттуда файл незаметным образом невозможно. Какое слово мы
заменили словом "ХРАНИЛИЩА"?

Ответ:
Чертоги.

Комментарий:
Это цитата из статьи про утечку в Сеть последнего эпизода сериала
"Шерлок".

Источник:
https://www.kp.ru/daily/26630.7/3649011/

Автор:
Константин Крюков

Вопрос 10:
В 1964 году в преддверии Олимпийских игр в Токио в Японии было выпущено
устройство под названием "манпо-кеи". Учитывая, какое счастливое число
упоминается в дословном переводе этого названия, назовите цель, которую
ставит это устройство, абсолютно точно.

Ответ:
10 тысяч шагов в день.

Комментарий:
"Манпо-кеи" дословно переводится как "шагомер 10 тысяч шагов". Никакого
научного обоснования у такого количества шагов не было - просто 10 тысяч
считается счастливым числом в Японии, чем и воспользовались маркетологи.
Современные фитнес-трекеры задают такую же "цель" - пройти за день 10
тысяч шагов.

Источник:
https://health.mail.ru/news/chem_zamenit_10_000_shagov_v_den_ischem_vyhod/

Автор:
Екатерина Данина

Вопрос 11:
   <раздатка>
   Х: [counting reward sums of outlaws he has just killed] Ten
thousand... twelve thousand... fifteen... sixteen... seventeen...
twenty-two. Twenty-two? [Y comes from behind, Х turns and shoots him
dead] ...Twenty-seven.
   </раздатка>
   Человек, игравший персонажа, которого мы пометили иксом, по
образованию является ИМ. Другой ОН ведет свой отсчет с 1843 года.
Назовите ЕГО.

Ответ:
Экономист.

Комментарий:
Клинт Иствуд, персонажа которого мы пометили иксом, по образованию
является экономистом. Другой ОН - это журнал "The Economist", известное
британское издание.

Источник:
   1. https://www.imdb.com/title/tt0059578/quotes/qt0452682
   2. https://nv.ua/style/art/kino/kosmicheskij-kovboj-desjat-luchshih-filmov-kultovogo-aktera-i-rezhissera-klinta-istvuda-1238655.html
   3. https://ru.wikipedia.org/wiki/The_Economist

Автор:
Константин Крюков

Вопрос 12:
Американке Саре Ректор было девять лет, когда в 1911 году на
принадлежащем ей участке земли нашли нефть. Сара стала стремительно
богатеть за счет ренты от нефтяной компании, в результате парламент
Оклахомы принял специальный закон, объявивший девочку ТАКОЙ. Назовите
нобелевского лауреата, в названии произведения которого упомянут ТАКОЙ.

Ответ:
[Редьярд] Киплинг.

Комментарий:
Темнокожая Сара была очень богата, в результате чего по закону была
признана белой. "Бремя белого человека" - произведение Редьярда
Киплинга.

Источник:
   1. https://en.wikipedia.org/wiki/Sarah_Rector
   2. https://ru.wikipedia.org/wiki/Бремя_белого_человека

Автор:
Наталья Самсонова

Вопрос 13:
В 2017 году компания "Адидас" выпустила спортивную обувь в ретро-стиле,
окрашенную в красный цвет. Мы не спрашиваем вас, какому событию из жизни
шести британцев она посвящена. Назовите либо точное число выпущенных пар
обуви, либо их цену в фунтах.

Ответ:
1992.

Зачет:
92.

Комментарий:
Кроссовки выпущены в честь 25-летия выпуска "Класса 92", в который
входили шесть молодых футболистов из "Манчестер Юнайтед": Дэвид Бекхэм,
Райан Гиггз, Пол Скоулз, Ники Батт, Фил и Гари Невилл. В 1992 году,
впервые за тридцать лет, "Манчестер Юнайтед" выиграл Молодежный Кубок
Англии а в 1999 году - Лигу чемпионов УЕФА.

Источник:
   1. https://www.sports.ru/football/1056899433.html
   2. https://en.wikipedia.org/wiki/The_Class_of_%2792

Автор:
Максим Скрипниченко

Вопрос 14:
В 2003 году группа "Jeff Killed John" сменила название на нынешнее. Мы
не спрашиваем, какое имя упоминается в нынешнем названии группы.
Назовите день выхода их дебютного альбома в США.

Ответ:
14 февраля.

Комментарий:
Нынешнее название группы - "Bullet For My Valentine".

Источник:
https://ru.wikipedia.org/wiki/Bullet_for_My_Valentine

Автор:
Екатерина Данина

Вопрос 15:
(pic: 20180279.jpg)
   В фанфике "Гарри Поттер и методы рационального мышления" ЕГО можно
найти в компании философа и композитора. Также его имя присутствует при
описании лестниц Хогвартса. Назовите ЕГО.

Ответ:
Эшер.

Комментарий:
В фанфике "Гарри Поттер и методы рационального мышления" несколько раз
упоминается книга "Гёдель, Эшер, Бах", а при описании постоянно
движущихся лестниц Хогвартса главный герой замечает, что даже картины
Эшера не сравнятся с этим зрелищем. На раздатке изображен альтернативный
ракурс "невозможного" треугольника, или треугольника Пенроуза, под
впечатлением от которого Эшер написал одну из самых известных своих
работ "Водопад", что должно было послужить дополнительной подсказкой.

Источник:
   1. Э.Ш. Юдковски. Гарри Поттер и методы рационального мышления.
http://flibusta.is/b/463799/read
   2. https://ru.wikipedia.org/wiki/Водопад_(литография)
   3. https://ru.wikipedia.org/wiki/Треугольник_Пенроуза
   4. http://www.bolshoyvopros.ru/questions/305598-chto-takoe-treugolnik-penrouza-i-pochemu-on-ne-suschestvuet.html

Автор:
Максим Скрипниченко

Вопрос 16:
Осенью 2017 года творческий дуэт "Life of a Craphead" организовывал в
Торонто перформанс, во время которого реплика конной статуи Эдуарда VII
сдвигалась с постамента и отправлялась в плавание по реке, после чего
статуя возвращалась на место. Один из участников дуэта утверждает, что
перформанс получил большой резонанс после некоторого события. А с чьим
именем было связано это событие?

Ответ:
[Генерал] [Роберт] Ли.

Комментарий:
Резонанс перформанс получил после событий в американском Шарлоттсвилле,
где решение властей снести памятник генералу конфедератов Роберту Ли
привело к восстанию - маршу "Объединенных правых".

Источник:
   1. https://en.wikipedia.org/wiki/Life_of_a_Craphead
   2. "Geo", 2018, N 2. - С. 31.

Автор:
Константин Крюков

Вопрос 17:
В одном из фанфиков о Гарри Поттере Сириус Блэк знакомится с футболом и
предметом своего обожания выбирает клуб из второго дивизиона за его
название. Назовите город, который представляет этот клуб.

Ответ:
Блэкпул.

Комментарий:
Сириусу Блэку пришелся по нраву клуб, созвучный с его фамилией.

Источник:
   1. http://fanfics.me/fic41204
   2. https://en.wikipedia.org/wiki/Blackpool_F.C.

Автор:
Наталья Самсонова

Вопрос 18:
В 2016 году появилась новая информация, согласно которой ОНА должна была
стать всего лишь демонстрацией силы для предотвращения захвата
артиллерийских орудий. Наступление гондорцев на Осгилиат во "Властелине
Колец" является отсылкой к НЕЙ. Назовите ЕЕ тремя словами.

Ответ:
Атака легкой кавалерии.

Зачет:
Атака легкой бригады.

Комментарий:
Во время Балаклавского сражения Крымской войны легкая британская
кавалерия была ошибочно направлена в сторону позиций русской артиллерии
и понесла тяжелейшие потери. Согласно новой информации, лорд Раглан
отдал распоряжение капитану Нолану следовать за врагом и попытаться
избежать захвата орудий. Нолан добавил в приказ слово "атаковать", что
привело к разгрому. Гондорцы тоже направили в атаку кавалерию с
аналогичными последствиями.

Источник:
   1. https://ru.wikipedia.org/wiki/Атака_лёгкой_бригады
   2. https://en.wikipedia.org/wiki/Charge_of_the_Light_Brigade

Автор:
Константин Крюков

Вопрос 19:
Отечественные лингвисты сравнивают ПЕРВЫЕ со ВТОРЫМИ, отмечая схожую
лаконичность и неформальность языковых средств, хотя между падением
популярности ПЕРВЫХ и началом использования ВТОРЫХ прошло, по разным
оценкам, более пяти веков. Мы не просим вас назвать ПЕРВЫЕ и ВТОРЫЕ.
Назовите город, в котором день ПЕРВОЙ отмечается 26 июля.

Ответ:
Новгород.

Комментарий:
ПЕРВЫЕ - берестяные грамоты, ВТОРЫЕ - SMS.

Источник:
   1. http://russkiy-na-5.ru/articles/812
   2. https://ru.wikipedia.org/wiki/Берестяные_грамоты

Автор:
Екатерина Батракова

Вопрос 20:
(pic: 20180280.jpg)
   Перед вами ОН канала "2x2". Назовите ЕГО двумя словами на одну и ту
же букву с использованием заимствования.

Ответ:
Спойлер сезонов.

Зачет:
Сезонный спойлер.

Источник:
https://www.youtube.com/watch?v=gvdKZxklYzk

Автор:
Екатерина Данина

Вопрос 21:
Деньги, выпускавшиеся во время гражданской войны в Средней Азии, как
правило, ничем реально не обеспечивались. Город Верный (нынешний Алматы)
был единственным, в котором местные большевики выпустили деньги на сумму
20 миллионов золотых рублей, указав на реверсе каждой купюры, что
кредитные билеты обеспечены товаром на сумму 40 миллионов золотых
рублей. Назовите этот товар точно.

Ответ:
Опиум.

Комментарий:
Мешки с опиумом лежали в подвалах банка в качестве обеспечения денежной
эмиссии.

Источник:
"Литературная газета", 2017, N 41.

Автор:
Валерий Клементьев

Вопрос 22:
Кавер-версия композиции группы "Aerosmith" "Walk This Way", записанная
совместно с коллективом "Run-D.M.C.", включает в себя элементы
рок-музыки и рэпа. В клипе на эту песню вокалист "Aerosmith" Стивен
Тайлер в прямом смысле слова ДЕЛАЕТ ЭТО. В начале клипа другой группы
1989 года ЭТО ДЕЛАЕТ локомотив. Что мы заменили словами "ДЕЛАЕТ ЭТО"?

Ответ:
Ломает стену.

Зачет:
Пробивает стену и прочее по смыслу.

Комментарий:
Композиция тоже сломала стену между двумя жанрами, кроме того, это было
также символическое разрушение границ между белыми и темнокожими
музыкантами. Клип "Queen" начинается с того, что локомотив пробивает
стену, выстроенную на железной дороге.

Источник:
   1. Клип на песню "Run-D.M.C." feat "Aerosmith" "Walk This Way".
   2. Клип на песню "Queen" "Breakthru".

Автор:
Константин Крюков

Вопрос 23:
В программе "Орел и решка", снятой в Квинстауне, показывают ЕГО.
Согласно мнению, высказанному в передаче, сама возможность встретить ЕГО
в Новой Зеландии обусловлена тем, что там очень тонкий озоновый слой и
нещадно палит солнце, а использование солнцезащитных средств в данном
случае проблематично. Ответьте тремя словами: кто ОН?

Ответ:
Конь в пальто.

Источник:
"Орел и решка", выпуск "Райский Куинстаун".
https://www.youtube.com/watch?v=pVX1K4XFrkw

Автор:
Екатерина Данина

Вопрос 24:
На одном из развлекательных пабликов автору вопроса попалась картинка, в
верхней части которой изображен классический скриншот из файтинга с
соответствующим призывом к действию, а в нижней - фото пяти людей,
считающих, что это обращение к ним. Автор не просит вас назвать этих
людей. Назовите страну, которую они представляют.

Ответ:
Финляндия.

Комментарий:
Надпись была "finish him", которая созвучна с фразой "finnish HIM",
которую музыканты упомянутой группы приняли на свой счет.

Источник:
https://imgflip.com/i/gyk3p

Автор:
Екатерина Батракова

Тур:
5 тур. "РТЫ" (Калуга - Москва - Ярцево)

Инфо:
Команда благодарит за помощь в работе над туром Павла Анохина, Марину
Анохину, Марианну Выдревич, Максима Мерзлякова, Сергея Митько, Анастасию
Митько, Тимура Мухаматулина, Бориса Окуня, Софью Окунь, Дмитрия Петрова
и Константина Сахарова.

Вопрос 1:
   <раздатка>
   Смотрим мы, бывалоча, с ДимБорисычем какой-нить свежий пакетец,
вздыхаем и молвим _________: - Как здорово, что нам этого не играть!
   </раздатка>
   Перед вами цитата из интервью 2011 года. С тех пор турниров с плохими
вопросами стало гораздо больше. Заполните пропуск словом с удвоенной
согласной.

Ответ:
Синхронно.

Комментарий:
Некоторые считают, что качество вопросов растет не так быстро, как
количество отыгрываемых пакетов. Большинство современных турниров
проводятся в синхронном режиме. Впрочем, иные критики утверждают, что
самый лютый трэш играется как раз на очных турнирах, просто резонанс от
них сильно меньше. Этот пакет, к счастью, больше нигде играться не
будет.

Источник:
https://abu-dli.livejournal.com/159139.html

Автор:
Владислав Король (Москва)

Вопрос 2:
В романе Стивена Кинга небольшой город неожиданно оказывается под
непроницаемым прозрачным куполом. К концу повествования воздух под
куполом становится непригодным для дыхания, и лишь нескольким жителям
чудом удается спастись. Один читатель назвал купол "ИКСОМ наоборот".
Назовите ИКС одним словом.

Ответ:
Занавес.

Комментарий:
В начале романа купол опустился, в конце поднялся - и некоторым удалось
выжить. Такой вот обратный занавес, к тому же прозрачный.

Источник:
   1. С. Кинг. Под Куполом.
   2. Суждение автора вопроса.

Автор:
Владислав Король (Москва)

Вопрос 3:
Герои романа "Любовь живет три года" - писатель Марк Марронье и его
возлюбленная Алиса. Марк - автор трех романов, в которых он
последовательно влюбляется, женится и разводится. Автору вопроса
показалось интересным совпадением еще и то, что последний текст Марка
называется... Ответьте несколькими трехбуквенными словами: как именно?

Ответ:
"Где нас нет".

Зачет:
"Там, где нас нет"; "Нас нет".

Комментарий:
Другие Алиса и "всего лишь писатель" Марк - герои альбома Оксимирона
"Горгород". "Где нас нет" - произведение оксимироновского Марка,
написанное незадолго до гибели, так же называется заключительный трек
альбома, ставший хитом. Для романа о расставании это название вполне
подходит. А влюбленность и семейную жизнь, по мнению автора вопроса,
аллегорически описывают четвертый и шестой треки "Горгорода".

Источник:
   1. Ф. Бегбедер. Любовь живет три года.
http://flibusta.is/b/243097/read
   2. https://ru.wikipedia.org/wiki/Горгород

Автор:
Владислав Король (Москва)

Вопрос 4:
Внимание, в вопросе пропущены два слова.
   Чтобы восстановить силы, герой одной песни налёг ПРОПУСК, а затем
потребовал второго. Видимо, котлеток или макарончиков. Из букв,
содержащихся в двух пропущенных словах, можно составить название
профессии. Назовите эту профессию.

Ответ:
Наборщик.

Комментарий:
Пропущены слова "на борщик" - их можно составить из букв слова
"наборщик", прямо как в игре "Наборщик".

Источник:
   1. Валерий Шунт, песня "Заочницы".
https://otvet.mail.ru/question/185657427
   2. https://ru.wikipedia.org/wiki/Наборщик

Автор:
Владислав Король (Москва)

Вопрос 5:
Один не очень богатый человек предложил такой способ употребления
алкоголя: сначала для градуса залпом выпить полстакана разбавленного
дешевого спирта, а затем для вкуса смаковать рюмку какого-нибудь более
благородного напитка. Этот способ он назвал "АЛЬФОЙ". Три года назад
"АЛЬФА" принимала участие в Окском марафоне - да и сейчас, вероятно,
несколько АЛЬФ играют этот вопрос. Назовите АЛЬФУ тремя словами,
начинающиеся на соседние буквы алфавита.

Ответ:
Разведенка с прицепом.

Комментарий:
Так иногда называют одиноких матерей с ребенком. Немного старого доброго
сексистского юмора этот пакет точно сильно хуже не сделает.

Источник:
   1. ЛОАВ.
   2. https://koenig-away.livejournal.com/647006.html
   3. Общие соображения о законах популяционной биологии и
демографической статистике Российской Федерации.

Автор:
Владислав Король (Москва)

Вопрос 6:
Внимание, ИКС - это замена.
   Еще до исследования человеческой сексуальности Зигмунд Фрейд изучал
половую жизнь ИКСОВ. Кому недавно предложили рекламировать средство от
ИКСОВ?

Ответ:
Гнойный.

Зачет:
Вячеслав Машнов и многочисленные псевдонимы.

Комментарий:
Первая научная статья Фрейда была написана после исследования речных
угрей. Средство от угрей на лице предложили рекламировать Гнойному.
Отказался.

Источник:
   1. http://cpp-p.ru/biblioteka/zigmund-freyd/
   2. https://meduza.io/feature/2017/11/02/ya-chestno-hochu-zarabotat-na-kvartirku-v-chem-problema

Автор:
Алексей Пономарёв (Калуга)

Вопрос 7:
Один из первоначальных вариантов названия был "СупротИвЪ", но в итоге
верх одержало другое название. Напишите его.

Ответ:
"Versus [Battle]".

Зачет:
"Версус".

Комментарий:
В баттле латинского и древнерусского названия победило первое.

Источник:
https://www.youtube.com/watch?v=1pAyBTpHYJs&t=2m11s

Автор:
Ким Аристов (Тула - Москва)

Вопрос 8:
   <раздатка>
   безумное, ______, мимолетное
   </раздатка>
   В одной композиции подряд звучат три прилагательных. Напишите второе
из них, пропущенное на раздаточном материале.

Ответ:
Злое.

Комментарий:
"Безумное, злое, мимолетное сеять", в противоположность разумному,
доброму, вечному.

Источник:
Композиция "Есть предложение" рэп-коллектива "Красные пидарасы".

Автор:
Владислав Король (Москва)

Вопрос 9:
В песне "Красный мед войны" белая АЛЬФА рифмуется с именем египетской
богини ХАтор, а зеленый ИКС - с грОбом. Назовите АЛЬФУ и ИКС.

Ответ:
Вата, укроп.

Комментарий:
"И жужжат шрапнели, словно пчелы, собирая ярко-красный мед" -
гумилёвский образ, но эта песня написана в 2014 году.

Источник:
http://hm6.ru/branimir/75757-branimir-krasnyj-med-vojny.html

Автор:
Владислав Король (Москва)

Вопрос 10:
Во время суда над белым генералом Анатолием Пепеляевым обвинители
утверждали, что ему по нраву только такой ИКС: японские сапоги,
английские шинели, американское оружие. Напишите слово, которое мы
заменили ИКСОМ.

Ответ:
Интернационал.

Источник:
Л.А. Юзефович. Зимняя дорога. Генерал А.Н. Пепеляев и анархист И.Я.
Строд в Якутии. 1922-1923. http://flibusta.is/b/421781/read

Автор:
Сергей Печёнкин (Ярцево)

Вопрос 11:
За народнические убеждения Анатолия Пепеляева презрительно называли
ТАКИМ генералом. Роль ТАКОГО исполнил в 2011 году нидерландский актер
Рутгер Хауэр. Какое слово мы заменили словом "ТАКОЙ"?

Ответ:
Мужицкий.

Комментарий:
Художник Питер Брейгель Старший носил прозвище "Мужицкий".

Источник:
   1. Л.А. Юзефович. Зимняя дорога. Генерал А.Н. Пепеляев и анархист
И.Я. Строд в Якутии. 1922-1923. http://flibusta.is/b/421781/read
   2. https://ru.wikipedia.org/wiki/Мельница_и_крест
   3. https://ru.wikipedia.org/wiki/Брейгель,_Питер_(Старший)

Автор:
Сергей Печёнкин (Ярцево)

Вопрос 12:
Летом 2015 года перед просмотром фильма о зомби автор вопроса выпил пиво
отечественной пивоварни, производство которого было приурочено к
столетию известного события. Напишите название из двух слов, под которым
известно это событие.

Ответ:
Атака мертвецов.

Комментарий:
Событие - контратака русских войск при обороне крепости Осовец в 1915
году.

Источник:
   1. ЛОАВ.
   2. http://russiancraftbeer.ru/style/1915
   3. https://ru.wikipedia.org/wiki/Атака_мертвецов

Автор:
Денис Карабанов (Калуга)

Вопрос 13:
(pic: 20180281.jpg)
   Догадавшись, какая фамилия пропущена на раздаточном материале,
напишите два слова, пропущенных в выдуманной автором вопроса теме
курсового проекта.

Ответ:
1000 чертей.

Зачет:
Тысяча чертей.

Комментарий:
Пропущена фамилия Боярский.

Источник:
ФАВ.

Автор:
Алексей Пономарёв (Калуга)

Вопрос 14:
Внимание, в вопросе есть замена.
   В фильме "Ирония судьбы, или С легким паром!" Ипполит говорит, что
Женю, находящегося в бессознательном состоянии, посадили в самолет, как
АЛЬФУ. В какой игре используют пять АЛЬФ?

Ответ:
Городки.

Комментарий:
АЛЬФА - это чурка. В то время фраза "посадили, как чурку" звучала не
двусмысленно. Пять деревянных чурок используются в городках. Кстати,
среди городошных фигур есть и "самолет".

Источник:
   1. http://www.pravmir.ru/grammar-natsi-zhivut-protsvetayut-i-razmnozhayutsya/#ixzz3UXcn0DPw
   2. https://ru.wikipedia.org/wiki/Городки

Автор:
Алексей Пономарёв (Калуга)

Вопрос 15:
Недавно депутат Госдумы Дмитрий Носов анонсировал выход своей книги под
названием "Моя борьба". Согласно шутке автора вопроса, ОН бывшего
спортсмена Носова оказался нацистом. Ответьте двумя словами: кто ОН?

Ответ:
Литературный негр.

Комментарий:
Кстати, начинающие политики, авторы одноименных автобиографических книг
- почти ровесники.

Источник:
https://www.sports.ru/tribuna/blogs/greatness/1499139.html

Автор:
Денис Карабанов (Калуга)

Вопрос 16:
В одной экранизации раздосадованные подростки разбивают коллекцию
"снежных шаров" с достопримечательностями разных городов. Во что они ими
бросаются?

Ответ:
Купол.

Комментарий:
Окруженный прозрачным куполом город и сам напоминает снежный шар, только
без снега. Подростки огорчены тем, что не могут посетить другие города.

Источник:
Телесериал "Under the Dome", s01e08, 26-я минута.

Автор:
Владислав Король (Москва)

Вопрос 17:
(pic: 20180282.jpg)
   Какое прилагательное закрыто на этом коллаже?

Ответ:
Нетрадиционный.

Комментарий:
На обычных пачках, где изображены парень и девушка, пишут
"традиционный".

Источник:
(pic: 20180283.jpg)

Автор:
Александр Финошин (Калуга)

Вопрос 18:
Когда автор вопроса пошел в сауну в мужской компании с друзьями из
Мьянмы, его удивило, что они парились в плавках, хотя русские были
голыми. Еще больше автора вопроса удивило, что в своей стране они ходят
в сауну без плавок только с девушками. На вопрос, почему у них так
принято, ответ был такой: то, что находится под плавками, ПРОПУСК.
Согласно проведенному британской телекомпанией опросу, более трети
опрошенных женщин делают макияж ПРОПУСК. Заполните пропуск двумя
словами.

Ответ:
Для женщин.

Зачет:
Для девушек; для леди; for ladies; по смыслу.

Комментарий:
То, что под плавками - не для мужчин, а для женщин. Автору вопроса есть
над чем подумать.

Источник:
   1. ЛОАВ.
   2. https://www.kp.ru/daily/25723.6/2715730/

Автор:
Александр Финошин (Калуга)

Вопрос 19:
В фильме Пьера Паоло Пазолини поэт созерцает обнаженных юношей и
сочиняет строчку о дивных ИКСАХ Багдада. На одном из ИКСОВ Касабланки
периодически загорается лазерный луч. Что мы заменили ИКСОМ?

Ответ:
Минарет.

Комментарий:
Луч указывает направление на Каабу в Мекке во время намаза.

Источник:
   1. Х/ф "Цветок тысячи и одной ночи" (1974), реж. Пьер Паоло Пазолини.
   2. https://www.colors.life/post/239270/

Автор:
Алексей Пономарёв (Калуга)

Вопрос 20:
В начале книги ОН обезглавил оленя и отрубил женщине кисть руки.
Назовите ЕГО.

Ответ:
Купол.

Комментарий:
В начале романа Стивена Кинга, как вы помните, купол опустился. Многие
пострадали: например, в купол сразу же врезался самолет.

Источник:
С. Кинг. Под Куполом.

Автор:
Владислав Король (Москва)

Вопрос 21:
Издание "Inside Edition" организовало необычную встречу. На ней
присутствовали двадцать человек, ранее не знакомых друг с другом.
Ответьте двумя словами: кем когда-то подрабатывал старший из них?

Ответ:
Донор спермы.

Комментарий:
Издание организовало встречу 19 человек со всего света с их
биологическим отцом - художником Майклом Рубино.

Источник:
https://daily.afisha.ru/infoporn/6849-hudozhnik-podrabatyval-donorom-spermy-vot-ego-pervaya-vstrecha-s-19-detmi/

Автор:
Даниил Марченко (Калуга)

Вопрос 22:
Считается, что ЭТО - своеобразная плата за прямохождение. В рекламе
средства для борьбы с ЭТИМ хотели использовать песню "Огненное кольцо".
Назовите ЭТО.

Ответ:
Геморрой.

Комментарий:
Рекламщикам название песни показалось забавным применительно к симптомам
геморроя. Однако наследники Джонни Кэша запретили использование песни
"Ring of Fire" в этой рекламе. Другие млекопитающие не болеют геморроем,
поэтому считается, что прямохождение способствует развитию этой болезни.

Источник:
   1. http://www.gemorroi.com/prichiny/pryamohozhdenie.php
   2. https://www.foxnews.com/story/cash-family-draws-line-around-ring-of-fire

Автор:
Сергей Осипов (Калуга)

Вопрос 23:
После того как личинки пчелиной моли съедают весь воск в пчелином
гнезде, они начинают есть собственные экскременты. После того как
кончаются запасы старых экскрементов, личинки принимаются за новые
выделения и так далее. Описывая этот способ существования, Александр
Бернацкий упомянул ЕГО. Назовите ЕГО на латыни.

Ответ:
Перпетуум мобиле.

Зачет:
Perpetuum mobile.

Источник:
   1. А.С. Бернацкий. 100 великих рекордов животных.
http://flibusta.is/b/334006/read
   2. https://ru.wikipedia.org/wiki/Вечный_двигатель

Автор:
Алексей Пономарёв (Калуга)

Вопрос 24:
[Раздаточный материал - пронумерованные спичечные коробки, обклеенные
скотчем (по одному на команду). Раздать перед чтением вопроса. Ведущему
объявить: вскрывать раздаточный материал до зачтения ответа нельзя.]
   ЕГО суммарная масса - 2 килограмма 700 граммов. Так что, видимо,
работа по ЕГО созданию заняла несколько дней. Назовите ЕГО
по-итальянски, по-английски, по-французски, по-немецки или по-русски.

Ответ:
Дерьмо художника.

Зачет:
Merda d'artista; Artist's Shit; K&uuml;nstlerschei&szlig;e; Merde
d'artiste.

Комментарий:
Известный арт-объект, созданный Пьеро Мандзони, состоял из 90
пронумерованных баночек с 30 граммами содержимого в каждой. Что в них
находится на самом деле, достоверно неизвестно. Говорят, гипс.

Источник:
   1. https://ru.wikipedia.org/wiki/Дерьмо_художника
   2. Созданный автором вопроса арт-объект из 30 пронумерованных
коробочек по 10 граммов содержимого в каждой.

Автор:
Владислав Король (Москва)

Тур:
6 тур. "Благоест" (Коломна)

Редактор:
Валерия Комаровская

Вопрос 1:
После гражданской войны США практически перестали строить современные
корабли. А их соседи в Латинской Америке, напротив, стали активно
закупать новейшие "эльсвикские крейсеры". В результате указом Конгресса
от 3 марта 1883 года было решено строить так называемый "Новый Флот".
Его первыми кораблями стали бронепалубные крейсеры "Чикаго", "Бостон" и
"Атланта", а также посыльное судно "Дельфин". А как острословы называли
эту эскадру?

Ответ:
"ABCD".

Зачет:
"ABC"; "Азбука"; "Алфавит".

Источник:
https://ru.wikipedia.org/wiki/Бронепалубные_крейсера_типа_%C2%ABАтланта%C2%BB

Автор:
Александр Кожемякин

Вопрос 2:
На спортивном сайте championat.com в названии статьи, рассказывающей о
том, как поздравляли чемпионку мира и Европы Евгению Медведеву с днем
рождения поклонники и соперницы, использован неологизм, одновременно
указывающий на значимость произошедшего события и на сферу деятельности
спортсменки. Напишите этот неологизм, являющийся сложным словом.

Ответ:
Совершеннозимняя.

Источник:
https://www.championat.com/other/article-3321195-figuristka-evgenija-medvedeva-den-rozhdenija-18-let-pozdravlenija-podarki.html

Автор:
Сергей Бирюков

Вопрос 3:
По преданию, когда в VII веке персы вторглись в пределы Византийской
империи, то вознамерились разрушить один знаменитый храм. Однако, увидев
внутри мозаику, изображающую людей в характерных персидских головных
уборах, они решили, что это здание каким-то образом связано с их
собственной религией и пощадили его. Назовите город, в котором этот храм
стоит и поныне.

Ответ:
Вифлеем.

Комментарий:
Мозаика в базилике Рождества в Вифлееме изображала поклонение волхвов. В
греческом оригинале Евангелия волхвы названы магами. Изначально маги -
это жрецы в Древней Персии.

Источник:
   1. А.Ю. Низовский, М.В. Губарева. 100 великих храмов.
http://flibusta.is/b/236832/read
   2. https://ru.wikipedia.org/wiki/Маги

Автор:
Иван Морозов

Вопрос 4:
В мире есть несколько примеров использования концепции "ТАКОЙ ИКС".
Например, регулировка яркости уличных фонарей в Амстердаме, новая
автобусная сеть в Барселоне или специальные датчики, установленные по
всему Сантандеру. Персонажу одного произведения, чтобы поверить, что он
ТАКОЙ, потребовалась весьма необычная смесь. В ответе укажите не менее
двух ее составляющих.

Ответ:
Отруби, иголки, булавки (любые два из трех).

Зачет:
Штырьки вместо иголок.

Комментарий:
В версии Баума Страшила в качестве мозгов получает отруби и булавки
(игра слов: bran - отруби, pins - булавки или штырьки, brains - мозги).
Несмотря на то что Страшила стал носить прозвище Мудрый, поверить он
хотел не в свою мудрость (о таком большом прогрессе он и не мечтал), а в
то, что он стал просто умным.

Источник:
   1. https://ru.wikipedia.org/wiki/Умный_город
   2. https://ru.wikipedia.org/wiki/Страшила_(Волшебник_Изумрудного_города)

Автор:
Сергей Бирюков

Вопрос 5:
Внимание, в вопросе есть замены.
   На одной фотографии, сделанной во время матча Шотландия - СССР в
рамках чемпионата мира по футболу 1982 года, шотландские болельщики
держат в руках плакат с самоироничной надписью "ИКС против ИГРЕКА".
Назовите ИКС и ИГРЕК двумя рифмующимися словами.

Ответ:
Алкоголизм, коммунизм.

Источник:
https://pikabu.ru/story/chempionat_mira_1982_goda_ispaniya_match_shotlandiya__sssr_5492223

Автор:
Александр Романкин

Вопрос 6:
Прослушайте цитату: "Не затем пришел я сюда, чтобы смотреть на те ужасы,
которые наполняют твое царство, не затем, чтобы увести, подобно раку,
стража твоего...". В словах Орфея мы пропустили три буквы. Какие?

Ответ:
Г, е, л.

Комментарий:
Орфей подобно Гераклу собирался увести стража царства Аида -
трехголового Кербера (Цербера).

Источник:
Н.А. Кун. Легенды и мифы Древней Греции. http://flibusta.is/b/67461/read

Автор:
Николай Кузнецов

Вопрос 7:
   <раздатка>
   В одной поучительной истории рассказывалось, как одна женщина
лишилась своего ребенка, после чего, в отчаянии придя в часовню, отняла
за это у Божьей Матери младенца Иисуса. Когда же ребенок вскоре снова
вернулся к этой женщине, то она с благодарностью отнесла младенца Иисуса
обратно.
   </раздатка>
   Интересно, что книжник XVII века, переводивший эту историю на русский
язык, принципиально заменил в ней пропущенное нами слово другим. Какое
слово латинского происхождения мы пропустили в раздаче?

Ответ:
Статуя.

Комментарий:
В католическом оригинале женщина отломила младенца Иисуса от статуи. В
русской православной вере статуи не почитались, поэтому переводчик
заменил статую иконой, только не объяснил, как можно отнять
нарисованного младенца.

Источник:
М. Майзульс. Наказание святых: благочестивое богохульство в Средние века
и в раннее Новое время. // "Государство, религия, церковь в России и за
рубежом", 2017, N 2. - С. 26-28.

Автор:
Иван Морозов

Вопрос 8:
В одном американском сериале полицейские детективы приходят к свидетелю,
страдающему гипертимезией, чтобы расспросить его о событиях,
произошедших с ним несколько лет назад. Перед своим уходом от свидетеля
детективы НЕ ДЕЛАЮТ ЭТОГО. ДЕЛАТЬ ЭТО представителям Передней Азии надо
правой рукой, а, например, представителям Тайваня или Сингапура - двумя
руками с поклонами. Что мы заменили словами "ДЕЛАТЬ ЭТО"?

Ответ:
Давать визитку.

Комментарий:
Гипертимезия, гипертиместический синдром - способность личности помнить
и воспроизводить предельно высокое количество информации о ее
собственной жизни, исключительная автобиографическая память. Поэтому
полицейские считают бессмысленным просить человека: "Позвоните, если
вспомните что-то еще". При вручении партнерам из Азии (например, Япония,
Сингапур, Тайвань) визитку подают двумя руками с поклоном. Чем важнее
персона, тем больше поклон. Представителям Передней Азии визитку (как и
подарки, пищу) вручают только правой рукой (по местным традициям левая
рука считается грязной).

Источник:
   1. Телесериал "Помнить всё", s02e04.
   2. http://chernetskiy.ru/blog/111
   3. https://ru.wikipedia.org/wiki/Гипертимезия

Автор:
Валерия Комаровская

Вопрос 9:
Внимание, в вопросе есть замена.
   Согласно Википедии, три первых участника московской рок-группы "Йота"
впервые встретились на первой репетиции в начале осени прошлого века,
что, по одной из версий, послужило основой для названия группы. Стоит
отметить, что на русском языке название группы неотличимо на слух от
названия седьмого альбома певицы Лары Фабиан, который логичнее было бы
назвать немного иначе. Мы не просим восстановить замену. Напишите
максимально точную дату первой репетиции группы "Йота".

Ответ:
9 сентября 1999 года.

Зачет:
Любые формы записи правильной даты.

Комментарий:
Начало осени - это сентябрь - девятый месяц в году, "йота" - девятая
буква греческого алфавита, а седьмой альбом Лары Фабиан называется "9",
хотя логичнее его было бы озаглавить "7". Группа "Девять" на первую
репетицию собралась 9 сентября 1999 года, поэтому число "9" стало столь
символичным.

Источник:
   1. https://ru.wikipedia.org/wiki/Девять_(группа)
   2. https://www.larafabian.com/bio

Автор:
Александр Романкин

Вопрос 10:
Внимание, в вопросе есть замены.
   Заболевшая героиня одного сериала просит своего кавалера не
приближаться к ней, ведь у нее АЛЬФА. Удивленный кавалер переспрашивает:
ТАКАЯ АЛЬФА или СЯКАЯ? "ТАКАЯ, - вздыхает героиня и уточняет, - хотя я
даже на сквозняке не была". Что мы заменили словами "ТАКАЯ АЛЬФА"?

Ответ:
Ветряная оспа.

Источник:
Телесериал "Друзья", s02e23, 7-я минута.

Автор:
Валерия Комаровская

Вопрос 11:
Не так давно в Фейсбуке была опубликована жизнеутверждающая история про
двух мальчиков, согласно которой пятилетний Джэкс Роузбуш и его лучший
друг Редди хотели разыграть своего учителя, сделав себе одинаковые
прически. Ребята надеялись, что трюк сработает, и учитель не сможет
отличить одного от другого, однако их розыгрыш оказался невозможным по
одной очевидной причине. Эта же причина поставила в замешательство героя
карикатуры, на которой его товарищ просит передать ему... Ответьте тремя
словами: что?

Ответ:
Карандаш телесного цвета.

Зачет:
Фломастер/маркер/ручку телесного цвета.

Комментарий:
У ребят разный цвет кожи. История о Джэксе и Редди преподносилась как
история о двух мальчиках, которые еще не стали жертвами ненависти,
сегрегации и дискриминации.

Источник:
   1. https://medialeaks.ru/0303mms-dva-malchika-hoteli-zaputat-uchitelya-pridya-s-odinakovyimi-pricheskami-a-v-itoge-prepodali-vsem-vazhnyiy-urok/
   2. http://joyreactor.cc/post/3322534

Автор:
Александр Романкин

Вопрос 12:
Когда жившая в XIII веке византийская принцесса Феодора ушла в
монастырь, то увлеклась там определенным занятием. Однажды она написала
патриарху Георгию, что хотела бы воспользоваться помощью одного из его
специалистов. Практичный же патриарх в ответ посоветовал ей отложить
свое занятие до Пасхи. А что это было за занятие?

Ответ:
Переписывание книг.

Зачет:
Копирование манускриптов; синонимичные ответы.

Комментарий:
Принцесса просила патриарха, чтобы тот прислал ей своего писца для
переписывания сочинений Демосфена. Но патриарх намекнул принцессе, что
во время Великого поста, когда мясо не употребляют в пищу, пергамен
очень дорог, и рекомендовал подождать до окончания поста, т.е. до Пасхи.

Источник:
С.А. Иванов. Прогулки по Стамбулу в поисках Константинополя.
https://books.google.ru/books?id=c2g6DAAAQBAJ&pg=PA421#v=onepage&q&f=false

Автор:
Иван Морозов

Вопрос 13:
В декабре 1941 года хостес Рут Ли вышла позагорать на пляж Майами.
Ответьте двумя словами: что она установила рядом со своим шезлонгом?

Ответ:
Флаг Китая.

Комментарий:
Таким образом Рут наглядно демонстрировала, что она не японка.

Источник:
https://www.reddit.com/r/HistoryPorn/comments/69ifgb/ruth_lee_a_hostess_at_a_chinese_restaurant_flies/

Автор:
Валерия Комаровская

Вопрос 14:
В одном из романов Валентина Пикуля описана открытка с язвительным
содержанием. В левой ее части был изображен ПРОПУСК1 с метром в руках,
измеряющий калибр снаряда, а в правой части был представлен унылый
ПРОПУСК2, который аршином измерял калибр тайного удилища. ПРОПУСК1 и
ПРОПУСК2 различаются только именем. Заполните оба пропуска.

Ответ:
Вильгельм II и Николай II.

Комментарий:
Описана одна из открыток, которыми немцы забрасывали русские позиции для
подрыва боевого духа. На ней Вильгельм II измерял на открытке калибр
снаряда, а Николай II, опустившись на колени, измерял тайное удилище
Распутина. Слова "метр" и "аршин" должны подсказать, что нужно искать
представителей двух стран, одна из которых - Россия. Валентин Пикуль
известен романами, описывающими реалии военных лет.

Источник:
В.С. Пикуль. Три возраста Окини-сан. http://flibusta.is/b/76721/read

Автор:
Николай Кузнецов

Вопрос 15:
Согласно мифологии индуизма, Чандра взял в жены 27 дочерей Дакши.
Однажды сестры пришли к отцу и сказали, что Чандра уделяет внимание
только одной из них - Рохини. Рассердившийся Дакша проклял зятя,
пообещав, что тот будет таять на глазах, пока не исчезнет совсем.
Впоследствии Чандре удалось получить возможность временно возвращать
свои силы, совершив омовение в священной реке. Что на санскрите означает
"чандра"?

Ответ:
Луна.

Комментарий:
С Дакшей связан ряд мифологических мотивов и сюжетов, в которых
подчеркиваются его участие в творении и его роль отца. Данная легенда
дает мифологическое объяснение убыванию и росту лунного диска на небе, а
также длине лунного месяца.

Источник:
   1. https://ru.wikipedia.org/wiki/Чандра
   2. https://ru.wikipedia.org/wiki/Дакша

Автор:
Сергей Бирюков

Вопрос 16:
Внимание, в вопросе есть замена.
   На одном антивоенном баннере "СНЕГОВИК" поверх двух букв в названии
известного фильма, второстепенным героем которого он является, рисует
две другие буквы. Мы не спрашиваем, что мы заменили словом "СНЕГОВИК".
Пользуясь методом мозгового штурма, напишите, какие два слова в итоге
получились на баннере.

Ответ:
STOP WARS.

Комментарий:
(pic: 20180284.jpg)
   "Снеговик" - одно из прозвищ имперских штурмовиков, которые им дали
члены Альянса (наряду с "белоголовые", "пластиковые солдаты", "мальчики
в белом", "ведроголовые"). Штурмовик написал "STOP WARS", использовав
название фильма "STAR WARS".

Источник:
   1. https://pikabu.ru/story/stop_wars_4968064
   2. http://ru.starwars.wikia.com/wiki/Штурмовик

Автор:
Александр Романкин

Вопрос 17:
На постановочной фотографии, посвященной нашумевшему событию 2012 года,
на столе лежат лоскутки ткани, ножницы и записка следующего содержания:
"Ушла в храм. Вернусь не скоро". Одна из женщин, которая могла бы
написать эту записку, вернулась домой спустя 14 месяцев, ведь она, к
своей радости, ПОТЕРЯЛА ПАМЯТЬ. Какие три слова мы заменили словами
"ПОТЕРЯЛА ПАМЯТЬ"?

Ответ:
Попала под амнистию.

Комментарий:
(pic: 20180285.jpg)
   Речь идет об участницах группы "Pussy Riot", которые были приговорены
к двум годам лишения свободы, но были выпущены по амнистии в честь
20-летия Конституции РФ. Слова "амнистия" (от греч. "забвение,
прощение") и "амнезия" (от греч. "память") - в некотором смысле
родственные. Слово "нашумевшее" - намек на панк-молебен.

Источник:
   1. https://vk.com/by_duran?w=wall-25336774_1380
   2. https://www.gazeta.ru/social/2013/12/23/5816125.shtml

Автор:
Александр Романкин, в редакции Николая Кирина

Вопрос 18:
В одном американском сериале два героя, Эл и Кэрри, размышляют о жизни
на необитаемом острове. Эл говорит Кэрри, что непременно хотел бы взять
ее с собой на остров, ведь она запоминает абсолютно всё и обо всём, а
значит, является ходячим Дэниэлом Рэдклиффом. Какие два слова мы
заменили в вопросе?

Ответ:
Швейцарский нож.

Комментарий:
Намекая на универсальность Кэрри, обладающей многими знаниями в
различных областях, Эл сравнивает ее с универсальным швейцарским ножом.
Забавно, что Кэрри сравнение обижает, хотя до выхода фильма "Человек -
швейцарский нож" еще почти два года. Ну а Дэниэл Рэдклифф играет в этом
фильме человека - швейцарский нож.

Источник:
   1. Телесериал "Помнить всё", s03e08.
   2. https://ru.wikipedia.org/wiki/Человек_%E2%80%93_швейцарский_нож

Автор:
Валерия Комаровская

Вопрос 19:
После развала СССР разные люди на флоте неоднократно организованно
совершали некоторые преступные деяния. Например, в одной статье
сообщалось, что офицер со своим сообщником ПРОПУСК1 с подводной лодки на
Северном флоте. Можно сказать, что Диомед и Одиссей мудро предрешили
судьбу Трои, когда, проникнув в осажденный город, ПРОПУСК2. Оба пропуска
состоят из двух слов и в различных источниках могут различаться
капитализацией. Заполните любой из них.

Ответ:
Похитили палладий.

Зачет:
По слову "Палладий" или "палладий" и глаголу со значением "похитить,
украсть, своровать".

Комментарий:
Диомед и Одиссей похитили Палладий - статую Афины-Паллады. Палладий в
переносном смысле - оберег, талисман. Оставшись "беззащитной", по мнению
Гомера, Троя была обречена. После развала СССР с подводных лодок
происходили хищения палладия - ценного металла, но никакого отношения к
этим деяниям богиня военной стратегии и мудрости не имела. Слова
"организованно" и "мудро" - это намеки на Афину-Палладу - богиню
организованной войны и мудрости.

Источник:
   1. http://www.businesspress.ru/newspaper/article_mId_34_aId_5887.html
   2. https://ru.wikipedia.org/wiki/Диомед

Автор:
Николай Кузнецов, в редакции Николая Кирина

Вопрос 20:
Согласно различным сонникам, ОНА - олицетворение нечистоплотности,
лицемерия, выражение образа навязчивого человека из реальности, с
которым вам бы очень хотелось порвать любые отношения. Однако в
известном произведении Николая Васильевича у нее таки нашелся обожатель
с самыми серьезными намерениями, даже несмотря на ее, возможно,
чрезмерно болтливый характер. А как ее звали в этом произведении?

Ответ:
Цокотуха.

Зачет:
Муха-цокотуха.

Комментарий:
ОНА - это муха, а ее жених - комарик. "Муха-Цокотуха" - произведение
Николая Васильевича Корнейчукова, известного под псевдонимом Корней
Иванович Чуковский. Согласно словарю синонимов, цокотуха - "тараторка,
болтушка, щебетунья, чечетка, балалайка бесструнная, говорунья, язык без
костей, трындычиха, сорока, болтунья, таранта, стрекоза, длинный язык,
трещетка, стрекотунья".

Источник:
   1. http://pros0n.ru/m/son-prisnilsja-muha.html
   2. https://45parallel.net/korney_chukovskiy/mukha-tsokotukha.html
   3. http://www.chukovskiy.ru/bio.html
   3. https://dic.academic.ru/dic.nsf/dic_synonims/195007/

Автор:
Александр Романкин, в редакции Николая Кирина

Вопрос 21:
Внимание, в вопросе есть замена.
   В начале 2017 года очередным бьюти-трендом стали АЛЬФЫ. Те, кто
АЛЬФАМИ обладали от природы, старались всячески их подчеркивать. Другим
же пришлось их рисовать, ну а самые смелые отправились в тату-салон.
Так, мастер из Мичигана Джессика Кнапик даже предложила располагать
тату-АЛЬФЫ не в хаотичном порядке, а виде созвездия. Из источника 1971
года мы знаем, что заглавному персонажу чуть более раннего произведения
явно не требовалось себе ничего рисовать, чтобы соответствовать данному
бьюти-тренду. Назовите имя этого персонажа.

Ответ:
Антошка.

Комментарий:
АЛЬФЫ - веснушки. Словарь синонимов русского языка говорит, что
конопушки - синоним веснушек. Антошка - главный герой мультфильма 1969
года - является визитной карточкой режиссера Леонида Носырева. Также он
использовал его образ в мультфильмах "Рыжий, рыжий, конопатый", "Два
веселых гуся" и "Фантазеры из деревни Угоры".

Источник:
   1. https://www.cosmo.ru/beauty/news/10-11-2017/novyy-byuti-trend-astrovesnushki-zvezdnaya-karta-na-tvoem-nosu/
   2. https://ru.wikipedia.org/wiki/Рыжий,_рыжий,_конопатый
   3. https://dic.academic.ru/dic.nsf/dic_synonims/64548/

Автор:
Александр Романкин, в редакции Валерии Комаровской

Вопрос 22:
По наиболее распространенной версии, ОНИ получили свое название от
греческого слова, означающего "чистые". Однако один средневековый
трактат утверждает, что ОНИ получили свое название от слова "кот, чей
зад они целовали и в чьем облике перед ними представал Сатана". В
Северной Франции ИХ называли бУграми, т.е. болгарами. Назовите ИХ.

Ответ:
Катары.

Комментарий:
Название средневековой ереси катаров происходит, как считается, от
греческого "катарос" - чистый. Другую этимологию породило созвучие слов
"катары" и "катус" (кот). А болгарами катаров считали за схожесть
взглядов с болгарской ересью богомилов.

Источник:
   1. https://ru.wikipedia.org/wiki/Катары
   2. https://ru.wikipedia.org/wiki/Срамный_поцелуй
   3. http://www.pravenc.ru/text/149507.html

Автор:
Николай Кузнецов

Вопрос 23:
Проповедник Яков де Витри записал историю об одном сарацине, которому
было уже больше шестидесяти лет, но который никогда не покидал родного
Дамаска, из города не выходил и не имел к тому никакого желания. Когда
об этом сообщили султану, тот позвал сарацина к себе. Убедившись, что
всё сказанное про этого человека - правда, султан принял некое
постановление. Вскоре старый сарацин впервые вышел из города, а казна
султана увеличилась на большую сумму. Что постановил султан?

Ответ:
Запретил этому сарацину покидать город.

Зачет:
По смыслу.

Комментарий:
На примере этого сарацина проповедник Яков де Витри показал, что
запретный плод может быть так сладок, что и угроза крупного штрафа за
нарушение запрета не будет препятствием.

Источник:
Н.С. Горелов. Царствие Небесное: Легенды крестоносцев XII-XIV веков. -
СПб.: Азбука-классика, 2006. - 4-я страница обложки.

Автор:
Иван Морозов

Вопрос 24:
Внимание, в вопросе есть замены.
   Сотрудники парка "Диснейленд" указывают посетителям на что-то строго
двумя пальцами. И это связано не только с тем, что во многих культурах
указывать одним пальцем недопустимо. Основная причина в том, что
практически на всех ТАКИХ фотографиях сам основатель компании Уолтер
Элайас Дисней постоянно куда-то указывает двумя сложенными пальцами
правой руки - средним и указательным. Фотографии пришлось сделать
ТАКИМИ, ибо Уолт Дисней был ИКСОМ, а это противоречит основным идеям
политики компании. В каком культовом сериале роль ИКСА блестяще сыграл
Уильям Б. Дэвис?

Ответ:
"Секретные материалы".

Зачет:
"The X-Files".

Комментарий:
Такие фотографии - отретушированные. Сам Дисней был заядлым курильщиком,
выкуривавшим до трех пачек в день, поэтому фотографам было сложно
улучить момент, когда бы Уолт был без сигареты. Всё это не вписывается в
образ доброго сказочника, который рисуют последователи из Disney, и
подает плохой пример детям в Диснейленде, поэтому на всех официальных
фотографиях мультипликатора сигарету в его руках заретушировали.
Курильщик - главный антагонист Малдера в сериале "Секретные материалы".
   Это был последний вопрос нашего пакета, так что все курильщики могут
перекурить.

Источник:
   1. https://bigpicture.ru/?p=989292
   2. https://ru.wikipedia.org/wiki/Курильщик_(Секретные_материалы)

Автор:
Валерия Комаровская, по идее Александра Романкина

Тур:
7 тур. "СэрПухов" (Серпухов)

Редактор:
Валерия Комаровская

Вопрос 1:
Николай Замяткин замечает, что самым сложным для иностранцев является
это слово, причем практически все учебники русского для иностранцев
начинаются именно с него. Напишите это слово.

Ответ:
Здравствуйте!

Источник:
Н.Ф. Замяткин. Вас невозможно научить иностранному языку.
http://flibusta.is/b/109742/read

Автор:
Святослав Холодов

Вопрос 2:
Внимание, в вопросе есть замена.
   В 2017 году Дед Мороз заявил, что собирается обзавестись фермой, где
будет выращивать свою IOTУ [йОту]. Ховард Маркс сравнил IOTУ с
тюльпаноманией. Что мы заменили словом "IOTУ"?

Ответ:
Криптовалюту.

Комментарий:
Дед Мороз собрался завести свою криптоферму и выращивать свою
криптовалюту - "дедульки". IOTA - это тоже криптовалюта.

Источник:
   1. https://ria.ru/economy/20171204/1510157036.html
   2. https://coinspot.io/people/investor-milliarder-hovard-marks-bitcoin-eto-shema-piramidy/
   3. https://ru.wikipedia.org/wiki/IOTA_(технология)

Автор:
Святослав Холодов

Вопрос 3:
Внимание, в вопросе есть замена.
   В ситкоме "Вице-президент" находящегося в коме больного сравнивают с
Большой Косой. В фильме "И целого мира мало" под косу замаскированы
пулемет с огнеметом. Какое слово мы заменили словом "коса"?

Ответ:
Волынка.

Комментарий:
Кроме широко известной литовки существует менее распространенная коса
волынка.

Источник:
   1. Телесериал "Вице-президент", s01e01.
   2. Х/ф "И целого мира мало" (1999), реж. Майкл Эптед.
   3. https://ru.wikipedia.org/wiki/Коса_(инструмент)

Автор:
Святослав Холодов

Вопрос 4:
В мини-сериале "Наводчица" главным героям приходится скрываться по лесам
от преследователей. В конце фильма, когда всё закончилось хорошо, один
из главных героев говорит, что благодаря всей этой истории они нашли
ЭТО. А где было известное ЭТО в 1970-е годы?

Ответ:
На обочине.

Комментарий:
Место для пикника.

Источник:
   1. Мини-сериал "Наводчица".
   2. https://ru.wikipedia.org/wiki/Пикник_на_обочине

Автор:
Святослав Холодов

Вопрос 5:
В оригинальном рассказе венгерского писателя ОН был черный, и
произведение должно было научить детишек бороться за права чернокожего
населения. В советском мультфильме 1976 года было решено уйти от
вопросов и рассказать сказку о романтичном герое "не от мира сего". Но
Судьба не любит, когда пытаются обмануть, и "нанесла удар ниже пояса".
Назовите этот советский мультфильм.

Ответ:
"Голубой щенок".

Комментарий:
В оригинальном рассказе венгерского писателя Дюлы Урбана Щенок был
черный, и произведение должно было научить детишек бороться за права
чернокожего населения. В советской сказке он приобрел голубой окрас. На
момент создания мультфильма слово "голубой" означало "не от мира сего"
или "романтичный, прекрасный, возвышенный" и не имело никакого
гомосексуального подтекста. Однако с начала 1990-х годов это слово
приобрело новое значение и стало устойчиво ассоциироваться с
гомосексуальной ориентацией.

Источник:
https://ru.wikipedia.org/wiki/Голубой_щенок

Автор:
Олег Холодов

Вопрос 6:
Меир Соловейчик отмечает, что при создании ПРОЛОГА автора волновала
судьба еврейского народа, который к тому моменту еще не обрел родину.
Поэтому еврейская тема пронзает ПРОЛОГ от начала и до конца, от конца и
до начала. Напишите название произведения, которое мы заменили словом
"ПРОЛОГ".

Ответ:
"Хоббит, или Туда и обратно".

Комментарий:
По данной версии, гномы - это евреи.

Источник:
http://www.isrageo.com/2016/08/18/evtayna/

Автор:
Олег Холодов

Вопрос 7:
Поэт Борис Рыжий говорил о районе, в котором он жил, что с советского
времени там почти ничего не изменилось, только надписи на заборах стали
ТАКИМИ. Какими - ТАКИМИ?

Ответ:
Англоязычными.

Зачет:
Английскими.

Источник:
Хроника из фильма Алёны Ван дер Хорст "Борис Рыжий" (2008).

Автор:
Святослав Холодов

Вопрос 8:
По мнению авторов одного чрезвычайно популярного на Ютубе ролика, ОН
является основоположником ИКСА. Это заявление основано на анализе одного
из его широко памятных творений, которое, по утверждениям создателей
ролика, "состоит из гипертрофированного самовозвеличивания". Кроме того,
в ЕГО творчестве присутствует множество штампов и шаблонов, которые
охотно используют и современные авторы. Мы не просим назвать ЕГО.
Назовите ИКС.

Ответ:
Рэп.

Комментарий:
Речь идет о Александре Пушкине и "Rhythmic African Poetry" (Ритмическая
африканская поэзия).

Источник:
   1. https://lenta.ru/news/2017/06/06/pushkinrap/
   2. https://www.youtube.com/watch?v=agfhqoWzlN8

Автор:
Святослав Холодов

Вопрос 9:
Документальный фильм Би-Би-Си о новой науке под названием "Синтетическая
биология" называется "ДЕЛАЯ ЭТО". Вячеслав Тихонов, Ума Турман и
некоторые другие "ДЕЛАЛИ ЭТО", а индийский актер Нандамури Тарака Рама
Рао СДЕЛАЛ ЭТО семнадцать раз. Какие слова мы заменили словами "ДЕЛАТЬ
ЭТО"?

Ответ:
Играть роль бога.

Комментарий:
В фильме "Андерсен. Жизнь без любви" Вячеслав Тихонов сыграл Бога. В
фильме "Приключения барона Мюнхгаузена" Ума Турман сыграла роль богини
любви Венеры и роль Розы. Индийский актер Нандамури Тарака Рама Рао
семнадцать раз играл бога Кришну.
   z-checkdb: Нандамури Тарака Рама Рао сыграл гораздо больше ролей
богов; семнадцать раз - это именно роль Кришны, см. авторский источник.

Источник:
   1. https://en.wikipedia.org/wiki/Playing_God_(2012_film)
   2. https://ru.wikipedia.org/wiki/Андерсен._Жизнь_без_любви
   3. https://ru.wikipedia.org/wiki/Приключения_барона_Мюнхгаузена_(фильм)
   4. https://www.film.ru/articles/odno-lico?page=10

Автор:
Святослав Холодов, Олег Холодов

Вопрос 10:
Телевизионный альманах, выходящий на канале "Культура", носит название
"Бермудские треугольники. Антарктида". По словам его создателей,
Бермудский треугольник является символом смелой научной гипотезы, а
Антарктида - символом научной тайны. Мы не спрашиваем, какие слова мы
заменили словами "Бермудские треугольники". Ответьте, какие два слова мы
заменили словом "Антарктида".

Ответ:
Белые пятна.

Зачет:
Белое пятно.

Комментарий:
"Черные дыры. Белые пятна".

Источник:
https://tvkultura.ru/brand/show/brand_id/20863/

Автор:
Валерия Говоруха

Вопрос 11:
На обложке классического произведения, впервые появившегося в печати в
1865 году, выпущенного в США под эгидой ООН и ЮНЕСКО, белый голубь
переносит в своем клюве некоторый элемент из одного английского слова в
другое. Мы не спрашиваем вас, что это за слова. Назовите автора
произведения.

Ответ:
Лев Толстой.

Комментарий:
Голубь переносит букву "a" из слова "war" в слово "peace". Классическое
произведение - "Война и мир". Отрывок из романа под названием "1805 год"
появился в "Русском вестнике" в 1865 году.

Источник:
   1. ЛОАВ.
   2. https://ru.wikipedia.org/wiki/Война_и_мир

Автор:
Ирина Холодова

Вопрос 12:
Внимание, в вопросе есть замена.
   Статья, посвященная жизни Булгакова, называлась "Перекур,
белобилетники! Булгаков на связи". В альбоме "Энергия" группы "Алиса"
также присутствует фраза "Перекур, белобилетники" - именно ею
заканчивается первая сторона пластинки. Какие два слова мы заменили в
вопросе?

Ответ:
Антракт, негодяи.

Комментарий:
Раньше негодяями называли людей, не годных для строевой службы.
"Антракт, негодяи!" - известная цитата из "Мастера и Маргариты".

Источник:
   1. http://lifeglobe.net/blogs/details?id=302
   2. https://reproduktor.net/gruppa-alisa/albom-energiya/
   3. http://books.rusf.ru/unzip/xussr_av/bulgam01.htm?21/50

Автор:
Святослав Холодов

Вопрос 13:
(aud: 20180012.mp3)
   Блогер Игорь Войтенко бросил вызов всем своим подписчикам, ДЕЛАЯ ЭТО
особым образом в течение ста секунд, после чего призвал их не отставать.
Какое слово мы заменили словами "ДЕЛАТЬ ЭТО"?

Ответ:
Подтягиваться.

Источник:
https://www.youtube.com/watch?v=nbUvUVGTEM0

Автор:
Святослав Холодов

Вопрос 14:
Двукратный олимпийский чемпион 1996 и 2000 годов в беге на 10 тысяч
метров Хайле Гебреселассие из Эфиопии отличался особой осанкой во время
бега. Сам спортсмен объяснял свою особенность ПЕРВОЙ. По мнению
классика, ПЕРВАЯ может являться суррогатом ВТОРОГО. Назовите ВТОРОЕ.

Ответ:
Счастье.

Комментарий:
ПЕРВАЯ - привычка. "Привычка свыше нам дана: // Замена счастию она".

Источник:
   1. https://www.kommersant.ru/doc/1535144
   2. https://dic.academic.ru/dic.nsf/dic_wingwords/2245/

Автор:
Святослав Холодов

Вопрос 15:
(pic: 20180286.jpg)
   Статья, рассказывающая об особняке "Island House", называется "ТАКАЯ
ОНА". "ТАКАЯ ОНА" - это термин, означающий практику ставить женщин или
представителей меньшинств на высокие посты в кризисный момент, когда
риск неудачи наиболее высок. Какие два слова мы заменили?

Ответ:
Стеклянная скала.

Зачет:
Стеклянная гора.

Источник:
   1. https://thearchitect.pro/ru/news/4194-Stekljannaja_skala
   2. http://www.vokrugsveta.ru/news/263843/

Автор:
Виктория Холодова

Вопрос 16:
Блиц.
   1. Явление в квантовой механике, суть которого заключается в том, что
квантовая система при определенных условиях может повести себя так, как
если бы частицы и их свойства были разделены в пространстве, носит
название "квантовый ИКС". Какое устойчивое выражение мы заменили ИКСОМ?
   2. Интеллектуальная игра "ИКС" напоминает "Брэйн-ринг", в котором две
команды отвечают на вопросы, и после каждого правильного ответа команда
высаживает из-за стола одного игрока. Побеждает та команда, которой
первой удалось "исчезнуть" полностью. Какое устойчивое выражение мы
заменили ИКСОМ?

Ответ:
   1. Чеширский кот.
   2. Чеширский кот.

Источник:
   1. https://ru.wikipedia.org/wiki/Квантовый_Чеширский_Кот
   2. http://chgk.wikia.com/wiki/Чеширский_кот

Автор:
Святослав Холодов

Вопрос 17:
По словам редактора этого журнала, в 2017 году на обложку юбилейного
номера хотели поместить изображение или известного изобретателя, или
классика русской литературы, или известной певицы. После долгих споров
решено было поместить фото классика. Назовите журнал, о котором идет
речь.

Ответ:
"MAXIM".

Комментарий:
Хайрем Стивенс МАксим, Максим Горький и певица МакSим.

Источник:
https://www.maximonline.ru/zhurnal/vnomere/_article/maxim-april-2017/

Автор:
Олег Холодов

Вопрос 18:
Макс Мюллер, живший в XIX веке и преподававший долгое время в Оксфорде,
выдвинул четыре простые причины ВАВИЛОНСКОГО СТОЛПОТВОРЕНИЯ. Все эти
причины были описаны в теориях с говорящими названиями, а именно: теория
"Ку-ку", теория "Пу-пу", теория "Динь-дон" и теория "Йо-хе-хо". Какие
слова мы заменили словами "ВАВИЛОНСКОЕ СТОЛПОТВОРЕНИЕ"?

Ответ:
Почему люди начали разговаривать.

Зачет:
По смыслу.

Источник:
http://www.h-rf.ru/history/456/56143/

Автор:
Святослав Холодов

Вопрос 19:
   <раздатка>
   - Давай разделим задачу на две части, - начал, наконец, я, - АЛЬФА и
ты.
   - АЛЬФА? - рука Софьи невольно дернулась к полотенцу на волосах.
   - Тьфу ты... - рассмеялся я невесело и пояснил: - Это я так батину
полюбовницу называю.
   </раздатка>
   Перед вами цитата из романа "Квинт Лициний - 3", автор - Oxygen
[Оксиген]. В другом романе другого автора свою вторую жену главный герой
звал АЛЬФОЙ, или просто седьмой. Какое женское имя мы заменили АЛЬФОЙ?

Ответ:
Бабетта.

Комментарий:
&Eta;, &eta; (эта) - седьмая буква греческого алфавита. Бабетта - эта
баба. Бабетта - прическа из длинных волос, при которой волосы
укладываются в валик сзади и частично на макушке.

Источник:
   1. http://samlib.ru/k/koroljuk_m_a/all_3.shtml
   2. http://samlib.ru/s/sewer_o/

Автор:
Святослав Холодов

Вопрос 20:
В первое четверостишие этого произведения входят двенадцать букв "о"; по
мнению литературоведа Александра Янушкевича, это создает образ толпы
зевак с широко открытыми ртами. По мнению конспиролога Александра
Сомсикова, это произведение посвящено заключению Тильзитского мира и
отражает сложившиеся на тот момент исторические реалии. Назовите это
произведение.

Ответ:
"Слон и Моська".

Комментарий:
Сомсиков анализирует басню Крылова "Слон и Моська" и возводит
происхождение слова "моська" к слову "Москва".

Источник:
   1. http://www.sciteclibrary.ru/texsts/rus/stat/st6042.pdf
   2. https://ru.wikipedia.org/wiki/Слон_и_Моська

Автор:
Ирина Холодова

Вопрос 21:
Святой Франциск и священномученик Власий, да и некоторые другие святые
ДЕЛАЛИ ЭТО. Это дало право писателю Олегу Северу предположить, что они,
как и Пифагор, верили в переселение душ. Ответьте точно: что мы заменили
словами "ДЕЛАЛИ ЭТО"?

Ответ:
Проповедовали животным.

Источник:
   1. http://rmvoz.ru/forums/index.php?topic=478.0
   2. http://megapoisk.com/kak-svjatye-s-dikimi-zhivotnymi-druzhili
   3. http://pandia.ru/text/77/465/22134-6.php
   4. http://samlib.ru/s/sewer_o/

Автор:
Олег Холодов

Вопрос 22:
Как сообщила "Herald Sun", с 7 марта 2017 года в Австралии вслед за
Германией по предложению общественной организации "Фокусная группа
будущего" наряду с мужским ИКСОМ стали использовать женский ИКС. Как
заметил в комментариях один из читателей, очевидно, следует ожидать, что
скоро появится и радужный ИКС. Какое сложное слово мы заменили ИКСОМ?

Ответ:
Светофор.

Комментарий:
Наряду с мужскими фигурками в Австралии в светофорах стали использовать
женские фигурки.

Источник:
https://lenta.ru/news/2017/03/08/zhenskie_svetofory/

Автор:
Святослав Холодов

Вопрос 23:
По мнению интернет-пользователя, заглавный герой произведения 1929 года
ДЕЛАЛ ЭТО. По мнению следствия, серийный убийца В. Кулик тоже ДЕЛАЛ ЭТО.
Какие три слова мы заменили словами "ДЕЛАЛ ЭТО"?

Ответ:
Тренировался на животных.

Комментарий:
Имеется в виду доктор Айболит, который почему-то лечил животных.

Источник:
   1. https://otvet.mail.ru/question/29805222
   2. https://ru.wikipedia.org/wiki/Айболит
   3. https://annetjohnson.livejournal.com/499412.html

Автор:
Святослав Холодов

Вопрос 24:
Перед тем как на Алексея Навального после оглашения приговора надели
наручники, он успел СДЕЛАТЬ ЭТО. В Калифорнии с 2013 года СДЕЛАТЬ ЭТО
можно на законных основаниях после задержания. Какие три слова мы
заменили словами "СДЕЛАТЬ ЭТО"?

Ответ:
Послать последний твит.

Зачет:
По смыслу.

Источник:
   1. http://obkon.ucoz.com/news/2013-07-18-285
   2. http://stolitsa.ee/47796

Автор:
Святослав Холодов

Тур:
8 тур. "Сплинтерелло" (Белгород)

Редактор:
Дмитрий Богданов (Белгород)

Инфо:
Редактор благодарит за помощь в работе над вопросами Максима Мерзлякова,
Серафима Шибанова, Дианну Давтян и команду "Хомяки" (Белгород), Романа
Ли и команду "Пикачу" (Белгород).

Вопрос 1:
"У нас почти как в театре..." - говорит героиня советского комедийного
фильма перед началом. Догадавшись, перед началом чего, назовите этот
фильм.

Ответ:
"Большая перемена".

Комментарий:
Перед началом каждого занятия дается два звонка, а в театре - три.

Источник:
Х/ф "Большая перемена" (1972), реж. Алексей Коренев, 1-я серия, 35-я
минута.

Автор:
Дмитрий Богданов (Белгород)

Вопрос 2:
Актриса Зоя Фёдорова познакомилась с капитаном Джэксоном Тэйтом в
феврале 1945 года. Почти через год у них родилась дочь. Как ее назвали?

Ответ:
Виктория.

Зачет:
Вика.

Комментарий:
По словам матери, ребенка зачали 9 мая, поэтому заранее решили назвать
Виктором или Викторией. Такой союз уже в конце 1946 года посчитали
преступным: Фёдорову обвинили в шпионаже и осудили на 25 лет заключения.

Источник:
https://ru.wikipedia.org/wiki/Фёдорова,_Виктория_Яковлевна

Автор:
Дмитрий Богданов (Белгород)

Вопрос 3:
Блиц.
   В одном из вариантов дубляжа герой мультфильма "Король Лев" весьма
оригинально рассказывает о ситуации в саванне.
   1. Он говорит: "Слоны бродят без дела". Какое слово в вопросе мы
заменили тремя?
   2. Он говорит: "Бабуины всё время подражают другим". Какое слово в
вопросе мы заменили двумя?
   3. Он говорит: "Леопарды себя опозорили". Какое слово в вопросе мы
заменили?

Ответ:
   1. Слоняются.
   2. Обезьянничают.
   3. Запятнали.

Комментарий:
Так игру слов передали для русскоязычного зрителя.

Источник:
   1. Мультфильм "Король Лев" (1994), 11-я минута.
   2. https://dic.academic.ru/dic.nsf/ogegova/223634/
   3. https://dic.academic.ru/dic.nsf/ogegova/129938/
   4. https://dic.academic.ru/dic.nsf/ogegova/195426/

Автор:
Дмитрий Богданов (Белгород)

Вопрос 4:
Бет Шапиро пишет, что для создания главной достопримечательности
сибирского парка не нужно находить превосходно сохранившуюся ЕЕ -
достаточно лишь нескольких клеток. Саркофагом для НЕЕ могло стать болото
или озеро со смолистым придонным слоем. Назовите ЕЕ двумя словами,
которые начинаются на одну и ту же букву.

Ответ:
Мумия мамонта.

Комментарий:
Вместо клонирования японские генетики предложили оплодотворить слониху,
чтобы получить 88-процентного мамонта. При наличии подходящих
сперматозоидов проект "Парка плейстоценовой эпохи" будет реализован в
этом веке. Смолы, масла и парафины могут миллионы лет сохранять мягкие
ткани попавшего в водоем животного.

Источник:
   1. https://indicator.ru/article/2017/08/27/kak-klonirovat-mamonta/
   2. http://www.evolbiol.ru/docs/docs/large_files/taphonomy.pdf
   3. https://ru.wikipedia.org/wiki/Плейстоценовый_парк

Автор:
Дмитрий Богданов (Белгород)

Вопрос 5:
Герои американского сериала сравнивают наклонный пол с НЕЙ в последние
20 минут. На одной из восьми ИХ располагался обеденный салон на 550
персон. Назовите ЕЕ, используя имя собственное.

Ответ:
Палуба "Титаника".

Комментарий:
Пол в квартире настолько кривой, что вещи скатываются к одной из стен
комнаты, а героям едва удается сохранить равновесие. На огромной палубе
знаменитого лайнера были зарезервированы столики для всех пассажиров
первого класса.

Источник:
   1. Телесериал "Как я встретил вашу маму", s03e12, 10-я минута.
   2. https://ru.wikipedia.org/wiki/Титаник

Автор:
Дмитрий Богданов (Белгород)

Вопрос 6:
Блиц.
   Вопросы могут показаться свояком за 50. Что ж, флаг вам в руки!
   1. Назовите автора пьесы, премьера которой состоялась в 1947 году в
Немецком театре Берлина.
   2. Назовите монаха из Аугсбурга, написавшего первый учебник санскрита
в Европе.
   3. Назовите художника, написавшего портрет раненного под Кёнигсбергом
солдата.

Ответ:
   1. [Евгений Львович] Шварц.
   2. [Генрих] Рот.
   3. [Герман Моисеевич] Гольд.

Комментарий:
Фамилии деятелей складываются в прозвище немецкого триколора -
"шварц-рот-гольд". На флаг Германии намекают преамбула и упомянутые
города.

Источник:
   1. https://dic.academic.ru/dic.nsf/dic_synonims/388202/
   2. https://de.wikipedia.org/wiki/Schwarz-Rot-Gold
   3. https://ru.wikipedia.org/wiki/Шварц,_Евгений_Львович
   4. https://ru.wikipedia.org/wiki/Рот,_Генрих
   5. https://ru.wikipedia.org/wiki/Гольд,_Герман_Моисеевич
   6. https://brenik.livejournal.com/6129794.html

Автор:
Дмитрий Богданов (Белгород)

Вопрос 7:
Надеемся, этот вопрос не станет кнопкой.
   В выборах талисмана чемпионата мира по хоккею 2016 года активно
участвовали пользователи Интернета, голосовавшие в соцсетях и на сайте
турнира. Ответьте абсолютно точно: кого выбрали пользователи?

Ответ:
Лайку.

Комментарий:
Чемпионат проходил в России, и в конкурсе лайков победила лайка по
кличке Байкал. Гостям первенства дружелюбный талисман очень понравился.

Источник:
https://www.sports.ru/tribuna/blogs/fotoblog/901246.html

Автор:
Дмитрий Богданов (Белгород)

Вопрос 8:
В 2010 году жители американского города вышли на демонстрацию. Они несли
плакаты с надписью "Eight is not enough" [эйт из нот инАф], т.е. "Восьми
недостаточно". Какое имя собственное они скандировали?

Ответ:
Плутон.

Зачет:
Pluto [чтецу: плУто].

Комментарий:
Так Иллинойс, где Плутон еще считают планетой, отметил 80 лет со дня
открытия, протестуя против решения Международного астрономического
союза.

Источник:
   1. Документальный фильм "Direct from Pluto: First Encounter" (2015),
27-я и 31-я минуты.
   2. https://ru.wikipedia.org/wiki/Плутон

Автор:
Дмитрий Богданов (Белгород)

Вопрос 9:
Майкл Браун дал прозвище "ИКС" карликовой планете, которую обнаружил
весной 2005 года. В мультфильме Питера Рэмси ИКС перемещается через
порталы в земле, что отсылает к более известному произведению. Назовите
ИКСА двумя словами.

Ответ:
Пасхальный кролик.

Зачет:
Пасхальный заяц.

Комментарий:
Планету открыли сразу после Пасхи. В итоге ей дали имя рапанУйского бога
МАкемАке. В мультфильме "Хранители снов" кролик использует норы в
качестве телепорта, как и его белый собрат из "Алисы в Стране чудес".
Имя режиссера - намек на Кролика Питера.

Источник:
   1. http://www.mikebrownsplanets.com/2008/07/whats-in-name-part-2.html
   2. Х/ф "Хранители снов" (2012), реж. Питер Рэмси, 81-я минута.

Автор:
Дмитрий Богданов (Белгород)

Вопрос 10:
Персонаж Уильяма Стауэлла упомянул о НЕМ, говоря о наследственной
болезни. Желая узнать о НЕМ, бельгиец Эркюль Пуаро произнес слово
"буфет". Назовите ЕГО тремя словами.

Ответ:
Скелет в шкафу.

Комментарий:
Английский драматург впервые употребил в печати выражение, обозначающее
семейную тайну. Знаменитый детектив отлично владеет английским, но
иногда всё же допускает ошибки.

Источник:
   1. http://www.phrases.org.uk/meanings/skeleton-in-the-closet.html
   2. Телесериал "Пуаро Агаты Кристи", s13e02, 16-я минута.

Автор:
Дмитрий Богданов (Белгород)

Вопрос 11:
По словам Джона Митчинсона, человеческий ОН утратил свои важные функции
с приручением собаки. В книге Екатерины Васиной ОН угрожает героине, что
та может повторить судьбу одного военного. Назовите ЕГО.

Ответ:
Нос.

Комментарий:
Автор научно-популярной книги утверждает, что роль обоняния уменьшилась
с появлением животных, более компетентных в этом деле. Из-за
невыносимого перегара нос девушки угрожает покинуть ее, намекая на
происшествие с майором Ковалёвым - героем сатирической повести Гоголя.

Источник:
   1. Дж. Митчинсон, Дж. Ллойд. Вторая книга всеобщих заблуждений.
http://flibusta.is/b/285785/read
   2. http://samlib.ru/w/wasina_e/dogonu.shtml
   3. https://ru.wikipedia.org/wiki/Нос_(повесть_Гоголя)

Автор:
Дмитрий Богданов (Белгород)

Вопрос 12:
Один из подвигов Суворова - взятие Измаила. На медали, врученной
Суворову в память об этом событии, можно увидеть ЕЕ. Другому герою
пришлось воспользоваться зубами, чтобы снять ЕЕ. Назовите ЕЕ двумя
словами.

Ответ:
Шкура льва.

Зачет:
Львиная шкура.

Комментарий:
Ни одно оружие не могло повредить шкуру немейского чудовища, но боги
надоумили Геракла использовать челюсти самого льва. На награде Александр
Васильевич изображен в облике античного полубога.

Источник:
   1. http://niros.org/lib/pictures/suvorov_medal.jpg
   2. http://www.pravmir.ru/suvorov-ne-lyubil-zerkal/
   3. Р. Грейвс. Мифы Древней Греции. http://flibusta.is/b/499494/read

Автор:
Дмитрий Богданов (Белгород)

Вопрос 13:
Слова "ИКС" и "ИКСИК" заменяют по два слова.
   На ИКСЕ оказался Робинзон Крузо после освобождения из плена
берберских пиратов. Пляжи на острове ПхукЕт в 2016 году были закрыты для
купания из-за ИКСИКОВ. Назовите ИКС и ИКСИКА.

Ответ:
Португальский корабль, португальский кораблик.

Комментарий:
Робинзона подобрали и доставили в Бразилию моряки-европейцы. Шупальцы
медузы-полипа редко убивают человека, но могут нанести ему тяжелые
травмы.

Источник:
   1. Д. Дефо. Жизнь и удивительные приключения Робинзона Крузо.
http://flibusta.is/b/273303/read
   2. http://www.nat-geo.ru/accidents/916389-plyazhi-pkhuketa-zakryli-iz-za-nashestviya-portugalskikh-korablikov/

Автор:
Дмитрий Богданов (Белгород)

Вопрос 14:
(pic: 20180287.jpg)
   Блиц.
   Мы раздали вам фрагменты креативных логотипов, созданных ПратИком
ЛАлой.
   1. Перед вами ВТОРАЯ. Воспроизведите все пять.
   2. Перед вами СЕДЬМАЯ и ДЕСЯТАЯ. Воспроизведите все десять.
   3. Перед вами ПЯТАЯ и ШЕСТАЯ. Воспроизведите все одиннадцать.

Ответ:
   1. Gauss.
   2. Heisenberg.
   3. Schr&ouml;dinger.

Зачет:
Похожие по написанию с соблюдением числа букв.

Комментарий:
(pic: 20180288.jpg) (pic: 20180289.jpg) (pic: 20180290.jpg)
   В логотипах обыграны фамилии известных ученых. Так, в честь
математика Карла Гаусса названа кривая распределения, в честь физика
Вернера Гейзенберга - принцип неопределенности, а Эрвин Шрёдингер часто
ассоциируется с мысленным экспериментом, главным героем которого
является кот.

Источник:
   1. http://www.lookatme.ru/mag/live/experience-news/200329-science-logos
   2. https://ru.wikipedia.org/wiki/Гаусс,_Карл_Фридрих
   3. https://ru.wikipedia.org/wiki/Гейзенберг,_Вернер
   4. https://ru.wikipedia.org/wiki/Шрёдингер,_Эрвин

Автор:
Дмитрий Богданов (Белгород)

Вопрос 15:
Встретившись однажды с Людовиком XV, Петр I произнес: "В моих руках -
вся Франция!". В тот момент Петр поднимал ЕГО. Назовите ЕГО.

Ответ:
Людовик [XV].

Зачет:
Король [Франции].

Комментарий:
Людовику было всего семь лет. Памятная скульптура об этом событии
установлена в Петергофе.

Источник:
   1. http://www.peterhof.ru/?m=246&p=267
   2. С.А. Мезин. Взгляд из Европы: французские авторы XVIII века о
Петре I.
http://annuaire-fr.narod.ru/bibliotheque/Mezin-book/Glava-1.html
   3. https://ru.wikipedia.org/wiki/Людовик_XV

Автор:
Дмитрий Богданов (Белгород)

Вопрос 16:
Отвечая на вопрос, как же ОН успевает, фанаты Джоан Роулинг полагают,
что ОН путешествует с помощью летучего пороха и использует патронусов.
Назовите ЕГО.

Ответ:
Санта-Клаус.

Зачет:
Санта; Святой Николай.

Комментарий:
Так объясняют появления Санты из дымохода, волшебных оленей в его
упряжке и то, как они успевают всех облететь за одну ночь. Философский
камень наделяет Санту богатством и бессмертием, заклинание расширения
пространства дает возможность возить с собой сразу все подарки, которые
изготовили сотни домовых эльфов.

Источник:
   1. https://www.reddit.com/r/harrypotter/comments/5h54l3/the_whole_santa_coming_down_the_chimney_thing_is/
   2. https://regnum.ru/news/2218060.html

Автор:
Дмитрий Богданов (Белгород)

Вопрос 17:
Дуплет.
   1. На плакате компании "Mercedes-Benz" [мЭрсэдэс-бэнц] рекламирующем
систему удержания автомобиля в полосе движения, курицу направляют
бегущие по бокам лисы, овцу - пастушьи собаки. А кто сопровождает
бегущего слона?
   2. Марк-Антуан ФардИн стал лауреатом за доказательство того, что с
возрастом ОНИ становятся всё более текучими. Назовите ИХ.

Ответ:
   1. Мыши.
   2. Кошки.

Зачет:
   1. Мышки; крысы.
   2. Коты; котята.

Комментарий:
   1. Рекламщики концерна обыграли стереотип, что слон боится мелких
грызунов.
   2. Проанализировав сотни фотографий, французский физик убедительно
обосновал, что коты ведут себя как жидкость, и получил Игнобелевскую
премию за 2017 год.

Источник:
   1. https://www.adsoftheworld.com/campaign/mercedes-bbdo-06-2015
   2. http://www.ig-nobel.ru/a2017fiz.php

Автор:
Дмитрий Богданов (Белгород)

Вопрос 18:
В заглавии статьи об ученом, изобразившем на плече перо археоптерикса, и
его глубоко копающих коллегах упоминается ОНА, как и в названии
произведения. Назовите ЕЕ двумя словами, которые начинаются на парные
согласные.

Ответ:
Татуировка дракона.

Комментарий:
Палеонтологи изобразили на себе объект научных интересов - кости и
скелеты динозавров. "Глубоко копающих" - намек на исследование ранних
геологических периодов.

Источник:
   1. http://www.maleus.ru/index.php/exkathedra/332-tattoo
   2. https://ru.wikipedia.org/wiki/Миллениум_(трилогия)

Автор:
Дмитрий Богданов (Белгород)

Вопрос 19:
По мнению фанатов, "IKEA" [икЕа] стоит назвать дверной стоппер в честь
НЕГО. Назовите ЕГО. ЕГО.

Ответ:
Ходор.

Зачет:
Уиллис; Уолдер.

Комментарий:
Герой "Игры престолов" получил прозвище от слов "Hold the door" [холд зе
до] - "Держи дверь". Фанаты нашли применение фигуркам персонажа.

Источник:
   1. https://meduza.io/shapito/2016/05/25/hodor-izmenil-otnoshenie-lyudey-k-liftam-i-dveryam
   2. http://ru.gameofthrones.wikia.com/wiki/Ходор

Автор:
Дмитрий Богданов (Белгород)

Вопрос 20:
Согласно индейской легенде, во время войны между птицами и зверями ОНА
поочередно воевала то за одних, то за других. Назовите ЕЕ.

Ответ:
Летучая мышь.

Комментарий:
Каждый раз плутовка присоединялась к побеждавшим, но в итоге осталась
одна против всех и была раздавлена.

Источник:
http://www.indiansworld.org/legmalet.html

Автор:
Дмитрий Богданов (Белгород)

Вопрос 21:
Тысячи евреев были эвакуированы из Йемена в ходе операции под названием
"ИКС". "Меховой ИКС" - прозвище малайского шерстокрыла. Назовите ИКС.

Ответ:
Ковер-самолет.

Комментарий:
Эвакуацию проводили по воздуху. Шерстокрыл, как следует из названия,
использует планирующий полет для перемещения с одного дерева на другое,
не соприкасаясь с землей.

Источник:
   1. https://ru.wikipedia.org/wiki/Операция_%C2%ABОрлиные_крылья%C2%BB
   2. https://nplus1.ru/material/2017/01/11/colugo

Автор:
Дмитрий Богданов (Белгород)

Вопрос 22:
В квартире террориста, который основал Лигу низкорослых людей,
следователи нашли ЕГО. При создании одного из НИХ ученик Давида
вдохновлялся работой Фидия. Назовите ЕГО.

Ответ:
Портрет Наполеона [Бонапарта].

Комментарий:
По мнению Петра Волынского, высокие граждане угнетают и унижают низких.
Жертвами шизофреника ростом в 164 сантиметра стали 10 человек. Наполеон
до сих пор часто ассоциируется с маниакальным карликом. На картине Энгра
Бонапарт сидит на императорском троне в позе, напоминающей Зевса
Олимпийского.

Источник:
   1. https://ru.wikipedia.org/wiki/Взрыв_автобуса_в_Краснодаре
   2. https://en.wikipedia.org/wiki/Napoleon_I_on_His_Imperial_Throne
   3. https://ru.wikipedia.org/wiki/Энгр,_Жан_Огюст_Доминик

Автор:
Дмитрий Богданов (Белгород)

Вопрос 23:
Поприветствовав Джорджа Буша-старшего, последний телефонный разговор
президентов СССР и США Михаил Горбачёв начал... С чего?

Ответ:
С рождественского поздравления.

Зачет:
С поздравления семьи президента с Рождеством.

Комментарий:
Разговор состоялся 25 декабря, а на следующий день Советский Союз уже
прекратил существование. О своей отставке Горбачёв сообщил президенту
Соединенных Штатов за два часа до выступления по всесоюзному
телевидению.

Источник:
   1. http://back-in-ussr.com/2015/12/stenogrammy-telefonnyh-razgovorov-elcina-i-gorbacheva-s-dbushem.html
   2. https://ru.wikipedia.org/wiki/Союз_Советских_Социалистических_Республик

Автор:
Дмитрий Богданов (Белгород)

Вопрос 24:
Рабочим названием пятого фильма известной серии было слово "baby"
[бЭйби]. Поддержка "Baby" [бЭйби] была прекращена весной 2017 года.
Какое слово в этом вопросе мы заменили словом "baby" [бЭйби]?

Ответ:
Vista.

Зачет:
Виста.

Комментарий:
Одно из слов культовой фразы киборга Т-800 - "Hasta la vista, baby"
[Аста ла вИста, бЭйби] мы заменили другим. А фильм назвали "Терминатор.
ГЕнезис".

Источник:
   1. https://www.imdb.com/title/tt1340138/trivia?item=tr2468514
   2. https://ru.wikipedia.org/wiki/Терминатор:_Генезис
   3. https://ru.wikipedia.org/wiki/Windows_Vista
   4. https://ru.wikipedia.org/wiki/Hasta_la_vista,_baby

Автор:
Дмитрий Богданов (Белгород)

Тур:
9 тур. "Я больше не увижу Валинор" (сборная)

Вопрос 1:
Согласно песне, причин, почему ОН не может играть в регби, множество: у
Него недостаточное количество друзей, флэшбэки от вида штанг,
запрещенный головной убор, мяч пролетает сквозь Его руки, а Его Отец
может повлиять на исход матча. Назовите ЕГО.

Ответ:
Иисус. Незачет: Игорь Николаев.

Комментарий:
У Него на это как минимум пять причин: количество апостолов, боязнь
больших деревянных столбов, терновый венец, дырявые руки и отец-Бог.

Источник:
http://www.beerandrugby.com/jesuscantplayrugby.html

Автор:
Алексей Штых

Вопрос 2:
Блиц.
   1. В возрасте 12 лет ОН занял второе место на конкурсе песни
"Стратфордский идол". Напишите ЕГО фамилию.
   2. Ногти на третьих пальцах ЕЕ лап покрыты кожаной оболочкой.
Назовите ЕЕ.
   3. Четвертой ЕГО работой в кино стал третий фильм тетралогии.
Напишите ЕГО фамилию.

Ответ:
   1. Бибер.
   2. [Большая] виверра.
   3. Гигер.

Комментарий:
Слова в ответах построены по схожему принципу с использованием второй,
третьей и четвертой букв алфавита.

Источник:
   1. https://ru.wikipedia.org/wiki/Бибер,_Джастин
   2. https://ru.wikipedia.org/wiki/Азиатская_цивета
   3. https://en.wikipedia.org/wiki/H._R._Giger

Автор:
Дмитрий Карякин

Вопрос 3:
Героиня британского фильма "Лок" находится в роддоме и на протяжении
всего фильма ждет отца своего ребенка. Каким коротким именем она
называет этого отца?

Ответ:
Годо.

Комментарий:
Героиня сравнивает отца ребенка с персонажем пьесы Беккета, которого
другие персонажи ждут на протяжении всей пьесы.

Источник:
Х/ф "Лок" (2013), реж. Стивен Найт.

Автор:
Владимир Сушков

Вопрос 4:
Герой новеллы Джеффа Вандермеера получает письмо с просьбой быть гостем
на необычном событии. Сразу после этого он вспоминает птицу. Назовите
эту птицу.

Ответ:
Сирин.

Комментарий:
В мире Амбры, в котором происходит действие, существует двойник
Набокова, который тоже написал "Приглашение на казнь".

Источник:
Дж. Вандермеер. Трансформация Мартина Лейка.
http://flibusta.is/b/274896/read#t16

Автор:
Дмитрий Карякин

Вопрос 5:
В одной компьютерной игре можно увидеть пятнистых красно-белых существ,
которые являются гибридом представителей двух разных царств. В
английском названии этих существ есть четыре одинаковые гласные.
Напишите это название.

Ответ:
Mooshroom.

Комментарий:
Это гибриды муууухомора и коровы, а не свиньи, как вы могли подумать,
исходя из их окраски.

Источник:
http://minecraft.gamepedia.com/Mooshroom

Автор:
Дмитрий Карякин

Вопрос 6:
В докладе Bank of America говорится, что неожиданное и резкое падение
биржевых котировок может привести к политическому кризису в России.
Доклад называется "ОНО в 2018 году". Назовите ЕГО.

Ответ:
Лебединое озеро.

Комментарий:
Сложнопредсказуемые катастрофические события (особенно экономической
природы) часто называют "Черными лебедями". Возможно, благодаря одному
из них знаменитый балет много-много раз покажут по телевизору.

Источник:
https://www.rbc.ru/economics/14/12/2017/5a3242f79a7947348ca26990

Автор:
Михаил Новосёлов

Вопрос 7:
Действующие лица пьесы Леся Подервянского - Толпа и ОН. ОН
последовательно лишается печени и почек. Назовите ЕГО.

Ответ:
Данко.

Комментарий:
В пьесе Подервянского Данко вначале вырывает себе сердце, а потом еще и
печень и почки.

Источник:
http://www.gamlet.org/pesi/danko-feeria

Автор:
Дмитрий Карякин

Вопрос 8:
   <раздатка>
   - Не забывай, что мы - многорасовое сообщество, и все религии суть
одна.
   - Попробуй сказать это...
   </раздатка>
   Закончите розданную цитату из романа Уильяма Голдинга двумя словами.

Ответ:
"... в мечети".

Зачет:
"... в мичети".

Комментарий:
Цитата перекликается со ставшим популярным в Интернете высказыванием
"попробовали бы они это сделать в мечети". Изначально так высказалась
Елена Ваенга по поводу скандального выступления группы "Pussy Riot" в
храме Христа Спасителя.

Источник:
У. Голдинг. Зримая тьма. http://flibusta.is/b/510962/read

Автор:
Владимир Сушков

Вопрос 9:
Умберто Эко считает, что произведение XX века начинается в определенном
месте, потому что автор произведения стремился к слиянию языков. Что это
за произведение?

Ответ:
"Улисс".

Комментарий:
Действие "Улисса" начинается в башне Мартелло, которая является аналогом
Вавилонской башни. Автор вопроса считает, что это можно спрашивать, так
как уж первую-то главу многие из вас наверняка осилили.

Источник:
У. Эко. О литературе. Эссе. http://flibusta.is/b/440047/read

Автор:
Дмитрий Карякин

Вопрос 10:
Название южноафриканского вина переводится как "Козлы бродяжничают". При
его производстве используется виноград сортов сирА, гренАш, мурвЕдр,
сенсО, кариньЯн. Какой европейский топоним упоминается в истории
появления названия этого вина?

Ответ:
Рона.

Зачет:
Кот дю Рон.

Комментарий:
Для этого вина используются традиционные ронские сорта, а название
"Goats do roam" похоже по звучанию на C&ocirc;tes du Rh&ocirc;ne [кот дю
рон] - берег Роны.

Источник:
https://simplewine.ru/catalog/product/fairview_goats_do_roam_red_2016_075/

Автор:
Дмитрий Карякин

Вопрос 11:
(pic: 20180291.jpg)
   Второе слово в английском названии ролика, кадр из которого мы вам
раздали, появилось благодаря ЮбЕру КундЕ. Напишите это название целиком.

Ответ:
Jurassic parkour.

Комментарий:
Слово "parkour" появилось благодаря французскому режиссеру из слова
"parcours" заменой "c" на "k" и отбрасыванием непроизносимой "s" на
конце. "Jurassic Park" - фильм Стивена Спилберга.

Источник:
   1. https://www.youtube.com/watch?v=v71to9IHALc
   2. https://en.wikipedia.org/wiki/Parkour

Автор:
Алексей Штых

Вопрос 12:
Незадолго до смерти Томас Вулф писал, что подозревает самое худшее, ведь
во время недавнего путешествия встречался с НИМ. "ОН" - название фильма
1951 года в стиле "нуар". Назовите ЕГО двумя словами.

Ответ:
Черный человек.

Источник:
   1. http://www.workinprogressinprogress.com/2013/04/thomas-wolfes-beautiful-deathbed-letter.html
   2. https://en.wikipedia.org/wiki/The_Dark_Man_(film)

Автор:
Владимир Сушков

Вопрос 13:
   <раздатка>
   В девятьсот девяносто пятом
   </раздатка>
   Назовите вышедший в 1995 году фильм, идентификатор которого в базе
IMDB равен 0113481.

Ответ:
"Джонни-мнемоник".

Комментарий:
Мы раздали вам мнемоническую фразу для запоминания года выхода фильма
"Джонни-мнемоник". Количество букв в словах этой фразы равно числу в
соответствующем разряде года выхода фильма. Автору вопроса полезность
этой мнемоники кажется сравнимой с полезностью всех остальных мнемоник.
Идентификатор же нужен просто для отсечки.

Источник:
https://www.imdb.com/title/tt0113481/

Автор:
Дмитрий Карякин

Вопрос 14:
Героиня пьесы Ибсена увлекается стрельбой из револьверов. В конце пьесы
она покончила жизнь самоубийством при помощи одного из этих револьверов.
В статье Википедии об этой пьесе упоминается ОНО. Назовите ЕГО двумя
словами.

Ответ:
Чеховское ружье.

Зачет:
Chekhov's gun.

Комментарий:
Револьвер, появившийся в начале пьесы, выстрелил в конце.

Источник:
   1. Г. Ибсен. Гедда Габлер. http://flibusta.is/b/23309/read
   2. https://en.wikipedia.org/wiki/Hedda_Gabler

Автор:
Владимир Сушков

Вопрос 15:
В вопросе есть замена.
   Героиня одного мультфильма говорит, что перед переездом в Австралию
старается побольше узнать о новой стране. Затем она упоминает коалу и
росомаху. Что мы заменили в этом вопросе?

Ответ:
Хью Джекман.

Комментарий:
Героиня упоминает австралийского актера Хью Джекмана, который
прославился благодаря роли Росомахи в серии фильмов "Люди Икс".

Источник:
   1. Мультфильм "Гадкий я - 2".
   2. https://ru.wikipedia.org/wiki/Джекман,_Хью

Автор:
Владимир Сушков

Вопрос 16:
В 2011 году Максим Дмитриевич, отвечая на критику в адрес Никиты
Михалкова, говорил: "В те времена чудовищный механизм, который давил и
уничтожал художника, был понятен. Его целью было сделать художника
управляемым, заставить творить в угоду власти". Далее Максим Дмитриевич
сказал, что фильмы Михалкова тоже называют ИМ. Назовите ЕГО одним
словом.

Ответ:
Сумбур.

Комментарий:
Максим Шостакович - сын Дмитрия Шостаковича.

Источник:
https://ru.wikipedia.org/wiki/Шостакович,_Максим_Дмитриевич

Автор:
Владимир Сушков

Вопрос 17:
Бойцы "Исламского государства" жалуются, что очереди на то, чтобы стать
смертником, приходится ждать слишком долго. Какое слово римского
происхождения присутствует в заголовке статьи газеты "Independent",
рассказывающей об этом?

Ответ:
Непотизм.

Комментарий:
Высокопоставленные командиры продвигают своих родственников и друзей в
очереди на то, чтобы стать смертником, и, соответственно, на попадание в
рай, что вызывает недовольство рядовых боевиков. Непотизмом любили
заниматься римские папы; "римского" - подсказка.

Источник:
https://www.independent.co.uk/news/world/middle-east/isis-fighters-complaining-of-saudi-nepotism-on-groups-waiting-list-for-suicide-bombers-10275690.html

Автор:
Михаил Новосёлов

Вопрос 18:
В конце XVIII века было распространено мнение, что Великая французская
революция была вызвана, в частности, пьесой Бомарше "Женитьба Фигаро".
Рассказывая об этом, Андрей Зорин упоминает, как на одном из
представлений зрители распевали последнюю строчку "Tous finit par des
chansons" [ту финИ пар де шансОн], т.е. "Всё заканчивается песнями",
немного изменив в ней одно слово. Чем же в результате всё стало
заканчиваться?

Ответ:
Пушками.

Комментарий:
Пьеса, в которой слуга успешно защищает свою невесту от домогательств
сеньора, по ходу дела притворно ухаживая за его женой, могла натолкнуть
на мысли о равенстве, а французское слово "canon" [канОн] родственно
английскому "cannon" [кЭннон] (фраза - "Tous finit par des canones" [ту
финИ пар де канОн]).

Источник:
А.Л. Зорин. Появление героя. Из истории русской эмоциональной культуры
конца XVIII - начала XIX века. http://flibusta.is/b/453915/read

Автор:
Дмитрий Карякин

Вопрос 19:
Существует сомнительная версия, что ЕГО название пришло в русский язык
из немецкого, так как одна порция обычно рассчитана для двоих. Среди
мусульман существует байка, что люди, употребляющие ЕГО при жизни, в
мусульманском раю будут регулярно получать порцию расплавленного свинца
под язык. Назовите ЕГО.

Ответ:
Насвай.

Зачет:
Нацвай; насыбай; нас; нац.

Комментарий:
По одной из версий, название вещества пошло из-за смешивания русского
предлога "на" и немецкого слова "zwei" [цвай] - "два".

Источник:
   1. Д.А. Савочкин. Марк Шейдер. http://flibusta.is/b/181351/read
   2. https://lurkmore.to/Насвай

Автор:
Александр Григорьев

Вопрос 20:
В этом вопросе словом "ОНА" заменены два слова.
   Можно сказать, что меньшая ОНА - "утка". Что ОНА делала в известном
источнике вместе с некой женщиной?

Ответ:
Рыдала.

Зачет:
Плакала.

Комментарий:
ОНА - это половина маршрутки, которая, согласно мему, рыдает вместе с
матерью, когда ее четырехлетняя дочь о чем-то просит Бога.

Источник:
   1. ЛОАВ.
   2. https://tjournal.ru/65587-mem-polovina-marshrutki

Автор:
Александр Марков

Вопрос 21:
В одном англоязычном комиксе распределяющая шляпа отправляет чернокожего
абитуриента Хогвартса в тюрьму. Затем выясняется, что ранее шляпа
служила ЕМУ. Назовите ЕГО.

Ответ:
Grand Wizard.

Зачет:
Великий Мудрец; Великий Маг; Великий Волшебник; Великий Чародей
[Ку-Клукс-Клана].

Источник:
https://www.reddit.com/r/funny/comments/4qf3hm/old_hat/

Автор:
Михаил Новосёлов

Вопрос 22:
В песне рэп-исполнителя Chemodan [чемодАн], которая посвящена метро,
упоминается ОНА. Идея создать другую "ЕЕ" пришла к режиссеру во время
пребывания в Мохаве. Назовите ЕЕ двумя словами.

Ответ:
"Дрожь земли".

Комментарий:
Серия фильмов, рассказывающая о монстрах-червях, которые проживают под
землей, преимущественно в песчаной местности. Мохаве - американская
пустыня.

Источник:
   1. http://rap-text.ru/the_chemodan/9644-the-chemodan-m-tekst-pesni.html
   2. https://ru.wikipedia.org/wiki/Дрожь_земли

Автор:
Владимир Сушков

Вопрос 23:
Члены группировки под названием "Преследуемые Пабло Эскобаром" называли
себя ОНИ. В статье о НЕМ упоминается египетский бог-амфибия Кек.
Назовите ЕГО.

Ответ:
Пепе.

Комментарий:
PErseguidos por Pablo Escobar - Los Pepes. Praise KEK.

Источник:
   1. https://ru.wikipedia.org/wiki/Лос_Пепес
   2. https://en.wikipedia.org/wiki/Pepe_the_Frog

Автор:
Нариман Намазов

Вопрос 24:
Американцы иногда упоминают полное имя этого человека, и в качестве
среднего имени в нем - инициал H. [эйч]. Согласно шутке, H. [эйч]
расшифровывается как гаплоид. Назовите этого человека.

Ответ:
Иисус.

Зачет:
Jesus; Jesus H. Christ.

Комментарий:
Это человек, которого частенько упоминают всуе и, для усиления эффекта,
порой по "полному имени". У Иисуса якобы не было биологического отца, а
только мать. Более вероятная теория говорит, что этот инициал происходит
из попытки расшифровать монограмму &Iota;&Eta;&Sigma; [йота-эта-сигма].

Источник:
https://en.wikipedia.org/wiki/Jesus_H._Christ

Автор:
Алексей Штых

Тур:
10 тур. "Кенгуру на подходе" (сборная)

Редактор:
Артем Колесов (Буй - Чистые Боры - Волгореченск - Пермь - Москва) и
Серафим Шибанов (Москва)

Инфо:
При тестировании ни одного знатока не пострадало.

Вопрос 1:
Внимание, черный ящик!
   Накануне одной битвы лидер мусульман послал португальскому полководцу
сутану и четки, а в ответ получил то, что сейчас находится в черном
ящике. Что же в черном ящике?

Ответ:
Зеркало.

Зачет:
Зеркальце.

Комментарий:
Мусульманин намекал своему коллеге, что тому следовало быть монахом.
Португалец же намекнул противнику, что тот - женщина. Такой вот обмен
любезностями.

Источник:
Н.Н. Непомнящий, Н.В. Кривцов. Неведомая Африка.
http://flibusta.is/b/92709/read

Автор:
Серафим Шибанов (Москва)

Вопрос 2:
Внимание, черный ящик!
   В черном ящике находится то, в чем Леонид Виноградов усмотрел брань и
купюры небольшого номинала. Что в черном ящике?

Ответ:
Матрешки.

Зачет:
Матрешка.

Комментарий:
Матрешки. Мат. Трешки.

Источник:
Л.А. Виноградов. Жалостные стихи. - М., 2004.

Автор:
Серафим Шибанов (Москва)

Вопрос 3:
Внимание, черный ящик!
   В черном ящике находится то, роль чего в аэропорту Эрика Нильсена на
Юконе выполняет старенький "Дуглас". Аналоги этого предмета в обиходе
называют "колбасой". Что в черном ящике?

Ответ:
Флюгер.

Комментарий:
Ветроуказатель на аэродромах часто называют колбасой. На территории
аэропорта Эрика Нильсена в Уайтхорсе, Юкон, в качестве флюгера
установили старый самолет "Дуглас". Этот самолет претендует на звание
крупнейшего в мире флюгера.

Источник:
   1. https://en.wikipedia.org/wiki/Weather_vane
   2. https://ru.wikipedia.org/wiki/Ветроуказатель

Автор:
Артем Колесов (Буй - Чистые Боры - Волгореченск - Пермь - Москва)

Вопрос 4:
Проведите слева направо прямую линию произвольной длины. Затем от
правого конца этой линии проведите перпендикулярную ей линию той же
длины вниз. От нижнего конца проведите влево линию той же длины
перпендикулярно второй. Наконец, от свободного конца третьей линии
проведите перпендикулярно вверх четвертую линию той же длины. У вас
должна была получиться часть китайского иероглифа, обозначающего то, что
в 1771 году Джеймс Кук подарил супруге. Что же в черном ящике?

Ответ:
Бумеранг.

Комментарий:
Очевидно, заканчивая последовательность действий, которые должны были
привести к изображению квадрата, вы заметили, что вернулись к исходной
точке. :-) "1771" и "Кук" - палиндромы, а палиндром тоже имеет свойство
возвратности.
   Этот блок - оммаж Михаилу Савченкову. Чемпион мира и известный
редактор как-то заявил, что нужно забыть слово "оммаж" и нельзя писать
вопросы с ответами "зеркало", "матрешка", "флюгер" и "бумеранг". Он
также заявил, что скоро в этот список попадет и ответ "кенгуру". В связи
с этим команда "Кенгуру на подходе" передает Михаилу пламенный привет!

Источник:
   1. https://zh.wikipedia.org/wiki/%E5%9B%9E%E5%8A%9B%E9%95%96
   2. https://diletant.media/cold_arms/29579829/

Автор:
Артем Колесов (Буй - Чистые Боры - Волгореченск - Пермь - Москва)

Вопрос 5:
(pic: 20180292.jpg)
   На самом деле, по замыслу автора вопроса, на месте слов "William The
Conqueror" должны стоять два английских слова. Напишите их.

Ответ:
Fat bastard.

Комментарий:
Как вы видите, подпись под фото Адольфа Гитлера мы напечатали шрифтом
Arial, под фото Михаила Саакашвили - шрифтом Georgia, а под портретом
Вильгельма Завоевателя, рожденного вне брака, - шрифт Fat Bastard.

Источник:
https://www.fonts.com/font/virus-fonts/bastard

Автор:
Серафим Шибанов (Москва)

Вопрос 6:
(pic: 20180293.jpg)
   Напишите фамилию футболиста, которого вы видите в правом нижнем углу.

Ответ:
Немец.

Комментарий:
Вы видите соответствия "Руда - Неруда", "Глинка - Неглинка" и "Мец -
Немец".

Источник:
Общие знания.

Автор:
Серафим Шибанов (Москва)

Вопрос 7:
(pic: 20180294.jpg)
   Перед вами фрагмент обложки произведения, появившегося на свет в 2010
году. Назовите любого из двух человек, указанных в качестве его авторов.

Ответ:
[Бен Х.] Уинтерс.

Зачет:
[Лев Николаевич] Толстой.

Комментарий:
Это обложка мэшап-романа "Андроид Каренина".

Источник:
https://en.wikipedia.org/wiki/Android_Karenina

Автор:
Серафим Шибанов (Москва)

Вопрос 8:
(pic: 20180295.jpg)
   Послушав комментарии заказчика пакета, Серафим Шибанов вспомнил
строчку из песни. Какие два числа упоминаются в этой строке?

Ответ:
228, 282.

Зачет:
В любом порядке.

Комментарий:
"Кому 228, кому 282", - спела группа "25/17". Увидев эти вопросы в
пакете, заказчик обратил внимание, что в первом случае вопрос про
наркотики, а во втором - про расовое превосходство.
   Этот блок был посвящен вопросам с раздаточным материалом.

Источник:
Подготовка пакета "Енисейской знати - 2017" и сателлитных турниров
вместе с Артемом Колесовым.

Автор:
Серафим Шибанов (Москва)

Вопрос 9:
На основе ЕЕ сценария в 1973 году был снят фильм "Ищу человека".
Напишите ЕЕ фамилию.

Ответ:
Барто.

Комментарий:
Агния Барто долгое время вела телепередачу с похожим названием - "Найти
человека". Позже эта телепередача была взята за основу для "Жди меня".

Источник:
https://ru.wikipedia.org/wiki/Барто,_Агния_Львовна

Автор:
Серафим Шибанов (Москва)

Вопрос 10:
Википедия пишет, что ЕГО часто возвращали на фермы для отпоя молодняка.
В 100 граммах НЕГО - в среднем 113 мг кальция и 156 мг калия. Назовите
ЕГО одним словом.

Ответ:
Обрат.

Комментарий:
Возвращали обратно, отсюда и название.

Источник:
https://ru.wikipedia.org/wiki/Обезжиренное_молоко

Автор:
Серафим Шибанов (Москва)

Вопрос 11:
В финском варианте фамилия была образована от слова "reppu" [рЕппу],
т.е. рюкзак. В русском же - от НЕЕ. ОНА входит в название известной
группы. Назовите ЕЕ одним словом.

Ответ:
Торба.

Комментарий:
Речь идет о локализации фамилии Бэггинс. В России ее переводили как
Сумкинс или Торбинс. Группа, о которой идет речь, - "Торба-на-Круче".

Источник:
   1. https://ru.wikipedia.org/wiki/Бильбо_Бэггинс
   2. https://ru.wikipedia.org/wiki/Торба-на-Круче_(группа)

Автор:
Серафим Шибанов (Москва)

Вопрос 12:
Спустя несколько лет, в 2012 году, министр иностранных дел Ержан
Казыханов признался: "Я благодарен ЕМУ за то, что он помогает
привлечению туристов". Назовите ЕГО.

Ответ:
"Борат".

Комментарий:
Поначалу фильм Саши "Барона" Коэна в Казахстане приняли не так тепло. Но
потом поняли собственную выгоду. Все ответы на вопросы этого блока -
анаграммы друг друга.

Источник:
https://ru.wikipedia.org/wiki/Борат

Автор:
Серафим Шибанов (Москва)

Вопрос 13:
Дуплет.
   1. Своим названием ОНА обязана тому, что считалась красивой, а под
воздействием солнечных лучей темнела. Назовите ЕЕ.
   2. Своим названием ОНА обязана тому, что ЕЕ получали возгонкой.
Назовите ЕЕ.

Ответ:
   1. КАломель.
   2. СулемА.

Комментарий:
Этот дуплет посвящен хлориду ртути (I) и хлориду ртути (II).
   1. Слово "каломель" происходит от греческих корней
&kappa;&alpha;&lambda;&#972;&sigmaf; [кАлос] - красивый и
&mu;&#941;&lambda;&alpha;&sigmaf; [мЕлас] - черный.
   2. Слово "сулема" происходит от позднелатинского "sublimatum" -
вознесенное, т.е. добытое возгонкой.

Источник:
   1. И.А. Леенсон. Язык химии. Этимология химических названий.
https://books.google.ru/books?id=BmJDDQAAQBAJ&pg=PT153#v=onepage&q&f=false
   2. https://ru.wikipedia.org/wiki/Хлорид_ртути(I)
   3. https://ru.wikipedia.org/wiki/Хлорид_ртути(II)

Автор:
Артем Колесов (Буй - Чистые Боры - Волгореченск - Пермь - Москва)

Вопрос 14:
Дуплет. Ответы на вопросы дуплета должны быть записаны друг под другом.
   1. Аппер Бэй в дельте Гудзона можно назвать ЕЮ. На одном рисунке ЕЕ
образует английская буква M [эм]. Назовите ЕЕ двумя словами.
   2. Боттом Бэй на Барбадосе можно назвать ЕЮ. На одном рисунке ЕЕ
образует английская буква U [ю]. Назовите ЕЕ двумя словами.

Ответ:
   1. Верхняя губа.
   2. Нижняя губа.

Комментарий:
Губа - это еще и принятый в русском языке синоним слова "бухта". На
рисунке автора вопроса буквы слова "MOUTH" образуют рот: M и U - верхнюю
и нижнюю губы, O - внутренние очертания открытого рта, T и H - зубы.
Рисунок - по запросу.

Источник:
   1. https://en.wikipedia.org/wiki/Upper_New_York_Bay
   2. https://en.wikipedia.org/wiki/Bottom_Bay
   3. Рисунок автора вопроса.

Автор:
Артем Колесов (Буй - Чистые Боры - Волгореченск - Пермь - Москва)

Вопрос 15:
Дуплет.
   1. Отца Ричарда Дэвида Фэлко звали Тони, а приемного отца - Натаном.
Под каким прозвищем из двух слов Ричард Дэвид Фэлко стал известен?
   2. ЕГО внук Омар Фароян в свое время играл за молодежный состав
киевского "Динамо" и помогал товарищам по команде возвращать украденные
иномарки. Назовите ЕГО.

Ответ:
   1. Сын Сэма.
   2. Дед Хасан.

Зачет:
   2. Аслан Усоян.

Комментарий:
   1. Ричард Дэвид Фэлко, он же Ричард Берковитц, был известным серийным
убийцей, но сыном Сэма с точки зрения генеалогии не был.
   2. Дед Хасан, наоборот, действительно был дедом.

Источник:
   1. https://en.wikipedia.org/wiki/David_Berkowitz
   2. https://www.mk.ru/social/2013/05/30/862461-vnuk-deda-hasana-obosnovalsya-v-kievskom-dinamo.html

Автор:
Серафим Шибанов (Москва)

Вопрос 16:
Дуплет. Проверим, хорошо ли вы помните творчество Ильфа и Петрова!
   1. В романе "Двенадцать стульев" первое слово в именительном падеже
встречается два раза - в главах 14 и 33, а второе - один раз, в самой
первой главе. Эти два слова образуют название другого романа. Какого
именно?
   2. В романе "Золотой теленок" первое слово в именительном падеже
встречается девять раз - в главах 5, 9, 12, 21, 27 и 29, а второе в
именительном - ни разу, но один раз в 20-й главе в предложном. Эти два
слова образуют название другого романа. Какого именно?

Ответ:
   1. "Золотой теленок".
   2. "Двенадцать стульев".

Комментарий:
Вряд ли бы мы стали проверять вашу память. :-)
   Этот блок был посвящен дуплетам.

Источник:
   1. И. Ильф, Е. Петров. Двенадцать стульев.
http://az.lib.ru/i/ilfpetrov/text_0100.shtml
   2. И. Ильф, Е. Петров. Золотой теленок.
http://az.lib.ru/i/ilfpetrov/text_0130.shtml

Автор:
Серафим Шибанов (Москва)

Вопрос 17:
Вопрос задает Евгений Маргулис.
   Блиц.
   1. В хайку Ёса Бусона сквозь ЕЕ ветви мельтешит и мерцает храм.
Назовите ЕЕ.
   2. В хайку Виталия Фатеева ОНА дрожит в руках перед хризантемой.
Назовите ЕЕ.
   3. В хайку Мацуо Басё аромат ЕГО источает океан. Назовите ЕГО.

Ответ:
   1. Сакура.
   2. Катана.
   3. Сакэ.

Комментарий:
"Сакура, катана, сакэ" - известная песня Евгения Маргулиса.

Источник:
   1. http://japanpoetry.ru/poetry/20197
   2. https://www.stihi.ru/2014/05/18/4068/
   3. http://wikilivres.ca/wiki/Аромат_сакэ
   4. https://music.yandex.ru/album/950925/track/9059292

Автор:
Серафим Шибанов (Москва)

Вопрос 18:
Среди работ Джузеппе Арчимбольдо есть "Флора" и "Времена года".
Считается, что Арчимбольдо стоял у истоков ЕЕ. ЕЮ можно назвать
творчество группы, которая появилась в 1969 году и всё еще с нами. Какое
слово мы заменили ЕЮ?

Ответ:
Цветомузыка.

Комментарий:
Флора - это в том числе цветы, а "Времена года" есть не только у
Вивальди. Творчество группы "Цветы" можно назвать цветомузыкой. "С нами"
- намек на С. Намина. :-) К слову, Стас Намин образовал свой псевдоним
от имени своей матери, которую зовут Нами. И она тоже всё еще с нами.

Источник:
   1. https://ru.wikipedia.org/wiki/Светомузыка
   2. https://ru.wikipedia.org/wiki/Цветы_(группа)

Автор:
Артем Колесов (Буй - Чистые Боры - Волгореченск - Пермь - Москва)

Вопрос 19:
Внимание, в вопросе одно слово заменено двумя другими.
   "Проповедь в селе", "Сватовство" и "Гитарист-бобыль" принадлежат
кисти ЧЕРЕЗ ЯЙЦА. На территории ЧЕРЕЗ ЯЙЦА обитают кряквы, гоголи,
перепелятники, речные крачки, белоспинные дятлы, иволги, скворцы, грачи,
сороки, галки. А в какой группе пела Елена ЧЕРЕЗ ЯЙЦА?

Ответ:
"Лицей".

Зачет:
"Амега".

Комментарий:
Как утверждает Google Translate, ЧЕРЕЗ ЯЙЦА на латыни будет звучать как
"Per Ova". Указанные картины принадлежат кисти Василия Перова, на
территории Перова обитает много видов птиц, а еще есть певица Лена
Перова. :-)

Источник:
   1. https://ru.wikipedia.org/wiki/Перов,_Василий_Григорьевич
   2. https://ru.wikipedia.org/wiki/Перово_(район_Москвы)
   3. https://ru.wikipedia.org/wiki/Перова,_Елена_Вячеславовна

Автор:
Серафим Шибанов (Москва, 2003-2016)

Вопрос 20:
Согласно источнику 1996 года, ВТОРОЙ проводит ремонтные работы в столице
Ломбардии, а ТРЕТИЙ за некоторую сумму передал посторонним лицам
предметы культа. У ПЕРВОГО, товарища ВТОРОГО и ТРЕТЬЕГО, можно
диагностировать отклонение в сексуальном поведении. Какое именно
отклонение?

Ответ:
Эксгибиционизм.

Комментарий:
"Добрыня плюнул на Россию и в Милане чинит газ. Алеша, даром что
Попович, продал весь иконостас. Один Илья пугает девок, скача в одном
носке", - поется в песне "Древнерусская тоска".
   Этот блок посвящен олдскулу и олдскульной музыке.

Источник:
   1. Аквариум - Древнерусская тоска.mp3.
   2. https://ru.wikipedia.org/wiki/Древнерусская_тоска
   3. https://ru.wikipedia.org/wiki/Милан
   4. https://ru.wikipedia.org/wiki/Эксгибиционизм

Автор:
Серафим Шибанов (Москва, 2004)

Вопрос 21:
Дмитрий Чернышёв в качестве примера сомнительного каламбура приводит
цитату из "Попрыгуньи": "В этой поездке, как непременный член общества,
будет принимать участие и сама Ольга Ивановна". На самом деле Чехов в
этой цитате использовал написанное по-русски французское слово. Какое?

Ответ:
СосьетЕ.

Комментарий:
Не "член общества", а "член сосьете".

Источник:
   1. https://mi3ch.livejournal.com/2535361.html
   2. А.П. Чехов. Попрыгунья.
http://www.feb-web.ru/feb/chekhov/texts/sp0/sp8/sp8-007-.htm

Автор:
Артем Колесов (Буй - Чистые Боры - Волгореченск - Пермь - Москва)

Вопрос 22:
   <раздатка>
   Как стройно пальцами я щелкаю,
   Шелк нот меня имел за единицу.
   Я изогнусь иглою колкою,
   Я сам - оркестр и дрожь цимбал.
   </раздатка>
   Какое слово в отрывке из стихотворения Валентина Парнаха мы заменили
выражением "имел за единицу"?

Ответ:
Заколебал.

Комментарий:
"Кол" и "единица" - это синонимы. В стихотворении Парнаха глагол
"заколебал" значит "заставил колебаться", а не то, что вы подумали.

Источник:
В.Я. Парнах. Веселый мим.
http://lanterne.ru/valentin-parnah-veselyiy-mim.html

Автор:
Серафим Шибанов (Москва)

Вопрос 23:
(pic: 20180296.jpg)
   Подписью к розданной вам картинке один интернет-пользователь сделал
название французского фильма 1956 года. Напишите это название.

Ответ:
"И Бог создал женщину".

Комментарий:
Фильм Роже Вадима.

Источник:
   1. http://funimg.ru/collections/wulffmorgenthalerrus/9268/
   2. https://ru.wikipedia.org/wiki/И_Бог_создал_женщину_(фильм,_1956)

Автор:
Артем Колесов (Буй - Чистые Боры - Волгореченск - Пермь - Москва)

Вопрос 24:
   <раздатка>
   Сняв с нее цепи и маску, они по очереди овладели ею.
   </раздатка>
   Перед вами ПРОПУСК О. Назовите американца, написавшего ПРОПУСК.

Ответ:
[Фрэнсис] Фукуяма.

Комментарий:
Пропущены слова "конец истории". На раздаточном материале - одно из
предложений из концовки романа Полин Реаж "История О".

Источник:
   1. П. Реаж. История О. http://flibusta.is/b/45802/read
   2. https://ru.wikipedia.org/wiki/Конец_истории_и_последний_человек

Автор:
Серафим Шибанов (Москва)

Тур:
11 тур. "Энергетические напитки Freon" (Ярославль)

Редактор:
Арсений Глазовский

Инфо:
Редактор благодарит за тестирование и ценные советы: Анну Мурину
(Ярославль), Дмитрия Сахарова (Тутаев), Артема Гулецкого (Минск), Юлию
Мещерякову и Владимира Чеснокова (оба - Воронеж).

Вопрос 1:
Английская шутка рассказывает о панде в ресторане, которая ест, стреляет
и уходит. А какие два действия присутствуют в аналогичной русской фразе?

Ответ:
Казнить, помиловать.

Комментарий:
Английская фраза "Panda eats shoots and leaves" (панда ест побеги и
листья) из-за неверно поставленной запятой превращается в "Panda eats,
shoots and leaves" (панда ест, стреляет и уходит). Пожалуй, самым
известным русским аналогом является фраза "Казнить нельзя помиловать" из
книги "В стране невыученных уроков".

Источник:
   1. https://en.wikipedia.org/wiki/Eats,_Shoots_%26_Leaves
   2. Л.Б. Гераскина. В стране невыученных уроков.
http://flibusta.is/b/474556/read

Автор:
Андрей Горошников

Вопрос 2:
Имя одного из злодеев в сериале "Черный Плащ" в русском переводе
утратило игру слов, поэтому его зовут просто "Тарас Бульба". Название
одной из серий с участием этого злодея совпадает с названием
произведения 1970 года. Напишите это название из двух коротких слов.

Ответ:
Час быка.

Комментарий:
В оригинале персонажа зовут Taurus Bulba: taurus по-латыни - бык, bull -
тоже бык, но по-английски. "Час быка" - роман Ивана Ефремова.

Источник:
   1. http://darkwingduck.info/index/0-53
   2. https://ru.wikipedia.org/wiki/Час_Быка

Автор:
Юрий Чертов

Вопрос 3:
Забавно, что Хью Драйден настаивал на использовании ЭТОГО СЛОВА, говоря,
что исследоваться будет всё, а не только некоторые объекты. Предложение
Драйдена было отклонено, зато простым людям стало чуть удобнее различать
соперников. Напишите это слово.

Ответ:
Космонавт.

Комментарий:
Драйден предлагал назвать американских космических путешественников
"космонавтами", а не "астронавтами", но предложение не поддержали.
Простым же людям стало в итоге чуть удобнее различать соперников по
"космической гонке".

Источник:
https://vk.com/video-55155418_456239369

Автор:
Иван Плаксин

Вопрос 4:
Герои сериала "Star Trek" попадают на планету, которой управляет мощный
компьютер. Этот компьютер дает жителям пищу, защищает от смерти и
болезней, но при этом заставляет соблюдать ряд правил. В конце серии
один из членов экипажа говорит, что капитан как будто вручил жителям...
Что?

Ответ:
Яблоко.

Зачет:
Запретный плод.

Комментарий:
События серии являются отсылкой к библейскому сюжету об изгнании из рая.
Команда "Энтерпрайза" уничтожила компьютер и нарушила райскую жизнь
аборигенов.

Источник:
Телесериал "Star Trek", The Original Series, s02e05 "Apple".

Автор:
Иван Плаксин

Вопрос 5:
В одной песне сказано, что ОН - "страшный сон Мариса Лиепы". В той же
песне сказано, что таких, как ОН, предпочитают девчонки поумнее.
Назовите ЕГО.

Ответ:
Плохой танцор.

Комментарий:
Группа "Тараканы!" считает, что плохие танцоры - это кошмар для
советского балетмейстера Мариса Лиепа. И всё же и у плохих танцоров есть
свои преимущества.

Источник:
https://music.yandex.ru/album/2155884/track/19212635

Автор:
Иван Плаксин

Вопрос 6:
   <раздатка>
   N..Demon
   </раздатка>
   Перед вами имя персонажа вселенной DC Comics, который как-то раз
составил список из десяти советских чиновников - предателей коммунизма.
Напишите буквы, которые мы пропустили в этом имени.

Ответ:
KV.

Комментарий:
NKVDemon в комиксах олицетворял собой советскую коммунистическую угрозу.
Кстати, девять чиновников из списка он таки убил, и только Михаила
Горбачёва спас Бэтмен.

Источник:
https://en.wikipedia.org/wiki/NKVDemon

Автор:
Юрий Чертов

Вопрос 7:
Фирма "БрокАр и Ко" на московской выставке в 1882 году в рекламных целях
установила необычный фонтан. В фонтане была "вода...". Из какого города?

Ответ:
Из Кёльна.

Комментарий:
Фонтан был установлен в рекламных целях к началу выпуска одеколона
"Цветочный". "Одеколон" в переводе с французского - "вода из Кёльна".

Источник:
https://ru.wikipedia.org/wiki/Брокар,_Генрих_Афанасьевич

Автор:
Павел Петухов

Вопрос 8:
Знакомый автора вопроса в шутку заметил, что если уж пытаться играть
нечестно во время викторины по астрономии, то только с помощью НЕГО.
Назовите ЕГО одним словом.

Ответ:
Спутник.

Комментарий:
Использовать поисковики во время викторины, конечно, нельзя, но
использование поискового сервиса "Спутник" хотя бы символично.

Источник:
   1. Шутка Юрия Богомолова.
   2. https://www.sputnik.ru/

Автор:
Иван Плаксин

Вопрос 9:
Оба отрицательных персонажа так и остались для зрителей безымянными. А
вот положительному имя придумал Алеша за просмотром фильма про
"неуловимых мстителей". Назовите короткую фамилию Алеши.

Ответ:
Хайт.

Комментарий:
Речь идет о серии мультфильмов про кота Леопольда. Автору сценария
Аркадию Хайту долго не удавалось придумать имя заглавного персонажа.
Васька, Барсик, Снежок, Мурзик и другие "стандартные" имена для
интеллигентного кота не подходили. В итоге знаменитого сценариста спас
сын, который предложил назвать кота в честь Леопольда Кудасова. Кстати,
имена мышей в сценарии тоже были - их звали Митя и Мотя, но в
мультфильме это отражения не нашло.

Источник:
http://polit.ru/news/2014/12/25/hayt

Автор:
Павел Петухов

Вопрос 10:
Чарли в одном интервью сказал, что ЭТО - всего лишь экран смартфона или
телевизора. Назовите ЭТО.

Ответ:
Черное зеркало.

Комментарий:
"Черное зеркало", вынесенное в название известного сериала, если верить
его создателю Чарли Брукеру, - всего лишь выключенный экран телевизора,
монитор или смартфон.

Источник:
http://www.furfur.me/furfur/culture/culture/176445-uppit-o-chernom-zerkale

Автор:
Арсений Глазовский

Вопрос 11:
В серии игр "Mass Effect" [масс эффЕкт] многие объекты названы в честь
известных космонавтов. Есть, например, звезда "Терешкова" или
космическая станция "Гагарин". Напишите название любой из четырех планет
звездной системы "Память", находящейся в Центре Аида.

Ответ:
[Владимир] Комаров.

Зачет:
[Георгий] Добровольский; [Владислав] Волков; [Виктор] Пацаев.

Комментарий:
Все планеты в системе названы в честь трагически погибших советских
космонавтов. Владимир Комаров погиб при испытании "Союза-1", остальные
трое - из-за разгерметизации "Союза-11".

Источник:
   1. http://ru.masseffect.wikia.com/wiki/Память
   2. https://ru.wikipedia.org/wiki/Комаров,_Владимир_Михайлович
   3. https://ru.wikipedia.org/wiki/Союз-11

Автор:
Алексей Гришаев

Вопрос 12:
Статья об исследовании, объясняющем появление большого количества
научных работ низкого качества, называлась "ОНИ выяснили, откуда берутся
ОНИ". Назовите ИХ двумя словами.

Ответ:
Британские ученые.

Комментарий:
Данное исследование и правда было проведено университетами Бристоля и
Эксетера.

Источник:
https://www.popmech.ru/science/288942-britanskie-uchenye-vyyasnili-otkuda-berutsya-britanskie-uchenye/

Автор:
Иван Плаксин

Вопрос 13:
В разное время ИКСОВ играли Скотт Эдкинс, Арнольд Шварцнеггер и Дольф
Лундгрен. Джефф Бриджес ИКСА не играл, но сильно поспособствовал
популярности "ТАКОГО ИКСА". Какого - ТАКОГО?

Ответ:
Белого.

Комментарий:
Упомянутые актеры играли русских, а вот Джефф Бриджес, хоть никогда не
играл русских, поспособствовал популярности коктейля "Белый русский",
который является одним из атрибутов персонажа Бриджеса в фильме "Большой
Лебовски".

Источник:
   1. https://www.kinopoisk.ru/name/31319/
   2. https://www.kinopoisk.ru/name/6264/
   3. https://www.kinopoisk.ru/name/4768/
   4. https://ru.wikipedia.org/wiki/Белый_русский

Автор:
Юрий Чертов

Вопрос 14:
Галина и Мария Дайн в своей книге рассказывают про соломенных кукол,
которых иногда ставили рядом с окнами. Эти куклы помогали бороться с
ЭТИМ. Автомобилисты для борьбы с ЭТИМ нередко используют специальную
жидкость. Назовите ЭТО.

Ответ:
Иней.

Зачет:
Лед [на окнах/стеклах].

Комментарий:
Солома хорошо впитывает влагу, поэтому лед и иней на окнах не
образовывались. Автомобилисты обрабатывают стекла специальными
жидкостями, которые также препятствуют образованию льда и инея.

Источник:
   1. Галина Дайн, Мария Дайн. "Как войти в народную культуру".
   2. http://cartore.ru/261-zamerzanie-stekol-v-avtomobile-chto-delat-chtoby-stekla-ne-zamerzali.html

Автор:
Павел Петухов

Вопрос 15:
СОмбра - персонаж игры "Overwatch" [овервОч]. Одна из ее способностей
называется "Транслокатор". После какого события конца 2016 года
появилось много однотипных шуток о полезности и актуальности этой
способности?

Ответ:
Выборы президента США.

Зачет:
По смыслу.

Комментарий:
Сомбра - мексиканка, что можно предположить, например, по созвучию имени
со словом "сомбреро". А "транслокатор" помогает преодолевать
препятствия, к коим относится и стена между США и Мексикой, которую
грозился построить Дональд Трамп.

Источник:
https://pikabu.ru/story/a_ya_zdes_uzhe_davno_4874454

Автор:
Андрей Бритнев

Вопрос 16:
Закончите стихотворение-порошок одним словом:
   матроскин кто был твой хозяин
   с которым ты объездил мир
   я называл его обычно
   ...

Ответ:
мессир

Комментарий:
Автор проводит параллель между двумя котами - уже упомянутым Матроскиным
и булгаковским Бегемотом.

Источник:
https://vk.com/sandalporoshki?w=wall-31481258_88833

Автор:
Андрей Горошников

Вопрос 17:
Автор одного ролика на Youtube от имени мутанта Росомахи рассказывает о
не самом приятном будущем, которое наступило после запрета ГМО. В конце
ролика зритель узнаёт, что всё это время рядом с ним находился еще один
мутант. Ответьте максимально точно: в чем заключается суперспособность
этого второго мутанта?

Ответ:
Транслировать всё увиденное и услышанное в видеоформате в прошлое на
Youtube.

Зачет:
По смыслу.

Комментарий:
Ролик содержит аллюзию на фильм "Люди Икс: Дни минувшего будущего", в
котором Росомаха пытается предупредить людей в прошлом о надвигающейся
опасности. В данном случае именно благодаря безымянному мутанту мы и
можем теперь посмотреть этот ролик, за что должны быть ему очень
благодарны.

Источник:
https://www.youtube.com/watch?v=oKRS2P-HMrs

Автор:
Арсений Глазовский

Вопрос 18:
В конце одного фильма героиня ДЕЛАЕТ ИКС на могиле. Что в этом вопросе
мы заменили ИКСОМ?

Ответ:
Ничего.

Комментарий:
Героиня фильма "Логан", который относится к франшизе "Люди Икс",
превращает крест на могиле в букву X [икс].

Источник:
Х/ф "Логан" (2017), реж. Джеймс Мэнголд.

Автор:
Андрей Бритнев

Вопрос 19:
Нина Яковлева вспоминает, что однажды летом в Канаде одна социальная
реклама призывала сломать ЕЕ сразу после использования. Назовите ЕЕ.

Ответ:
Спичка.

Комментарий:
Таким образом в летнее время канадцы боролись с пожарами, причиной
которых легко могла стать брошенная непотушенная спичка. Зачастую это
происходит не из злого умысла, а из-за обыкновенной невнимательности.
При этом, чтобы сломать спичку, ее придется потушить, чтобы случайно не
обжечься. По воспоминаниям Нины Ивановны, в итоге количество пожаров в
стране сократилось почти втрое, хотя, возможно, это просто совпадение.

Источник:
Разговор автора вопроса с Ниной Яковлевой.

Автор:
Павел Петухов

Вопрос 20:
   <раздатка>
   Matthew Gamble
   Barbara Schett
   Mats Wilander
   </раздатка>
   По мнению автора вопроса, молодой теннисист Мэттью Геймбл подошел бы
для шоу, ведущими которого сейчас являются Барбара Шетт и Матс Виландер.
Как бы тогда звучало название этого шоу?

Ответ:
"Gamble, Schett and Mats".

Зачет:
"Gamble, Schett, Mats"; "Gamble. Schett. Mats".

Комментарий:
Название шоу обыгрывает традиционное объявление победителя теннисного
матча - "Гейм, сет, матч".

Источник:
   1. Телеканал "Eurosport".
   2. Фантазия автора вопроса.

Автор:
Илья Козырев

Вопрос 21:
(pic: 20180297.jpg)
   По мнению автора вопроса, логотип компании "Мазда" служит прекрасной
демонстрацией утверждения, известного с начала 1990-х. Дело в том, что
многим кажется, что на эмблеме изображен какой-то цветок. Однако, по
задумке автора, там находится ОНА. Кто ОНА?

Ответ:
Сова.

Комментарий:
Прекрасная демонстрация того, что "Совы - не то, чем они кажутся".
Данное утверждение прозвучало в сериале "Твин Пикс" в начале 1990-х, а
совсем недавно оно же использовалось наряду с еще несколькими как
рекламный слоган третьего сезона сериала.

Источник:
http://fb.ru/article/360668/logotip-mazda-istoriya-sozdaniya

Автор:
Арсений Глазовский, по идее Яны Баловой

Вопрос 22:
Ученые полагают, что система небольших отверстий в храмовом комплексе
ЧавИн-де-УАнтара в Андах позволяла имитировать ЕГО. Можно сказать, что в
начале XXI века другой ОН был на гонках "Формулы-1". Назовите ЕГО двумя
словами.

Ответ:
Рев ягуара.

Зачет:
Рев "Ягуара".

Комментарий:
Ягуар нередко занимал одно из центральных мест в пантеонах
южноамериканских цивилизаций. Команда "Jaguar Racing" участвовала в
гонках "Формулы-1" с 2000 по 2004 год.

Источник:
   1. Журнал "Атлас", выпуск 413.
   2. https://ru.wikipedia.org/wiki/Ягуар_(команда_%C2%ABФормулы-1%C2%BB)

Автор:
Илья Козырев

Вопрос 23:
На острове БАрра расположен аэропорт, который работает нерегулярно.
Можно сказать, что всему виной ОНА. Назовите ЕЕ.

Ответ:
Луна.

Комментарий:
Аэропорт расположен на берегу острова, и взлетно-посадочная полоса во
время прилива скрывается под водой. Приливы и отливы являются
результатом изменения положений Луны. Если вы фанат Бориса Моисеева, то
форма вопроса также могла помочь ответить.

Источник:
   1. https://ru.wikipedia.org/wiki/Барра_(аэропорт)
   2. https://ru.wikipedia.org/wiki/Прилив_и_отлив

Автор:
Алексей Гришаев

Вопрос 24:
Герой одного фильма рассказывает, что встречался с вампиршей и долго
терпел ее странности, но в итоге всё же расстался с ней. Физически герой
не пострадал, но предложение заняться сексом в гробу всё же считает ЕЮ.
Назовите ЕЕ двумя словами.

Ответ:
Последняя капля.

Комментарий:
Хотя вампирша и не пила его кровь, герой всё равно говорит о последней
капле.

Источник:
Телесериал "Castle", s02e06 "Vampire Weekend".

Автор:
Арсений Глазовский

Тур:
12 тур. "Ingria Interna" (Санкт-Петербург)

Редактор:
Сергей Андреевич Григорьев и Юлия Дидбаридзе

Инфо:
Спасибо тестерам: Дмитрию Ожигову и Андрею Петухову (оба -
Санкт-Петербург), Александре Балабан и Тимофею Бокову (оба - Москва),
Елизавете Коротковой и Михаилу Папкову (оба - Тарту), Ирине Пинегиной
(Киров).

Вопрос 1:
В заголовке англоязычной статьи, посвященной нашумевшим
гипервычислениям, Кристоф Тойшер задается вопросом, являются ли они всё
же вычислениями или ИМ. Назовите ЕГО.

Ответ:
Хайп.

Комментарий:
Кристоф Тойшер находит в слове "hypercomputations" слово "hype" и
считает, что это достаточно смешно, чтобы вынести в заглавие статьи.
Пошумим!

Источник:
https://dl.acm.org/citation.cfm?id=545170

Автор:
Анна Свирина

Вопрос 2:
[Ведущему: кавычки в вопросе не озвучивать.]
   ОНО есть в заголовке недавней статьи, в которой рассказывается о Ван
Дейке и Левицком, Вермеере и Венецианове. Из него может получиться
"запрет" и не может "пармезан". Назовите ЕГО.

Ответ:
Импортозамещение.

Источник:
https://arzamas.academy/materials/1365

Автор:
Сергей Андреевич Григорьев

Вопрос 3:
Симпатичная героиня современного фильма, отказываясь от предложения
выпить, ДЕЛАЕТ ЭТО. Герой американской песни, название которой можно
перевести как "ДЕЛАТЬ ЭТО", говорит, что не придирчив, просто знает
подходящий ему тип. Какие два слова мы заменили словами "ДЕЛАТЬ ЭТО"?

Ответ:
Свайпить влево.

Зачет:
Смахивать влево.

Источник:
   1. Х/ф "Kingsman: Золотое кольцо" (2017), реж. Мэттью Вон.
   2. https://www.youtube.com/watch?v=kKyHp3w_JdU

Автор:
Юлия Дидбаридзе

Вопрос 4:
Прослушайте цитату из рецензии на оперу "Борис Годунов", в которой мы
заменили два слова: "Надменная, чопорная знать под звуки кадрили
предается хвастливым мечтам "кадрить Московско царство"". Напишите
замененные нами слова.

Ответ:
Полонез, полонить.

Источник:
http://www.mussorgsky.ru/hubov187.html

Автор:
Сергей Андреевич Григорьев

Вопрос 5:
В восемнадцатом веке из-за одной моды между любовниками при поцелуе
иногда пробегала искра. Чтобы добиться такого эффекта, девушкам
приходилось в чем-то уподобляться сказочной героине. Какой?

Ответ:
Золушке.

Комментарий:
Это было время хайпа по электричеству. Девушки заряжались статическим
электричеством и, чтобы не заземлиться, носили стеклянные туфли.

Источник:
Ф. Эшкрофт. Искра жизни. Электричество в теле человека.
https://books.google.ru/books?id=DmZoBgAAQBAJ&pg=PA26#v=onepage&q&f=false

Автор:
Анна Свирина

Вопрос 6:
(pic: 20180298.jpg)
   [Ведущему: прочитать "мур мур" медленно и нежно.]
   Розданную вам фотографию, сделанную на юге Европы, автор нашел по
запросу "мур мур". Восстановите два слова, в которых мы пропустили
буквы.

Ответ:
Мурал Мурсии.

Зачет:
Мурсийский мурал; мурал Мурсия; muro murciano.

Комментарий:
Перед вами граффити на одной из стен в Мурсии.

Источник:
   1. Результаты поиска в Гугл-картинках по запросу "мурсийский мурал".
   2. http://www.unurth.com/filter/Murcia

Автор:
Юлия Дидбаридзе

Вопрос 7:
[Ведущему: цитату в вопросе не выделять.]
   В рецензии на новый сборник рассказов Стивена Кинга журналист пишет,
что вместе с наркотиками и бухлом осталось позади и ПРОПУСК1. Герой
романа ПРОПУСК2 рассуждает, что "раньше и птицы пели звонче, <...> и
вода была мокрее, <...> и старики моложе...". Восстановите любой из
пропусков, на слух различающихся только ударением.

Ответ:
Время "Оно".

Зачет:
"Время оно".

Источник:
   1. http://itsmyday.ru/yes/75759
   2. М.Г. Успенский. Время Оно.
https://books.google.ru/books?id=ZCJIDwAAQBAJ&pg=PT200#v=onepage&q&f=false

Автор:
Сергей Андреевич Григорьев

Вопрос 8:
[Ведущему: кавычки в вопросе не озвучивать.]
   В песне Михаила Елизарова "Советская" есть разные отсылки к советской
истории и культуре. В том числе, ПЕРВЫЙ из "ВТОРОГО". С 1991 года четыре
ПЕРВЫХ сборной России по хоккею пришлись на ВТОРОЙ. Что мы заменили
словами "ПЕРВЫЙ" И "ВТОРОЙ"?

Ответ:
Проигрыш, День Победы.

Комментарий:
В песне Михаила Елизарова используется музыка из проигрыша песни "День
Победы". Матчи сборной России на чемпионатах мира по хоккею с шайбой 15
раз выпадали на девятое мая. Из них сборная одиннадцать выиграла и
четыре проиграла.

Источник:
   1. https://www.youtube.com/watch?v=TdSpZF36OHg
   2. http://wildice.ru/p/40/ch/WRL_WC_1997_F/stg/all/tour/all/team1/RUS
   3. http://wildice.ru/p/40/ch/WRL_WC_1999_F/stg/all/tour/all/team1/RUS
   4. http://wildice.ru/p/40/ch/WRL_WC_2011_F/stg/all/tour/all/team1/RUS
   5. http://wildice.ru/p/40/ch/WRL_WC_2013_F/stg/all/tour/all/team1/RUS

Автор:
Андрей Петухов, Сергей Андреевич Григорьев

Вопрос 9:
В проекте арт-группы "Зер Гут", посвященном судьбе искусства в эпоху
потребления, созданные художниками "графические" портреты постепенно
исчезают. Ответьте названием фильма: кто в этом проекте символизирует
современного зрителя?

Ответ:
Птицы.

Комментарий:
Потребление здесь происходит в прямом смысле. Созданные семечками
портреты постепенно склевывают птицы.

Источник:
http://www.gif.ru/afisha/rus-videoart/view_print/

Автор:
Юлия Дидбаридзе

Вопрос 10:
[Ведущему: выделить цитату в вопросе.]
   Внимание, в вопросе есть замена.
   В романе Фолкнера герой в июне смотрит на гребца, который, как
кажется, "через другой месяц, через другое лето <...> будет
величественно грести в царственном одиночестве". Какое слово в цитате мы
заменили?

Ответ:
Августейшем.

Комментарий:
Через другой от июня месяц гребец будет уже действительно в августейшем
одиночестве.

Источник:
У. Фолкнер. Звук и ярость. http://flibusta.is/b/437928/read

Автор:
Сергей Андреевич Григорьев

Вопрос 11:
Около месяца назад театральный критик Виктор Вилисов решил сделать
подарок подписчикам и выложить в своем паблике редкие записи спектаклей.
Но работы одного французского режиссера он не стал выкладывать, посчитав
это постироничным. Напишите имя этого режиссера.

Ответ:
Оливье.

Комментарий:
Вилисов говорил о записях спектаклей Оливье Пи, но в Новый год с оливье
лучше не перебарщивать.

Источник:
https://vk.com/apolloniada?w=wall-129481581_1835

Автор:
Юлия Дидбаридзе

Вопрос 12:
(pic: 20180299.jpg)
   [Ведущему: сделать небольшую паузу между "с" и "Юр" в первом
предложении.]
   В песне "Тает лед" слово "велюр" рифмуется с "Юр". Комментируя клип
на эту песню, Владимир Раннев упоминает известного человека. Кого?

Ответ:
[Рене] Магритта.

Комментарий:
На розданном вам кадре из клипа Владимир Раннев увидел что-то от
котелков с картин художника-сюрреалиста Магритта и назвал происходящее
сюрреалистической шизофренией. В первом предложении вопроса вам мог
послышаться "сюр".

Источник:
http://www.sobaka.ru/city/music/66515

Автор:
Юлия Дидбаридзе

Вопрос 13:
Этим вопросом мы отмечаем середину нашего пакета.
   Философ Моисей Мендельсон называет АЛЬФОЙ внешнюю сторону культуры со
всем ее блеском. Какое латинское слово мы заменили АЛЬФОЙ?

Ответ:
Политура.

Зачет:
Politura.

Комментарий:
Политура - это глянцевое покрытие для древесины. Вопросом про политуру
мы отмечаем полтура.

Источник:
http://www.deutschestextarchiv.de/book/view/mendelssohn_aufklaeren_1784?p=3

Автор:
Юлия Дидбаридзе

Вопрос 14:
Александр Эткинд, отмечая, что дворянство и интеллигенция отличались от
крестьян и с точки зрения культуры, и внешне, заменил две буквы тремя в
одном из слов в названии стихотворения конца XIX века. Напишите
получившееся словосочетание.

Ответ:
Бремя бритого человека.

Источник:
А.М. Эткинд. Внутренняя колонизация. Имперский опыт России.
http://flibusta.is/b/492636/read

Автор:
Сергей Андреевич Григорьев

Вопрос 15:
В книге Татьяны Левановой героев, способных путешествовать между мирами,
называют ИКСАМИ. По одной версии, известные строчки появились после
того, как Александр во время свидания пожаловался Михаилу на ИКСЫ. Что
мы заменили ИКСОМ?

Ответ:
Сквозняк.

Комментарий:
Речь идет о Михаиле Круге и начале припева песни "Владимирский централ".

Источник:
   1. Татьяна Леванова, серия книг "Сквозняки".
   2. http://veligrad.ru/category/vladimir/page/2/

Автор:
Анна Свирина, Юлия Дидбаридзе, Сергей Андреевич Григорьев

Вопрос 16:
В этом вопросе словом "ТАКОЙ" мы заменили другое слово.
   Среди ныряльщиков Великобритании имеет хождение предостерегающая
поговорка: "Дьявол ТАКОЙ Пит тридцать футов сторожит". В название какого
напитка входит слово "ТАКОЙ"?

Ответ:
Кислородный коктейль.

Комментарий:
Имеется в виду кислородное отравление, которое возникает при дыхании
воздухом при спуске более чем на восемь метров.

Источник:
Ф. Эшкрофт. На грани возможного. Наука выживания.
http://flibusta.is/b/479105/read

Автор:
Петр Назаров

Вопрос 17:
Дмитрий Кузьмин, рассматривая разные аспекты ситуации с денежными
выплатами спортсменам, не допущенным на Олимпийские игры 2016 года,
использует выражение "ОНА". В этом году на НЕЙ можно будет увидеть пхиып
и чхиыт. Назовите ЕЕ тремя словами.

Ответ:
Обратная сторона медали.

Зачет:
Другая сторона медали.

Комментарий:
Символом Олимпиады в Корее стали две буквы из корейского алфавита
Хангыль, входящие в написание слова "Пхёнчхан".

Источник:
   1. https://www.gazeta.ru/sport/rio2016/2016/10/31/a_10295645.shtml
   2. http://olympteka.ru/olymp/game/simbols/52.html
   3. https://ru.wikipedia.org/wiki/Хангыль#Названия_согласных

Автор:
Надежда Напольская

Вопрос 18:
Во время так называемой "битвы за Лос-Анджелес" в 1942 году 40% погибших
умерли от сердечных приступов. Назовите двумя словами, начинающимися на
одинаковые буквы, причину гибели бОльшей части.

Ответ:
Автомобильные аварии.

Комментарий:
Всего во время "битвы" в 1942 году погибло пять человек. Трое погибли в
авариях, а двое умерли от сердечных приступов, вызванных стрессом.
Причиной канонады стал неправильно идентифицированный метеорологический
зонд.

Источник:
https://warspot.ru/5390-foto-dnya-bitva-za-los-andzheles

Автор:
Сергей Андреевич Григорьев

Вопрос 19:
Поговорка "вусмерть расшибить голову об уголок торы", распространенная в
одной восточной стране, означает смерть невозможным или нелепым
способом. В предыдущем предложении мы изменили одно слово. Напишите его
в исходном виде.

Ответ:
Тофу.

Комментарий:
На самом деле в поговорке говорится об уголке тофу.

Источник:
https://ru.wikiquote.org/wiki/Японские_пословицы

Автор:
Юлия Дидбаридзе

Вопрос 20:
Дуплет.
   1. Вальтер Беньямин пишет, что многие снимки одного из пионеров
фотографии Дэвида Хилла сделаны на НЕМ, для того чтобы ничего не
отвлекало модель. Назовите ЕГО.
   2. Герой одной пьесы, удивленный тяжестью подарка от жены, которая,
как ему кажется, давно охладела к нему, предполагает, что там ОНО.
Назовите ЕГО.

Ответ:
   1. Кладбище.
   2. Надгробие.

Источник:
   1. В. Беньямин. Краткая история фотографии.
http://flibusta.is/b/390387/read
   2. Дж. Голдмен. Лев зимой. http://flibusta.is/b/426977/read

Автор:
Юлия Дидбаридзе, Сергей Андреевич Григорьев

Вопрос 21:
Лев ОсповАт сравнил с НЕЙ неудовлетворительную оценку, полученную Лоркой
на экзамене. В стихотворении Лорки с НЕЙ визуально рифмуется луна,
которая в творчестве поэта часто ассоциируется со смертью. Назовите ЕЕ.

Ответ:
Арена для корриды.

Зачет:
Арена для боя быков; арена.

Комментарий:
Ноль, поставленный на работе Лорки, своей формой напомнил Осповату
арену.

Источник:
   1. Л.С. Осповат. Гарсиа Лорка. http://flibusta.is/b/76216/read
   2. http://ciudadseva.com/texto/llanto-por-ignacio-sanchez-mejias/

Автор:
Юлия Дидбаридзе

Вопрос 22:
На монете номиналом в два евро, выпущенной в 2014 году, изображены
отпечатки ботинок на песке. Какой день упомянут в надписи на этой
монете?

Ответ:
D-Day.

Зачет:
D; Д; День Д; 6 июня 1944 года.

Комментарий:
В 2014 году высадке в Нормандии исполнилось 70 лет.

Источник:
http://www.coin-database.com/coins/2-euro-70th-anniversary-of-the-d-day-france-2014.html

Автор:
Сергей Андреевич Григорьев

Вопрос 23:
На одной карикатуре говорится, что ОНА данных о токсичности составляет 2
килограмма на килограмм веса. ЕЕ можно обозначить четырьмя символами.
Напишите любой из них.

Ответ:
L.

Зачет:
D; 5; 0; Л; Д.

Комментарий:
LD50 - средняя доза вещества, вызывающая гибель половины членов
испытуемой группы. Картинка смешная, но некорректная, так как испытуемая
группа, по-видимому, состояла из одного человека. Довольно широким
зачетом и предыдущими двумя вопросами мы попытались сделать этот вопрос
нелетальным.

Источник:
   1. https://xkcd.com/1260/
   2. https://en.wikipedia.org/wiki/Median_lethal_dose

Автор:
Анна Свирина

Вопрос 24:
Одно из достижений в игре "Hearts of Iron IV" можно получить, если,
играя за Испанию, опередить немцев и внезапно захватить Париж. На иконке
этого достижения изображен человек в красном, а его название можно
перевести на русский как ПРОПУСК. А кого ПРОПУСК в песне 1998 года?

Ответ:
Полковника.

Комментарий:
Пропущены слова "Никто не ждет". Наверное, к концу тура уже никто и не
ждал вопроса про испанскую инквизицию. Песня группы "Би-2" "Полковнику
никто не пишет" была записана в 1998 году.

Источник:
   1. https://hoi4-ru.paradoxwikis.com/Достижения
   2. https://ru.wikipedia.org/wiki/Полковнику_никто_не_пишет_(песня)

Автор:
Сергей Андреевич Григорьев, Юлия Дидбаридзе

Тур:
13 тур. "Кайло Рен убьет Сноука" (Москва - Тверь)

Вопрос 1:
   <раздатка>
   ACIDIC SOD OVEN FOP
   </раздатка>
   В начале одного из эпизодов мультсериала "Симпсоны", рассказывающего
о заговоре и поисках Лизой истинного сокровища, можно заметить надпись
"ACIDIC SOD OVEN FOP", что является ключом к пониманию сюжета. А
отсылкой к какому произведению является данный эпизод?

Ответ:
"Код да Винчи".

Зачет:
"The Da Vinci Code".

Комментарий:
Розданный текст - это анаграмма "DA VINCI COD SPOOF" (что можно
перевести как "пародия Код да Винчи").

Источник:
Мультсериал "Симпсоны", s20e13, 45-я секунда.

Автор:
Леонид Гольденберг

Вопрос 2:
Статья про киборгизацию называет ЕГО одним из старинных искусственных
усилений человека. Уважаемые знатоки! Через минуту назовите ЕГО.

Ответ:
Крюк.

Комментарий:
Форма вопроса напоминает о телевизионном "Что? Где? Когда?" и фамилии
его ведущего.

Источник:
http://22century.ru/docs/cyborgization

Автор:
Оксана Коробейникова

Вопрос 3:
Героиня экранизации начала 1930-х годов на вопрос своего жениха о ее
мистическом выздоровлении отвечает: "Я никогда не чувствовала себя
лучше". Какие два слова мы пропустили в предыдущем предложении?

Ответ:
В жизни.

Комментарий:
Экранизация - "Дракула" - фильм ужасов Тода Броунинга 1931 года. Фраза
принадлежит Мине Мюррей и намекает зрителю на причины ее выздоровления.

Источник:
Х/ф "Дракула" (1931), реж. Тод Броунинг.

Автор:
Леонид Гольденберг

Вопрос 4:
На тайоре, одном из языков австралийских аборигенов, отсутствуют понятия
"лево" и "право". Поэтому в общении с ними вам могло бы помочь
изобретение предположительно XII-XIII веков. Назовите это изобретение.

Ответ:
Компас.

Комментарий:
На тайоре все пространственные направления обозначаются абсолютными
понятиями: север, северо-восток, запад и т.д. Фразы вроде "ложка лежит
северо-западнее тарелки" для них совершенно обычны.

Источник:
   1. https://theoryandpractice.ru/posts/5662-izuchenie-yazyka-pomozhet-ponyat-chto-delaet-nas-lyudmi-psikholog-lera-boroditski-o-tom-kak-yazyk-formiruet-myshlenie
   2. https://ru.wikisource.org/wiki/ЭСБЕ/Компас

Автор:
Оксана Коробейникова

Вопрос 5:
   <раздатка>
   "Remington for my Kobein"
   </раздатка>
   В названии грайндкор-группы "Remington for my Kobein" мы заменили
одного советского музыканта американским. Назовите машиностроительную
компанию, которую мы заменили оружейной.

Ответ:
"Ikarus". Незачет: "Москвич".

Комментарий:
Группа называется "Ikarus for my Tsoy". Кобейн принял смерть от ружья
фирмы "Remington", а Виктор Цой - от "Икаруса".

Источник:
   1. https://vk.com/ikarusformytsoy
   2. https://ru.wikipedia.org/wiki/Цой,_Виктор_Робертович#Смерть_в_автокатастрофе
   3. https://en.wikipedia.org/wiki/Suicide_of_Kurt_Cobain

Автор:
Александр Костров

Вопрос 6:
Несмотря на то что дословно ИКС переводится как "раскол", "расщепление",
в музыкальной сфере этот термин обозначает совместную деятельность. ИКС
- фильм 2016 года и город на Адриатическом побережье. Назовите ИКС.

Ответ:
Сплит.

Комментарий:
Сплит - музыкальный альбом, на котором присутствуют композиции двух
(реже - больше) разных исполнителей. Отличается от сборника тем, что
включает в себя по несколько композиций каждого исполнителя, а не по 1-2
песни многих исполнителей. Главный герой фильма "Сплит" страдает
расстройством множественной личности (также известно как "расщепление
личности"). Хорватский город Сплит расположен на побережье Адриаики.

Источник:
   1. https://ru.wikipedia.org/wiki/Сплит_(звукозапись)
   2. https://ru.wikipedia.org/wiki/Сплит_(фильм)
   3. https://ru.wikipedia.org/wiki/Сплит_(город)

Автор:
Александр Костров

Вопрос 7:
Название этой московской рок-группы обыгрывает знаменитую спойлерную
фразу и переводится с английского языка как "посмотри, ПРОПУСК".
Характерно, что в первоисточнике данная фраза именно в этой формулировке
не встречается. Заполните пропуск тремя словами.

Ответ:
Я твой отец.

Комментарий:
Группа называется "Look, I'm Your Father". В фильме "Империя наносит
ответный удар" Дарт Вейдер в ответ на обвинения в убийстве отца Люка
говорит: "Нет, я твой отец".

Источник:
   1. https://vk.com/lookimyourfather
   2. https://www.youtube.com/watch?v=BOZzB_JYaE4

Автор:
Александр Костров

Вопрос 8:
В одной рекламе люди в очках постоянно мажут по мячику для пинг-понга и
танцуют не в такт музыке. Догадавшись, что рекламирует этот ролик,
ответьте, какие два слова мы пропустили в тексте вопроса.

Ответ:
Виртуальной реальности.

Зачет:
Google glass; Гугл гласс.

Комментарий:
В ролике люди в очках виртуальной реальности показывают, каково это -
жить с задержкой по времени, и тем самым рекламируют высокоскоростной
Интернет.

Источник:
https://vk.com/geekpicnic?w=wall-28587408_14090

Автор:
Оксана Коробейникова

Вопрос 9:
Зигмунд Фрейд считал прототипом этого человека легендарного египтянина
Осарсифа. По мнению Фрейда, Осарсиф покинул Египет после смерти ЭТОГО
ФАРАОНА. Назовите ЭТОГО ФАРАОНА.

Ответ:
Эхнатон.

Комментарий:
Эхнатон во время своего правления провел в Египте религиозную реформу и
перевел страну на монотеистические рельсы, однако после его смерти всё
вернулось на круги своя. По легенде, пересказанной Манефоном, его
последователь Осарсиф предпринял попытку объединить под эгидой веры в
Атона другой народ, совершив вместе с ним побег из Египта. Фрейд
отождествлял Осарсифа и Моисея. Впрочем, современные исследователи
данную точку зрения не разделяют.

Источник:
   1. https://ru.wikipedia.org/wiki/Моисей#Моисей_и_Эхнатон
   2. https://ru.wikipedia.org/wiki/Осарсиф

Автор:
Александр Костров

Вопрос 10:
В Калифорнии археологами была обнаружена двухметровая статуя Сфинкса.
Раскопки проходили в дюнах Гуадалупе-Нипомо, южнее Сан-Франциско. В
первой половине XX века это место выбрали потому, что оно схоже с
природой Египта. Ответьте, использовав числительное: для чего?

Ответ:
Для съемки фильма "Десять заповедей".

Комментарий:
Статуя сфинкса и остальные сооружения служили декорациями. Вывозить их
оказалось дорого, решили закопать.

Источник:
https://life.ru/t/история/1066649/v_pieskakh_kalifornii_otkopali_100-lietniuiu_statuiu_sfinksa

Автор:
Александр Костров

Вопрос 11:
Согласно лингвистическим и генетическим исследованиям, ОНИ покинули
территорию Египта в количестве, не превышающем тысячу человек. Сегодня в
мире ИХ насчитывается около десяти миллионов. Мы не просим вас назвать
ИХ. Название какой страны мы заменили Египтом?

Ответ:
Индия.

Комментарий:
ОНИ - это цыгане, а не евреи.

Источник:
https://ru.wikipedia.org/wiki/Цыгане#Ранняя_история_(VI%E2%80%94XV_века)

Автор:
Александр Костров

Вопрос 12:
В иврите у пяти букв имеются по две формы написания, одна из которых
используется в том случае, если буква стоит в конце слова. Некоторые
ученые считают, что это связано с тем, что в древние времена в иврите не
было ИХ. Назовите ИХ одним словом.

Ответ:
Пробелы.

Комментарий:
Одна из версий появления конечных букв: поскольку в древности слова
писались слитно, то конечные буквы были необходимы для разделения слов.
Возможно, особая форма написания была у всех букв, но до нас дошли лишь
эти пять. В родственном арабском языке сохраняется различное написание
начальных, срединных и конечных букв.

Источник:
https://ru.wikipedia.org/wiki/Иврит#Конечные_буквы

Автор:
Александр Костров

Вопрос 13:
   <раздатка>
   I got the prose of a pro your shit's subpar
   You're a pirate, you even stole my ПРОПУСК
   </раздатка>
   Перед вами текстовой отрывок одного из видеороликов из серии "Epic
Rap Battles of History". Соперниками в данном выпуске являются Джон
Толкин, которому и принадлежит реплика на раздатке, и Джордж Мартин.
Восстановите пропуск.

Ответ:
R.R.

Комментарий:
Речь идет об инициалах. В этом юмористическом ролике Джон Р.Р. Толкин
обвиняет Джорджа Р.Р. Мартина в плагиате и утверждает, что он украл даже
его инициалы. Толкин - Рональд Руэл, Мартин - Рэймонд Ричард.

Источник:
https://www.youtube.com/watch?v=XAAp_luluo0

Автор:
Александр Костров

Вопрос 14:
Кукушка обычно не тревожит небольших птиц, но иногда она делает
фатальную ошибку, и ее детеныш уподобляется персонажу известного
произведения. Назовите этого персонажа.

Ответ:
Винни-Пух.

Комментарий:
Кукушка откладывает яйцо в дупле синицы, и впоследствии слишком большой
по размеру птенец не может из него выбраться.

Источник:
http://www.ecosystema.ru/08nature/birds/085.php

Автор:
Оксана Коробейникова

Вопрос 15:
В канонических источниках не сообщается, когда ОН был снят. Владимир
Угличин снял "ЕГО" в 2006 году. Особенно широко ОН распространен на
Большом Барьерном Рифе. Назовите ЕГО двумя словами.

Ответ:
Терновый венец.

Комментарий:
В первом случае это головной убор, во втором - фильм, в третьем -
морская звезда.

Источник:
   1. https://www.google.ru/search?q=терновый+венец
   2. https://www.kinopoisk.ru/film/ternovyy-venec-2006-434703/
   3. https://ru.wikipedia.org/wiki/Терновый_венец_(морская_звезда)

Автор:
Александр Костров

Вопрос 16:
В одном из видеороликов из серии "Epic Rap Battles of History"
противником автора "Происхождения видов" Чарльза Дарвина является
главный герой японского аниме-сериала, основанного на популярной серии
видеоигр. Вышедшая в 2016 году игра из этой серии произвела настоящий
фурор. Напишите название этой игры.

Ответ:
"Pok&eacute;mon Go".

Комментарий:
Чарльз Дарвин - отец теории эволюции. Процесс перехода покемона из одной
формы в другую также называется эволюцией.

Источник:
   1. https://www.youtube.com/watch?v=EC7H99-7wxk
   2. https://ru.wikipedia.org/wiki/Pok%C3%A9mon_Go

Автор:
Александр Костров

Вопрос 17:
Помимо блокировки торрентов, российские власти намереваются "СДЕЛАТЬ
ЭТО". Ответьте тремя словами: где ЭТО полагается СДЕЛАТЬ, согласно
известной традиции?

Ответ:
В доме покойника.

Зачет:
В квартире покойника, в доме/квартире мертвеца и т.п. по смыслу.

Комментарий:
СДЕЛАТЬ ЭТО - закрыть зеркала.

Источник:
https://rg.ru/2016/02/11/roskomnadzor-kontrol-trafika-interneta-ne-kasaetsia-perepiski-grazhdan.html

Автор:
Оксана Коробейникова

Вопрос 18:
Советский экономист Дмитрий Трофимович, участвовавший в попытке смещения
Хрущёва с поста Генерального секретаря во время июньского пленума ЦК
1957 года, впоследствии написал мемуары, которые озаглавил
"Неприкаянный". Восстановите в первоначальном виде слово, которое мы
изменили.

Ответ:
"Непримкнувший".

Комментарий:
В докладе "Об изменениях в Уставе КПСС" на XXII съезде КПСС отмечалось:
"Ожесточенное сопротивление пыталась оказать осуществлению ленинского
курса, намеченного XX съездом партии, фракционная антипартийная группа,
в которую входили Молотов, Каганович, Маленков, Ворошилов, Булганин,
Первухин, Сабуров и примкнувший к ним Шепилов". Советский народ отметил
частое употребление клише "и примкнувшего к ним Шепилова"
соответствующим советскому народу образом. Водку, как правило,
соображали на троих, а если к распитию примыкал четвертый, то его
называли Шепиловым.

Источник:
   1. https://ru.wikipedia.org/wiki/Шепилов,_Дмитрий_Трофимович
   2. https://lurkmore.to/И_примкнувший_к_ним_Шепилов

Автор:
Александр Костров

Вопрос 19:
Недавние исследования показали, что ОНО действительно ТАКОЕ, причем не
только ЕЕ. Напишите название фильма 1969 года выпуска.

Ответ:
"Белое солнце пустыни".

Комментарий:
Желтым, оранжевым или еще каким-либо Солнце нам кажется из-за влияния
атмосферы. В пустыне, где небо чисто и безоблачно, Солнце кажется
особенно белым, т.е. практически таким, каким его видно из космоса.

Источник:
   1. https://www.popmech.ru/science/370142-kakogo-cveta-solnce/
   2. https://ru.wikipedia.org/wiki/Белое_солнце_пустыни

Автор:
Александр Костров

Вопрос 20:
В обеих порноверсиях "Игры престолов" ЕГО нет. В случае с самой молодой
матерью, Линой Мединой, ЕГО также не было, хотя у полиции возникли такие
подозрения. Назовите ЕГО словом латинского происхождения.

Ответ:
Инцест.

Комментарий:
Лина Ванесса Медина Васкес - самая молодая мать в медицинской истории,
родившая в возрасте 5 лет, 7 месяцев и 17 дней. Полиция изначально
подозревала отца в изнасиловании собственной дочери, но впоследствии его
отпустили ввиду отсутствия улик. В порноверсиях "Игры престолов" нет
сцен инцеста, потому как создатели опасаются общественного резонанса.
Впрочем, этими сценами вполне можно насладиться в оригинале.

Источник:
   1. Game of Bones: Winter Is Cumming, This Ain't Game of Thrones XXX
(Video 2014).
   2. https://ru.wikipedia.org/wiki/Медина,_Лина
   3. https://kp.ua/incidents/532807-stala-mamoi-v-5-let-y-perezhyla-syna-na-polveka

Автор:
Александр Костров

Вопрос 21:
Заметка об инновационной разработке сингапурских исследователей
называется "Дом поумнеет благодаря НОЖНИЦАМ". НОЖНИЦЫ часто упоминают в
связи с правильностью употребления двух других частей речи. Мы не просим
вас назвать две эти части речи. Ответьте, что мы заменили словом
"НОЖНИЦЫ".

Ответ:
Обои.

Комментарий:
Сингапурские исследователи собираются печатать микросхемы для "умного
дома" на подложке обоев. В связи с правильностью употребления
числительных "оба" и "обе" часто упоминаются и "обои".

Источник:
   1. https://vk.com/geekpicnic?w=wall-28587408_13931
   2. https://pishu-pravilno.livejournal.com/1580040.html

Автор:
Оксана Коробейникова

Вопрос 22:
Один американский издательский дом предлагал опубликовать этот
изначально англоязычный роман при условии, что заглавный персонаж будет
превращен в 12-летнего мальчика, а рассказчик - в фермера. В Европе же
это произведение было выпущено издательством, специализировавшимся, в
основном, на эротике. Назовите этот роман.

Ответ:
"Лолита".

Комментарий:
И тут, вы не поверите, товарищ следователь, Лолита превращается в
12-летнего мальчика...

Источник:
https://www.liveinternet.ru/users/meserer/post287818006/

Автор:
Александр Костров

Вопрос 23:
По мнению Стибора Остоя, Иван Грозный не преследовал геополитических
целей, когда брал Казань, - он жил в другом духовном континууме, где нет
интересов Московского Царства, но есть Небесная Царская Миссия, и ему
было необходимо пройти "живое" посвящение. Иоанн ассоциировал себя с
НИМ, и Казань стала его ИКСОМ. Назовите ЕГО и ИКС.

Ответ:
Иисус Навин, Иерихон.

Комментарий:
На протяжении всей жизни Иоанн ассоциировал себя с ветхозаветным Иисуиом
Навином, и ему был нужен свой Иерихон. Ему не составило труда найти его
по соседству.

Источник:
Журнал "История от Русской Семерки", ноябрь 2017 г.
http://ru.calameo.com/read/00511375604ab939985bf

Автор:
Александр Костров

Вопрос 24:
(pic: 20180300.jpg)
   Перед вами картина Эдварда Мунка "На 29 января". "На 29 января" -
постапокалиптический телефильм, вышедший раньше фильма-катастрофы 2004
года. Какие два слова мы заменили словами "29 января"?

Ответ:
Следующий день.

Зачет:
Завтрашний день.

Комментарий:
Фильм-катастрофа 2004 года - "Послезавтра", а "На следующий день" -
телефильм режиссера Николаса Мейера, рассказывающий о конфронтации между
НАТО и советским блоком, перешедшей в полномасштабную ядерную войну.

Источник:
   1. https://gallerix.ru/storeroom/1809742601/N/935375148/
   2. https://ru.wikipedia.org/wiki/На_следующий_день
   3. https://ru.wikipedia.org/wiki/Послезавтра

Автор:
Леонид Гольденберг

Тур:
14 тур. "Сборная села Тупино" (Ступино - Коломна - Воскресенск - Великий
Новгород)

Вопрос 1:
На рисунке одного из сетевых художников два литературных персонажа
изображены сидящими на мотоцикле - один за рулем, второй в коляске.
Назовите этих персонажей.

Ответ:
Дон Кихот, Санчо Панса.

Источник:
https://pikabu.ru/story/don_kikhot_5628296

Автор:
Виктор Плотников (Великий Новгород)

Вопрос 2:
Флешетта - это боеприпас, применявшийся в начале XX века. Флешетты
представляют собой металлические дротики, которые сбрасывали с самолетов
для поражения живой силы противника. Один интернет-пользователь,
обсуждая флешетты, упоминает ИХ. Назовите ИХ, использовав топоним.

Ответ:
Стимфалийские птицы.

Комментарий:
Стимфалийские птицы - хищные создания из древнегреческой мифологии, чьим
самым грозным оружием были медные перья, которые птицы сыпали на землю
как стрелы. Принцип действия очень похож на описанное применение
флешетт.

Источник:
   1. https://ru.wikipedia.org/wiki/Флешетта
   2. http://guns.allzip.org/topic/216/1426846.html
   3. https://ru.wikipedia.org/wiki/Стимфалийские_птицы
   4. https://ru.wikipedia.org/wiki/Стимфал_(город)

Автор:
Андрей Волков (Воскресенск)

Вопрос 3:
В компьютерной игре "Shadowrun: Hong Kong" [шЭдоу ран: гонкОнг] можно
встретить рассказ об огромной коллекции музыкальных альбомов. В этой
коллекции есть группы, играющие трэш-метал, хэви-метал, лайт-метал,
дэт-метал, нью-метал и даже группы с тройным ИМ в названии. Назовите
ЕГО.

Ответ:
Умляут.

Комментарий:
(pic: 20180301.jpg)
   Умляут - распространенное явление в названиях групп, играющих тяжелую
музыку. Вспомните, например, "Mot&ouml;rhead" и "M&ouml;tley Cr&uuml;e".

Источник:
Компьютерная игра "Shadowrun: Hong Kong".

Автор:
Василий Сумин (Егорьевск)

Вопрос 4:
Внимание, в вопросе нет замен, но будет раздатка к ответу.
   Надпись на одной из парт одного учебного заведения, изображение
которой мы раздадим вам в качестве ответа, не оставляет никаких
сомнений, что это ОНА. А кто изображен под этой надписью?

Ответ:
[Актер] [Джерард] Батлер [в гриме].

Зачет:
[Царь Спарты] Леонид [I].

Комментарий:
(pic: 20180302.jpg)
   Надеемся, для вас это не стало свояком за 300. :-)

Источник:
В ответе. :-)

Автор:
Олег Евстафьев (Ступино)

Вопрос 5:
Заголовок статьи в "Советском спорте" о победе сборной в Кубке
конфедераций отсылает к известному мультипликационному фильму. Какому?

Ответ:
"Король Лев".

Комментарий:
Статья была озаглавлена "Король - Лёв!", одним из ее главных героев
являлся тренер сборной Германии Йоахим Лёв.

Источник:
"Советский спорт. Итоги года. 2017". - С. 8.

Автор:
Борис Скоморохов (Ступино)

Вопрос 6:
Когда команда шашистов крайне неудачно начала турнир, ее тренер Андрей
Уголев высказал мнение, что ей стоило дать название футбольного клуба,
однажды победившего в Кубке УЕФА. А какое именно название?

Ответ:
"Шальке-04".

Комментарий:
В командных соревнованиях шашистов (как и шахматистов) матчи проходят на
четырех досках. Мнение было высказано, когда упомянутая команда
проиграла несколько матчей со счетом 0:4.

Источник:
   1. ЛОАВ.
   2. https://ru.wikipedia.org/wiki/Шальке_04

Автор:
Олег Евстафьев, Андрей Уголев (Ступино)

Вопрос 7:
Однажды, настаивая на своей версии в ходе игры в "Что? Где? Когда?",
автор вопроса в пылу обсуждения процитировал известного российского
спортсмена. Назовите этого спортсмена.

Ответ:
[Сергей] Овчинников.

Комментарий:
Речь, конечно, идет о высказывании про то, кто здесь является капитаном
команды.

Источник:
   1. ЛОАВ.
   2. https://ru.wikipedia.org/wiki/Овчинников,_Сергей_Иванович_(футболист)#Последний_матч

Автор:
Андрей Волков (Воскресенск)

Вопрос 8:
От Славы мы узнали, что ОНИ скачут по ИКСАМ. Та же фраза упоминается в
заголовке статьи еженедельника "Футбол" о том, что "Бешикташ" стал
главной неожиданностью Лиги чемпионов. Назовите ИХ и ИКС.

Ответ:
Турки, гроб.

Комментарий:
Осенью 2017 года "Бешикташ" показывал уверенную игру в чемпионате Турции
и еврокубках, чем, видимо, и было вызвано использование строчки из песни
Бутусова.

Источник:
   1. https://music.yandex.ru/album/211333/track/35386734
   2. https://www.ftbl.ru/issues/turki-skachut-po-grobam-kak-beshiktash-stal-glavnoj-neozhidannostyu-turnira

Автор:
Борис Скоморохов (Ступино)

Вопрос 9:
   <раздатка>
   тренер в фитнес-клубе
   потирал фингал
   леди совершенство
   зря он напевал
   </раздатка>
   Что мы заменили в этом четверостишии?

Ответ:
Тридцать три коровы.

Комментарий:
"Тридцать три коровы" и "Леди Совершенство" - песни из фильма "Мэри
Поппинс, до свидания".

Источник:
https://vk.com/depressyashki?w=wall-95488043_72896

Автор:
Олег Евстафьев (Ступино)

Вопрос 10:
(pic: 20180303.jpg)
   Перед вами кадры из фильма "Слуга государев". Название рецензии на
него одной буквой отличается от расхожей фразы, в которой упоминается
европейский город. Напишите это название.

Ответ:
"Увидеть парик и умереть".

Комментарий:
С париками костюмеры фильма перестарались. Если на французских дворянах
они еще выглядят уместно, то на лежащих на печи детях - не очень.

Источник:
   1. Х/ф "Слуга государев" (2007), реж. Олег Рясков.
   2. https://www.kommersant.ru/doc/745435
   3. http://www.aif.ru/dosug/1846440

Автор:
Андрей Волков (Воскресенск)

Вопрос 11:
   <раздатка>
   в рядок уселись партизаны
   звучит трофейная гармонь
   костер поляну освещает
   устало пляшут [...]
   </раздатка>
   Закончите стишок-пирожок по-русски или по-белорусски.

Ответ:
языки

Зачет:
языкi

Комментарий:
Слово "язык" одинаково на русском и белорусском как в значении
"пленный", так и в значении "подвижная часть чего-либо". В других
значениях это не всегда так. :-)

Источник:
   1. https://vk.com/perawki?w=wall-28122932_69266
   2. http://www.skarnik.by/rusbel/106850

Автор:
Олег Евстафьев (Ступино)

Вопрос 12:
   <раздатка>
   Султанбаев Махмуд из Киргизии
   Главным в армии был по провизии.
   Но готовил он так,
   Что блевали в кустах
   Даже танки Таманской дивизии.
   &nbsp;
   Молдаванин Иона Фитясовый
   Жил на острове в море Саргассовом.
   Маскируясь хитро,
   Он ловил комаров,
   И из бедных, из них, кровь высасывал.
   &nbsp;
   Старый зек, Отто Ав, из Эстонии
   Был зарезан в сибирской колонии.
   Говорят кореша,
   Умирал неспеша,
   Восемь месяцев корчась в агонии.
   </раздатка>
   ОН возник в апреле 1919 года на юго-западе Ирландии на фоне боев
между ИРА и английскими войсками и просуществовал совсем недолго. Серия
произведений, опубликованная на сайте stihi.ru и включающая среди других
те, что напечатаны на раздатке, была названа автором "ОНИ". Назовите ЕГО
или ИХ.

Ответ:
Советский Лимерик.

Зачет:
Советские лимерики.

Комментарий:
Во время войны за независимость Ирландии на территории графства Лимерик
ненадолго была провозглашена советская республика. Кроме того, лимерик -
стихотворный жанр, чье название происходит от графства. Автор на сайте
stihi.ru создал серию произведений в этом жанре, упоминая в каждом
какую-то советскую республику.

Источник:
   1. https://ru.wikipedia.org/wiki/Война_за_независимость_Ирландии
   2. http://www.stihi.ru/2007/04/13-402/

Автор:
Андрей Волков (Воскресенск)

Вопрос 13:
Героя одной карикатуры воспитали лососи, поэтому он пытается пройти
ПРОПУСК. Заполните пропуск названием романа 1965 года.

Ответ:
Вверх по лестнице, ведущей вниз.

Источник:
   1. https://condenaststore.com/featured/bob-was-raised-in-the-wilderness-by-salmon-drew-dernavich.html
   2. https://ru.wikipedia.org/wiki/Вверх_по_лестнице,_ведущей_вниз_(роман)

Автор:
Василий Сумин (Егорьевск)

Вопрос 14:
Тайский легковес Понгсаклек Вонджонгкам, долгое время владевший поясом
чемпиона мира по боксу, получил за свои достижения прозвище, совпадающее
с названием романа середины XX века. Назовите этот роман.

Ответ:
"Повелитель мух".

Комментарий:
Легковесов еще называют мухачами.

Источник:
   1. http://fightnews.ru/node/25845
   2. https://ru.wikipedia.org/wiki/Повелитель_мух

Автор:
Алексей Подлаткин (Воскресенск)

Вопрос 15:
Участвуя в одном летнем фестивале интеллектуальных игр, автор вопроса -
и, вероятно, не только он - последовал императиву, ставшему популярным
весной 2016 года. Напишите этот императив.

Ответ:
"В Питере - пить!".

Комментарий:
Один из известнейших фестивалей интеллектуальных игр, традиционно
проводящийся летом, - питерские "Белые ночи". Ну, а мы все прекрасно
знаем, чем знатоки занимаются на фестивалях. Песня "В Питере - пить!"
получила популярность в мае 2016 года с выходом клипа. Кстати, на два
предыдущих вопроса вы тоже давали ответ названиями произведений. :-)

Источник:
   1. http://rating.chgk.info/tournament/3855
   2. https://ru.wikipedia.org/wiki/В_Питере_%E2%80%94_пить

Автор:
Андрей Волков (Воскресенск)

Вопрос 16:
Заголовок интервью с российским хоккеистом Вычуровым, признанным
"Открытием года", носил название "Я добрый и не вычурный". Какое слово
мы заменили на "вычур"?

Ответ:
Каприз.

Комментарий:
"Открытием года" по версии газеты "Советский спорт" стал игрок ЦСКА
Кирилл Капризов. Синонимом слова "каприз" является слово "вычура".

Источник:
   1. "Советский спорт. Итоги года. 2017". - С. 15.
   2. https://dic.academic.ru/dic.nsf/dic_synonims/59772/

Автор:
Борис Скоморохов (Ступино)

Вопрос 17:
Согласно одной интернет-шутке, даже в НИХ ущемляются права черных.
Назовите ИХ.

Ответ:
Шахматы.

Зачет:
Шашки.

Источник:
https://www.anekdot.ru/id/628264/

Автор:
Алексей Подлаткин (Воскресенск)

Вопрос 18:
Узнав, о чем повествует фильм, который жюри премии "Оскар" признало
лучшим в 2017 году, автор вопроса предположил, что его главные герои
ОТТУДА. На самом деле фильм, главные герои которого ОТТУДА, вышел на
четверть века раньше. Какие три слова мы заменили словом "ОТТУДА"?

Ответ:
Из далекого космоса.

Зачет:
Из глубокого космоса.

Комментарий:
Главные герои фильма "Лунный свет", выигравшего "Оскар" в 2017 году, -
чернокожие гомосексуалисты. В 1992 году вышел фильм "Геи-ниггеры из
далекого космоса", персонажи которого имели много общего с героями
"Лунного света".

Источник:
   1. https://ru.wikipedia.org/wiki/Лунный_свет_(фильм,_2016)
   2. https://ru.wikipedia.org/wiki/Геи-ниггеры_из_далёкого_космоса

Автор:
Андрей Волков (Воскресенск)

Вопрос 19:
По мнению критика Джима Хобермана, в фильме "Порок на экспорт"
присутствует сильный гомоэротический подтекст, достигающий апогея в
сцене драки в бане. Комментируя этот эпизод, один интернет-пользователь
упоминает другой фильм. Назовите его заглавного героя.

Ответ:
Борат [Сагдиев].

Комментарий:
Как нетрудно догадаться, персонажи фильма "Порок на экспорт" дерутся в
бане в чем мать родила. Похожая сцена есть и в фильме о казахском
журналисте, познающем американскую культуру.

Источник:
   1. https://ru.wikipedia.org/wiki/Порок_на_экспорт
   2. https://www.youtube.com/watch?v=7cSP8u9N1Vg

Автор:
Андрей Волков (Воскресенск)

Вопрос 20:
Согласно цитате с Башорга, сюжет фильма "Форсаж" построен на том, что
все кому не лень ДЕЛАЮТ ЭТО. Какой эвфемизм мы заменили словами "ДЕЛАЮТ
ЭТО"?

Ответ:
Гоняют лысого.

Источник:
https://bash.im/quote/424482/

Автор:
Василий Сумин (Егорьевск)

Вопрос 21:
Собеседник автора вопроса как-то заметила: "Вот где нас таких честных и
бескорыстных брали! Что за альтруисты такие дефективные были?". Какие
буквы мы добавили в цитату?

Ответ:
льтру.

Комментарий:
No comments.

Источник:
ЛОАВ.

Автор:
Юлия Кощеева (Коломна), Борис Скоморохов (Ступино)

Вопрос 22:
Пользователь одного сайта, комментируя боксерский поединок, пишет: "Как
такой боец был допущен в бокс? Он же КРИВОРУКИЙ!". В ответ на это другой
пользователь шутит, что этого бойца сложнее просчитать. Какое слово мы
заменили в вопросе?

Ответ:
Косоглазый.

Комментарий:
Такого боксера сложнее просчитать, ведь непонятно, куда он смотрит.

Источник:
http://allboxing.ru/forum/index.php?topic=34536.12380

Автор:
Алексей Подлаткин, в редакции Андрея Волкова (оба - Воскресенск)

Вопрос 23:
   <раздатка>
   1.
   к новогодней ночи
   городская власть
   обещала очень город
   [...]
   &nbsp;
   2.
   гибедедешная бригада
   на днях приехала с [...]
   и принялась еще сильнее
   [...]
   </раздатка>
   Стихотворный дуплет.
   1. Закончите четверостишие одним словом из десяти букв.
   2. Какое слово из пяти букв было дважды пропущено в розданном
стишке-порошке?

Ответ:
   1. разукрасть
   2. гаить

Источник:
   1. https://vk.com/depressyashki?w=wall-95488043_85893
   2. https://vk.com/perawki?w=wall-28122932_68750

Автор:
Олег Евстафьев (Ступино)

Вопрос 24:
(pic: 20180304.jpg)
   Мы считаем, что информации, содержащейся на розданной картинке,
вполне достаточно, чтобы в ответном бланке написать в правильном порядке
то, что мы скрыли за звездочками.

Ответ:
T.HANKS, NO T.HANKS.

Источник:
https://imgur.com/i0T7Fwp

Автор:
Борис Скоморохов (Ступино)

Тур:
15 тур. "Ветряные мельницы вероятного противника" (Дубна - Москва)

Редактор:
Олеся Доронина

Инфо:
Тестеры пакета - Данила Лифшиц и Дмитрий Славин.

Вопрос 1:
В комиксе из серии "Polandball" [пОланд болл] юмористически описан ход
событий на одном из театров военных действий Тридцатилетней войны.
Испания марширует в поисках неприятеля, но, найдя его, решает, что
противник ей не по зубам, произносит некоторую фразу, в которой
упоминается число 1615, и бежит с поля битвы. А какое техническое
сооружение противника так напугало Испанию?

Ответ:
Ветряная мельница.

Комментарий:
Одним из ключевых эпизодов Тридцатилетней войны стала борьба
Объединенных Провинций Нидерландов за независимость от Испании. В 1615
году вышла книга Сервантеса "Дон Кихот", в которой главный герой
сражался с ветряными мельницами и был повержен. А обилие ветряных
мельниц в Голландии общеизвестно.

Источник:
https://www.reddit.com/r/polandball/comments/7ixnsm/the_dutch_deterrent/

Автор:
Давид Парунакян

Вопрос 2:
Эксклюзивный дистрибьютор компьютерной игры "Rocksmith", предназначенной
для обучения игре на гитаре или бас-гитаре, рекомендует пользователям
для облегчения обучения ставить определенный вид струн. Однако вряд ли
это поможет ИМ. Назовите фамилию человека, в честь кого ОНИ названы.

Ответ:
Дальтон.

Комментарий:
Струны DR Neon выпускаются в том числе в наборах, где каждая струна
определенного цвета. В игре "Rocksmith" для струн на экране используются
такие же цвета, что и в указанном наборе. А вот страдающим дальтонизмом
такой набор не сильно поможет: цвета некоторых струн будут казаться
одинаковыми.

Источник:
   1. http://rocksmith.ru/buy/133-struny-10-46-dr-nmce-10-neon-multicolor.html
   2. http://www.etre.com/tools/colourblindsimulator/
   3. https://ru.wikipedia.org/wiki/Дальтонизм

Автор:
Денис Завелев

Вопрос 3:
В современной России существенный процент ИКСОВ в какой-то момент своей
жизни становятся ИГРЕКАМИ. Любопытно, что цвет форменной одежды,
используемой другими ИГРЕКАМИ, совпадает с цветом одежды, которую в
совсем другом месте и в другое время носили другие ИКСЫ. Благодаря этому
цвету ИКСЫ и получили свое название. Какие два слова мы заменили ИКСОМ и
ИГРЕКОМ?

Ответ:
Кандидат и доктор.

Комментарий:
"Кандидат" в переводе с латыни - "обеленный", т.к. кандидаты на
государственную должность надевали обеленные тоги. Сейчас белые халаты,
в основном, носят доктора в больницах.

Источник:
https://ru.wikipedia.org/wiki/Тога

Автор:
Давид Парунакян

Вопрос 4:
Одно из произведений Велимира Хлебникова начинается так: "О Сад, Сад!
Где железо подобно отцу, напоминающему братьям, что они братья, и
останавливающему кровопролитную схватку". О каком саде идет речь?

Ответ:
О зоологическом.

Зачет:
По смыслу, включая слово "зверинец".

Источник:
В. Хлебников. Зверинец. https://rvb.ru/hlebnikov/tekst/02poemy/195.htm

Автор:
Ольга Степанянц

Вопрос 5:
Несмотря на то что еще в древности было доказано, что ОНО не существует,
если выбросить голову и подумать сердцем, следует признать, что по
крайней мере сейчас ОНО все-таки существует. Ответьте четырьмя словами:
о чем идет речь?

Ответ:
Самое большое простое число.

Комментарий:
Вопрос содержит подсказку ("Выброшу голову - пусть думает сердце!" -
название одного из альбомов одноименной группы).

Источник:
   1. https://ru.wikipedia.org/wiki/Теорема_Евклида
   2. https://ru.wikipedia.org/wiki/Самое_Большое_Простое_Число_(группа)

Автор:
Давид Парунакян

Вопрос 6:
Несмотря на то что на дворе XXI век, карточки социального страхования
США всё еще продолжают печататься на низкокачественной водоразлагаемой
бумаге. Поскольку номер социального страхования считается секретной
информацией, надпись на картах гласит, что их запрещено носить с собой и
ДЕЛАТЬ с ними ЭТО. Что именно еще запрещается делать с картами
социального страхования США?

Ответ:
Ламинировать.

Комментарий:
В числе прочих причин предполагается, что если гражданин всё же потерял
свою карту, то под воздействием влаги и прочих стихий она быстро
разложится, и записанный на ней номер не сможет быть использован
злоумышленниками, в то время как ламинация карты помешает этому.

Источник:
https://techcrunch.com/2009/07/07/lets-guess-each-others-social-security-numbers-for-fun/

Автор:
Давид Парунакян

Вопрос 7:
Согласно цитате с сайта bash.im, "кто учил, тот согласится, что сопромат
- практически непередаваемые ощущения". В этой цитате мы пропустили одну
букву. Какую?

Ответ:
С.

Комментарий:
Непересдаваемые.

Источник:
https://bash.im/quote/442949/

Автор:
Денис Завелев

Вопрос 8:
Блиц.
   В Эстонском национальном музее (в городе Тарту) есть зал, в котором
экспонируются иллюстрации, изображающие ряд важных, по мнению авторов,
событий в истории Эстонии и мира.
   (pic: 20180305.jpg)
   1. Первая картинка - из мировой истории. Какую надпись мы на ней
закрыли?
   (pic: 20180306.jpg)
   2. Вторая картинка - также из мировой истории. Чье изображение мы
закрыли?
   (pic: 20180307.jpg)
   3. Третья картинка - из истории Эстонии, мы привели только ее
фрагмент. Воспроизведите фамилию, от которой произошло эстонское слово,
являющееся подписью к данной картинке.

Ответ:
   1. Coca-Cola.
   2. Наполеон Бонапарт.
   3. Хрущёв.

Комментарий:
   2. Именно его военные походы способствовали массовому распространению
консервов.
   3. Слово - "hru&scaron;t&scaron;ovka".

Источник:
Фотографии, сделанные автором вопроса в музее.

Автор:
Денис Завелев

Вопрос 9:
В начале XX века Банк Англии немало поспособствовал развитию физики в
Великобритании. Однако дело было не в дополнительном финансировании:
британские лаборатории приобретали у банка ЭТО. Персонаж художественного
произведения второй половины XIX века предлагал своим собеседницам
обсудить ЭТО наравне с вопросами геополитики, огородничества, а также,
по всей видимости, современной моды и глобального потепления. Назовите
ЭТО словом из шести букв.

Ответ:
Сургуч.

Комментарий:
Многие физические лаборатории, экспериментировавшие с микроскопическими
количествами радиоактивных веществ, нуждались в удобном материале для
герметизации колб. Таким материалом долгое время служил сургуч. О
сургуче же предлагал поговорить Морж из стихотворения Льюиса Кэрролла в
произведении "Алиса в Зазеркалье".

Источник:
   1. http://www.jameshedberg.com/lettera/2015/02/01/the-craft-of-experimental-physics/
   3. Л. Кэрролл. Алиса в Зазеркалье. http://flibusta.is/b/392565/read

Автор:
Давид Парунакян

Вопрос 10:
На карикатуре художника Сергея Ёлкина, подписанной "Ёлкин Svoboda",
можно увидеть две даты - 1987 и 2017, два однотипных транспортных
средства и имена двух людей. При этом с более новым транспортным
средством (а точнее, со множеством таких объектов) и человеком,
указанным рядом, связана не отличающаяся благородством история. Имена
каких людей упомянуты на карикатуре?

Ответ:
Матиас Руст, Павел Дуров.

Комментарий:
Одномоторный самолетик Матиаса Руста приземлился на Красной площади,
преодолев все заслоны советской ПВО. Бумажный самолетик - символ
мессенджера Телеграм, который в последнее время приобрел репутацию
надежного анонимного инструмента общения, несмотря на угрозы блокировки
от Роскомнадзора и ФСБ. В 2012 году Павел Дуров запускал бумажные
самолетики из пятитысячных купюр из окна своего офиса, чем вызвал драку
на улице и негодование в СМИ.

Источник:
   1. https://vk.com/photo-130486220_456240048
   2. https://lenta.ru/articles/2012/05/29/durov/

Автор:
Давид Парунакян

Вопрос 11:
(pic: 20180308.jpg)
   Перед вами изображение продающегося в Японии набора некоторых
изделий. Большинству пользователей привычна более простая, чем в данном
наборе, форма этого изделия. Одни из первых подобных изделий
производились по заказу Королевских военно-воздушных сил Великобритании.
А в СССР какое-то время этим изделием запрещали пользоваться ученикам
младших классов. Ответьте максимально точно: о каком изделии идет речь?

Ответ:
Шариковая ручка.

Источник:
   1. https://vk.com/zenmarket?w=wall-54630614_361
   2. https://ru.wikipedia.org/wiki/Шариковая_ручка

Автор:
Денис Завелев

Вопрос 12:
В своем письме к Алексею Сергеевичу Суворину Антон Павлович Чехов писал:
"Хорошо поют эти дикие бестии. Их пение похоже на крушение поезда с
высокой насыпи во время сильной метели: много вихря, визга и стука...".
О чьем же пении так писал классик?

Ответ:
Цыганок.

Зачет:
Цыган.

Источник:
http://www.feb-web.ru/feb/chekhov/texts/sp0/pi3/pi3-1682.htm

Автор:
Денис Завелев

Вопрос 13:
[Ведущему: сказать: "Этот текст у вас в раздатках", зачитать текст
раздатки, после чего уже сказать: "Внимание, вопрос!"]
   <раздатка>
   В этом коротеньком тупиковом проезде находится учреждение, занимающее
старое двухэтажное строение, затертое соседними домами. Прежде чем
добраться до его дверей, нужно завернуть не за один угол, причем всё
время налево. Такими запутанными и замысловатыми раньше были пути только
в нелегальные дома терпимости или на явочные квартиры. Но этим
неприметным и внешне убогим домом раньше или позже интересуется едва ли
не половина жителей столицы.
   ...
   Что здесь делается по субботам в осенне-зимний сезон, если не очень
холодно! Небольшой двор забивается народом - протиснуться невозможно.
Наверное, знаменитая Сухаревка не знала такой плотности человеческих душ
на один квадратный метр территории. То и дело попадаются "пикетчики". А
некоторые не изготавливают рекламные плакаты, а их отсутствие заменяют
голосом, выкрикивая время от времени свое предложение. Люди хватают друг
друга за рукава, собираются группами, исписывают адресами и телефонами
свои блокноты. Занимательнейшее зрелище! Здесь вечно много толчётся
пенсионеров, у которых время в избытке и которым дома скучно.
   &nbsp;
   [Из раздела "С утра до вечера 18.06.71 глазами просто москвича" книги
Алексея Булгакова "XX век, Москва и мы".]
   </раздатка>
   Чем занималась организация, которой посвящен данный текст?

Ответ:
Обменом жилплощади.

Зачет:
Обменом квартир (комнат) и т.п. Незачет: Пропиской, куплей-продажей
жилья и т.д.

Комментарий:
В СССР не было вторичного рынка жилья, а маклеры были в тени, поэтому
желающим обменять свое жилье москвичам приходилось искать счастья в
Городском бюро обмена жилплощади.

Источник:
А.А. Булгаков. XX век, Москва и мы. - Ярославль, 2017.

Автор:
Денис Завелев

Вопрос 14:
Зайдя в трамвай и заняв сидячее место, автор вопроса заметил, что он
сидит на ВИРТУАЛЬНОЙ СРЕДЕ. Что мы заменили в предыдущем предложении?

Ответ:
Песочнице.

Комментарий:
Под отдельными сиденьями в ряде вагонов установлены ящики с песком -
песочницы. Песочницей также называют виртуальную среду для безопасного
исполнения компьютерных программ.

Источник:
   1. http://zinref.ru/000_uchebniki/05302_trollebus/010_00_tramvainie_vagon_t3_ivanov_1977/016.htm
   2. http://helpiks.org/6-30868.html
   3. https://ru.wikipedia.org/wiki/Песочница_(безопасность)

Автор:
Денис Завелев

Вопрос 15:
Что бы вы сделали, если бы были человеком определенной профессии в
дореволюционной Москве и вам надо было округлить клиента?

Ответ:
Постригли в кружок.

Зачет:
Постригли "под горшок".

Комментарий:
Речь идет о профессии парикмахера (цирюльника).

Источник:
В.С. Елистратов. Язык старой Москвы: Лингвоэнциклопедический словарь. -
2-е изд. - М.: ООО "Издательство АСТ", 2004.

Автор:
Денис Завелев

Вопрос 16:
На картинке изображены коты породы сфинкс. Один из них с удивлением
смотрит на обычного кота и спрашивает другого сфинкса:
   - Коллега, это кот?
   - Да, коллега, это кот! Только в ПРОПУСКЕ.
   Многие из вас приехали на марафон с ПРОПУСКОМ. Что мы заменили словом
"ПРОПУСК"?

Ответ:
Спальный мешок.

Зачет:
Спальник.

Источник:
   1. http://kotomatrix.ru/show/1262298/
   2. ЛОАВ.

Автор:
Ольга Степанянц

Вопрос 17:
При переводе некоторого комикса с английского языка на русский
упоминающаяся в нем фамилия прозаика была заменена фамилией поэта с
целью сохранения игры слов с ругательным оттенком корней. При этом
претерпела изменения не только фамилия, но и гендерная идентичность
корня. Напишите обе фамилии.

Ответ:
Диккенс, Мандельштам.

Источник:
https://bash.im/quote/437378/

Автор:
Давид Парунакян

Вопрос 18:
Первые в мире школы назывались в Месопотамии ИКСАМИ АЛЬФ. А
постановщикам соревнований по городскому ориентированию "Бегущий город"
часто приходится фотографировать АЛЬФЫ на ИКСАХ. Что мы заменили ИКСАМИ
и АЛЬФАМИ?

Ответ:
Дома, таблички.

Источник:
   1. Л. Млодинов. Прямоходящие мыслители. Путь человека от обитания на
деревьях до постижения мироустройства. http://flibusta.is/b/438947/read
   2. ЛОАВ.

Автор:
Ольга Степанянц, Денис Завелев

Вопрос 19:
В одной из статей Википедии можно прочитать, что расширение московских
улиц (в том числе прокладка Садового кольца) произошло благодаря Красной
площади. Что мы заменили в предыдущем предложении?

Ответ:
Пожару.

Комментарий:
Красную площадь раньше также называли Пожар. А статья посвящена пожару
Москвы 1812 года.

Источник:
   1. https://ru.wikipedia.org/wiki/Московский_пожар_(1812)
   2. https://ru.wikipedia.org/wiki/Красная_площадь

Автор:
Денис Завелев

Вопрос 20:
В эпилоге книги "Империя должна умереть" журналист Михаил Зыгарь
называет АЛЬФУ ИКСОМ истории. На сайтах об автомобилях можно найти коды
АЛЬФ ИКСОВ. Что мы заменили АЛЬФОЙ и ИКСОМ?

Ответ:
Ошибка, двигатель.

Источник:
   1. М.В. Зыгарь. Империя должна умереть: История русских революций в
лицах. 1900-1917. http://flibusta.is/b/521912/read
   2. http://www.1gai.ru/baza-znaniy/poleznoye/518036-oshibka-dvigatelya-diagnosticheskie-kody-neispravnosti.html

Автор:
Денис Завелев

Вопрос 21:
На одной из картин художника-любителя АДОЛЬФА ГИТЛЕРА изображен вулкан.
Примечательно, что клубы дыма, курящиеся над вершиной горы, художник
изобразил с натуры. Мы не просим сказать, какие имя и фамилию мы
заменили АДОЛЬФОМ ГИТЛЕРОМ. Назовите объект, с которого был срисован дым
вулкана.

Ответ:
Сигара.

Комментарий:
Уинстон Черчилль, чье имя было заменено, был художником-любителем,
изобразившим, в том числе, Везувий. А его любовь к сигарам в целом
общеизвестна.

Источник:
https://avderin.livejournal.com/404853.html

Автор:
Давид Парунакян

Вопрос 22:
Некто задается вопросом: "Интересно, а Льва Николаевича в детстве
дразнили фразой "Не произноси ложного свидетельства на ближнего
твоего"?". В вопросе мы заменили одно устойчивое выражение.
Воспроизведите это выражение.

Ответ:
"Почитай отца твоего и мать твою".

Зачет:
По смыслу.

Комментарий:
Речь про Льва Николаевича Гумилёва, сына Николая Гумилёва и Анны
Ахматовой.

Источник:
   1. https://bash.im/quote/443110/
   2. https://ru.wikipedia.org/wiki/Десять_заповедей

Автор:
Денис Завелев

Вопрос 23:
Поиски свидетельств ЕГО существования затянулись на несколько
десятилетий, хотя непосвященному человеку задача могла бы показаться
элементарной. Окончательно ЕГО существование было установлено лишь в
2016 году благодаря объединенным усилиям молекулярных биологов и
палеоантропологов. Напишите название, которым ЕГО окрестил журнал
"Science", максимально точно.

Ответ:
Higgs Bison.

Зачет:
Бизон Хиггса.

Комментарий:
"Бизон Хиггса" - игра слов на "Бозон Хиггса", поиски свидетельств
которого также затянулись на несколько десятилетий. А слово
"элементарно" являлось подсказкой.

Источник:
https://www.npr.org/sections/thetwo-way/2016/10/18/498281083/higgs-bison-is-the-missing-link-in-european-bison-ancestral-tree

Автор:
Давид Парунакян

Вопрос 24:
Сайт "Энциклопедия нашего детства", позиционирующий себя проектом для
всех выросших в СССР, называет некий бытовой агрегат практически
символом своего времени. Умение собирать и разбирать этот агрегат
ценилось едва ли не больше, чем умение собирать и разбирать автомат
Калашникова. О каком агрегате идет речь?

Ответ:
Мясорубка.

Источник:
   1. http://e-n-d.ru/tech_home/284.html
   2. http://e-n-d.ru/project/

Автор:
Денис Завелев

Тур:
16 тур. "Не для школы, а для жизни" (Москва)

Вопрос 1:
Увидев результат ЕГО работы, ЕГО жена воскликнула: "О боже, я вижу свою
смерть". Назовите ЕГО.

Ответ:
[Вильгельм Конрад] Рентген.

Комментарий:
Анна Берта Людвиг увидела скелет своей руки на рентгеновском снимке.

Источник:
Телесериал "Гений", s01e03.

Автор:
Мария Чамаева

Вопрос 2:
Согласно шутке друга автора вопроса, чтобы получить новые купюры в две
тысячи рублей, нужно войти в банк ТАК. Ответьте четырьмя словами: КАК?

Ответ:
С гранатою в кармане.

Зачет:
С чекою в руке; с гранатой в кармане; с чекой в руке.

Комментарий:
Владивосток-2000 и всё такое.

Источник:
Блистательная шутка Артема Красина.

Автор:
Мария Чамаева

Вопрос 3:
Героиня одного произведения заключает, что ее жизнь - это ОНА: вроде бы
стыдно, но после вина кажется прекрасной. ОНИ удостоились в общей
сложности 13 "Золотых граммофонов". Назовите ИХ тремя словами.

Ответ:
Песни Валерия Меладзе.

Источник:
   1. https://genius.com/Komsomolsk-meladze-lyrics
   2. https://ru.wikipedia.org/wiki/Золотой_граммофон

Автор:
Егор Фёдоров

Вопрос 4:
Алан Ломакс пишет, что в конце XIX века в Техас пришла гудящая,
пыхтящая, задыхающаяся железная дорога, и многие темнокожие разнорабочие
получили возможность путешествовать и обмениваться опытом. По мнению
Ломакса, так появилось ЭТО. Назовите ЭТО, используя дефис.

Ответ:
Буги-вуги.

Комментарий:
Железная дорога, соединившая Техас с другими южными штатами, как
позволила музыкантам путешествовать и распространять свое творчество,
так и вдохновила на новые ритмические решения и навсегда изменила игру
на пианино. Так появился новый жанр - буги-вуги.

Источник:
Alan Lomax, The Land Where the Blues Began.

Автор:
Егор Фёдоров

Вопрос 5:
В образовательной передаче, вышедшей в январе 2018 года, корреспондент
рассказывает о валуне, напоминающем гигантский кусок сыра. Дело в том,
что он посетил завод по производству... Ответьте абсолютно точно: чего?

Ответ:
Камней для кёрлинга.

Зачет:
По смыслу.

Комментарий:
Серия передач была посвящена грядущим Олимпийским Играм и зимним видам
спорта. Заготовки для камней для кёрлинга аккуратно вырезаются из
многотонного цельного куска гранита, после чего он остается испещрен
идеальными сферическими отверстиями.

Источник:
"Все на Матч", утренний выпуск от 16.01.2018 г.

Автор:
Егор Фёдоров

Вопрос 6:
В книге, вышедшей в конце 2003 года, вдова этого человека пишет: "... он
был замечательным мужем...". Назовите этого человека.

Ответ:
Ричард "Рик" Хасбенд.

Комментарий:
Такой вот каламбур. Впрочем, вдова командира экипажа шаттла "Columbia"
Эвелин Хасбенд отзывается так не о своем покойном муже, а о его коллеге
и друге их семьи Майкле Андерсоне, погибшем в той же катастрофе.

Источник:
Evelyn Husband, High Calling. The Courageous Life and Faith of Space
Shuttle Columbia Commander Rick Husband.

Автор:
Георгий Федосов

Вопрос 7:
   <раздатка>
   ЭТО У МЕНЯ, СЕРЁГА, ФАКС ПОШЕЛ.
   И В ЛЮСЬКИНЫХ ТАПОЧКАХ!
   А ВЕДЬ ПРАВИЛЬНО ПЕТРОВ ГОВОРИТ: НЕУДОБНО РАБОТАТЬ ЛОПАТАМИ С
КОРОТКИМИ ЧЕРЕНКАМИ!
   </раздатка>
   Перед вами несколько ИХ. При поиске картинок по запросу "ОН" Google в
числе первых результатов выдает фотографию человека, родившегося в
Ленинграде в 1962 году. Назовите этого человека.

Ответ:
Николай Бандурин.

Комментарий:
Согласно популярной шутке Дениса Чужого, Михаил Вашуков - это сетап, а
Николай Бандурин - панчлайн, так как Вашуков пел начало юмористического
куплета, а Бандурин - его окончание. При поиске в Google по запросу
"панчлайн" можно наткнуться на обрезанную версию только с Бандуриным:
   (pic: 20180309.jpg)
   В раздаточном материале перечислены панчлайны: "Это у меня, серёга,
факс пошел" - из анекдота о новых русских, которые вживляли в себя
современную (по меркам девяностых) технику; "А ведь правильно петров
говорит..." - из анекдота о том, как сложен и многообразен тюремный
жаргон; "И в люськиных тапочках" - из анекдота о мужчине, который не вел
половую жизнь даже несмотря на то, что был женат, и однажды повстречал
одноглазого бородатого гнома.

Источник:
   1. https://www.google.com/search?q=панчлайн&tbm=isch
   2. https://ru.wikipedia.org/wiki/Бандурин,_Николай_Юрьевич

Автор:
Георгий Федосов

Вопрос 8:
В результате небесного погребения от человека остаются только кости.
Другое название небесного погребения - это "раздача милостыни...". Кому?

Ответ:
Стервятникам.

Зачет:
Падальщикам.

Комментарий:
Тело умершего в некоторых регионах (например, в Тибете) скармливают
стервятникам. Иногда и костей не остается, их перемалывают и тоже
скармливают птицам.

Источник:
https://masterok.livejournal.com/1533064.html

Автор:
Мария Чамаева

Вопрос 9:
Иронизируя над избытком микрокредитных организаций в своем городе,
микроблогер Алексей Мельников говорит, что ОНИ - это отсутствие первого
взноса и низкая процентная ставка. А каких ИХ выделял известный уроженец
Санкт-Петербурга?

Ответ:
Армия [России], флот [России].

Источник:
   1. https://twitter.com/leshamelnikov/status/932259152767963136/
   2. https://dic.academic.ru/dic.nsf/dic_wingwords/2782/
   3. https://ru.wikipedia.org/wiki/Александр_III

Автор:
Георгий Федосов

Вопрос 10:
Этот человек готовится к 55-летнему юбилею, который состоится 9 июня
этого года. Ура! Ура! Он был номинирован на "Оскар" три раза, три. Ура!
Ура! Назовите его.

Ответ:
Джонни Депп.

Комментарий:
Текст вопроса отсылает к тексту американской патриотической песни "When
Johnny comes marching home":
   Get ready for the Jubilee,
   Hurrah! Hurrah!
   We'll give the hero three times three,
   Hurrah! Hurrah!

Источник:
   1. https://en.wikipedia.org/wiki/Johnny_Depp
   2. https://en.wikipedia.org/wiki/When_Johnny_Comes_Marching_Home

Автор:
Георгий Федосов

Вопрос 11:
В американском хоррор-телесериале, повторяющем сюжет одноименного
фильма, можно услышать песню "Por la noche". Как называется этот сериал?

Ответ:
"От заката до рассвета".

Комментарий:
"Por la noche" - испаноязычная версия песни "After dark" из
оригинального фильма.

Источник:
Телесериал "От заката до рассвета".

Автор:
Иннокентий Хумонен

Вопрос 12:
Согласно шутке, в этой сказке всё правда: если смотришь сквозь стекло
бутылки, то сразу находишь у себя положительные качества, но с дорогой
домой могут возникнуть проблемы. Назовите эту сказку.

Ответ:
"Волшебник Изумрудного города".

Зачет:
"Удивительный волшебник из Страны Оз".

Комментарий:
В сказке про Изумрудный город всё правда: если смотреть сквозь зеленое
дно бутылки, то сразу находишь у себя и смелость, и сердце, и мозги (а
вот с дорогой домой возникают проблемы). Также можно предположить, что
Оз дает Трусливому Льву алкоголь. Страшила получает мозги из отрубей,
иголок и булавок (игра слов: bran - отруби, pins - булавки, brains -
мозги), Лев - шипучий напиток, делающий его храбрее благодаря эффекту
плацебо (согласно некоторым намекам, можно предположить, что Оз угощает
Льва алкоголем), а Жестяной Дровосек - сердце из шелка, набитого
опилками.

Источник:
   1. https://twitter.com/sh_fitzwalter/status/954054727095996416/
   2. https://ru.wikipedia.org/wiki/Удивительный_волшебник_из_страны_Оз

Автор:
Иннокентий Хумонен

Вопрос 13:
(pic: 20180310.jpg)
   В статье о НЕЙ приводится пример эхолокации летучих мышей, принцип
которой понятен многим людям, но никому из них не доступен для
перцепции. Можно сказать, что другая ОНА перед вами. Назовите ЕЕ двумя
словами.

Ответ:
Комната Марии.

Комментарий:
На фото лаборатория Марии Кюри. "Комната Марии" - мысленный эксперимент,
призванный показать важность субъективного опыта. В нем Мария изучает
цвет, сидя в полностью черно-белой комнате. Несмотря на исчерпывающие
знания обо всём, что связано с цветом, - от длин волн до воздействия на
сетчатку и мозг человека, - она его никогда не видела. Та же проблема
недостатка личного опыта поднимается и в статье Томаса Нагеля "Каково
быть летучей мышью?".

Источник:
https://ru.wikipedia.org/wiki/Комната_Марии

Автор:
Алина Антипова

Вопрос 14:
Слова "ШАРЛОТТА КОРДЕ" и "ЖАН-ПОЛЬ МАРАТ" - замены.
   В одном сериале ШАРЛОТТА КОРДЕ, галлюцинируя, беседует с поедающим
гамбургер ЖАНОМ-ПОЛЕМ МАРАТОМ. ШАРЛОТТА КОРДЕ заявляет, что мужчины
подавляют женщин в искусстве из страха перед их превосходством. Даже
произнося имя "ШАРЛОТТА КОРДЕ", люди думают не о ее трудах, а о
ЖАНЕ-ПОЛЕ МАРАТЕ. Восстановите обе замены.

Ответ:
Валери Соланас, Энди Уорхол.

Комментарий:
3 июня 1968 года Валери Соланас - радикальная феминистка и писательница
- несколько раз выстрелила в Уорхола. Незадолго до этого Уорхол,
обещавший просмотреть написанный ею сценарий, заявил, что потерял его,
заплатив компенсацию всего в 25 долларов. А вместо творческого союза он
предоставил ей работу машинистки. Поныне большинство людей помнит ее как
девушку, стрелявшую в Уорхола, а не как автора. В одном из самых
знаменитых роликов Энди Уорхол поедает гамбургер.

Источник:
   1. https://www.youtube.com/watch?v=VelPrql-MUo
   2. https://www.youtube.com/watch?v=Ejr9KBQzQPM

Автор:
Алина Антипова

Вопрос 15:
В Хэллоуинском эпизоде "Симпсонов" недалекий и не очень богатый Гомер,
встретив мудрого американского политика, называет его ЭТИМ ИМЕНЕМ, путая
с персонажем произведения XX века. Напишите ЭТО ИМЯ.

Ответ:
Пеннивайз.

Комментарий:
Гомер встретил Линкольна, изображенного на так называемой пенни, и
назвал того Пеннивайзом.

Источник:
Мультсериал "Симпсоны", эпизод "Treehouse of horror".

Автор:
Алина Антипова

Вопрос 16:
Комментируя одну пикантную фанатскую теорию, Кэрри Фишер сообщила, что
вряд ли стоит полагаться на ее мнение. Ведь ее БИНОКЛЬ не так уж хорош,
о чем свидетельствует развод с мужем. Какое слово было заменено в
предыдущем предложении?

Ответ:
Гейдар.

Комментарий:
Фанаты вопрошали, возможны ли отношения между бывшим штормтрупером и
пилотом из перезапуска "Звездных войн". Однако актриса, муж которой ушел
к мужчине, считает, что не умеет определять на глаз сексуальные
меньшинства.

Источник:
https://www.reddit.com/r/funny/comments/6kl3fx/carrie_fisher_asked_about_finn_and_poe/

Автор:
Алина Антипова

Вопрос 17:
ОНА на картине русского художника XIX века благосклонна даже к
немолодому и горбатому мужчине. Интересно, что примером задачи о НЕЙ
является история привередливого и нерешительного Иоганна Кеплера,
который после печального события переписывался с 11 кандидатками.
Назовите ЕЕ двумя словами.

Ответ:
Разборчивая невеста.

Комментарий:
Павел Федотов прославился сатирической жанровой живописью на свадебную
тематику. "Задача о разборчивой невесте" - задача поиска лучшего
кандидата из имеющихся путем наименьшего перебора. Предложенная гораздо
позже формула сильно бы помогла Кеплеру, который пришел к выводу, что
лучшей была уже успевшая выйти замуж за время его поисков кандидатка N
5.

Источник:
   1. http://opisanie-kartin.com/opisanie-kartiny-pavla-fedotova-razborchivaya-nevesta/
   2. А. Беллос. Красота в квадрате. Как цифры отражают жизнь и жизнь
отражает цифры. http://flibusta.is/b/512007/read

Автор:
Алина Антипова

Вопрос 18:
Внимание, в вопросе есть замена.
   В научно-познавательном ролике Мэнни упоминает свою знакомую,
символом симпатии которой может служить ее фото с узором КОВРА
СЕРПИНСКОГО на груди. Недавно в Тюмени прошел перформанс "Вязание на
спицах и крючком КОВРА СЕРПИНСКОГО", иллюстрирующий пример бесконечной
работы. Восстановите замененные в вопросе слова.

Ответ:
Множество Мандельброта.

Комментарий:
Полностью детектива зовут Мэнни Брот. Множество Мандельброта содержит
кардиоиду, выглядящую как сердечко. Вязание фрактала, который
бесконечен, - довольно бесполезная работа.

Источник:
   1. https://www.youtube.com/watch?v=0C75vRVL5lE
   2. https://t-i.ru/article/new/10959
   3. https://ru.wikipedia.org/wiki/Множество_Мандельброта

Автор:
Алина Антипова

Вопрос 19:
Дуплет.
   1. "Она сказала: "Это как сказка, только для взрослых". <...> Все
любят пиратов. А мне нравилась идея о летающем корабле, и я решил
включить эпизод с ним для иллюстратора". Можно сказать, что только что
вы услышали ИХ Нила. Назовите ИХ четырьмя словами.
   2. Возможно, в прошлом году ОНА заключалась в том, что академики были
более очарованы темой взросления и поиска себя. Назовите ЕЕ тремя
словами.

Ответ:
   1. Воспоминания о "Звездной пыли".
   2. Магия "Лунного света".

Комментарий:
В первой части речь идет о "Звездной пыли" Нила Геймана, а во второй -
об успехе фильма "Лунный свет". В обоих случаях зашифрованы названия
фильмов Вуди Аллена.

Источник:
   1. http://www.mtv.com/news/1566862/stardust-author-neil-gaiman-tells-why-he-turns-down-most-adaptations-but-not-this-one/
   2. https://en.wikipedia.org/wiki/Stardust_(novel)
   3. https://en.wikipedia.org/wiki/Moonlight_(2016_film)
   4. https://ru.wikipedia.org/wiki/Фильмография_Вуди_Аллена

Автор:
Алина Антипова

Вопрос 20:
Это произведение искусства заказал Седьмой, однако оно было завершено
после его смерти при Третьем. По мнению британского искусствоведа
Кеннета Кларка, Седьмой хотел таким образом проиллюстрировать
разграбление Рима 1527 года, а Третий - судьбу еретиков и схизматиков.
Назовите это произведение.

Ответ:
"Страшный суд".

Комментарий:
Изначально роспись стены за алтарем Сикстинской капеллы заказал папа
Климент VII, но фреска была нарисована лишь после его смерти при Павле
III, который видел в ней уже другой смысл.

Источник:
   1. Документальный цикл Би-Би-Си "Цивилизация", 7-я серия "Величие и
смирение".
   2. https://ru.wikipedia.org/wiki/Страшный_суд_(Микеланджело)

Автор:
Иннокентий Хумонен

Вопрос 21:
В издании "Calvert Journal" недавно была опубликована статья. В ней
автор размышляет о судьбе так называемых "афрорусских" - людей,
родившихся от браков между русскими и темнокожими, приехавшими из Африки
и Латинской Америки. Заголовок статьи представляет собой название
известной англоязычной композиции из четырех слов, к которому добавлена
одна буква. Воспроизведите это название.

Ответ:
"Black in the USSR".

Источник:
https://www.calvertjournal.com/features/show/5388/red-africa-afrorussians-black-ussr-portraits-generation-identity

Автор:
???

Вопрос 22:
Согласно шутке Павла Столярова, сомнительному суши-бару подошло бы
название "СОЛИТЕРЕМОК". Какое десятибуквенное слово, оканчивающееся на
гласную, мы заменили в этом вопросе?

Ответ:
Нематодасё.

Источник:
https://twitter.com/_wren_1/status/953703403183591424/

Автор:
Георгий Федосов, Иннокентий Хумонен

Вопрос 23:
В песне Бориса Циммермана о городе Электросталь есть следующие строки:
"И руками ковали свой рыцарский щит, // И ходили цехами, где панцирь
трещит". В приведенной цитате мы заменили в том числе имя собственное.
Назовите это имя собственное.

Ответ:
Гейгер.

Комментарий:
И не рыцарский, а ядерный - в подмосковной Электростали производят,
помимо прочего, составляющие для ядерных реакторов.

Источник:
Борис Циммерман - Электросталь.
http://tekstovoi.ru/text/930520652_m95666765p585616003_text_pesni_jelektrostal.html

Автор:
Георгий Федосов

Вопрос 24:
Герой одного детективного рассказа нашел предназначавшееся не ему
любовное письмо. Если бы он соединил ИХ, то получил бы план укреплений
Портсмута. Назовите ИХ устойчивым словосочетанием.

Ответ:
Точки над i.

Источник:
А. Кристи. Девушка в поезде. http://flibusta.is/b/159028/read

Автор:
Софья Никифорова

Тур:
17 тур. "Все хотят" (Москва - Зеленоград)

Инфо:
Команда благодарит за помощь в подготовке пакета: Данила Куликова,
Елизавету Матвееву, Алексея Овчинникова, Юлию Чудакову, Шамиля Кадырова,
Егора Мухина, Викторию Полякову, Евгения Сельменёва, Бориса Абрамкина,
Анастасию Милютину, Владислава Клеймёнова, Екатерину Тран и Олесю
Доронину.

Вопрос 1:
По мнению Дмитрия Ивановича Писарева, ОНА гораздо лучше стихотворения,
поскольку читатель не тратит на нее времени, а редакция - денег.
Вероятно, каждый из нас тоже был бы ЕЙ очень рад. Назовите ЕЕ двумя
словами, начинающимися на парные согласные.

Ответ:
Белая полоса.

Комментарий:
Публицисту не нравилась привычка журналов заполнять стихотворениями
полосы, которые оставались пустыми, и связанный с этим неконтролируемый
рост числа поэтов, которых он считал тунеядцами. Пусть и в вашей жизни
будет больше белых полос!

Источник:
Д.И. Писарев. Реалисты. http://az.lib.ru/p/pisarew_d/text_0350.shtml

Автор:
Татьяна Хадыева

Вопрос 2:
Однажды ведущий "Versus battle" [вЕрсус баттл] Александр Тимарцев в
шутку назвал одного из участников дубом за то, что тот выступал,
практически не выучив текст. Какое слово с удвоенной согласной мы
заменили в предыдущем предложении?

Ответ:
Лиственница.

Комментарий:
Словом "дуб" мы заменили слово "лиственница". Все три своих раунда рэпер
Хип-Хоп Одинокой Старухи почти целиком прочитал по бумаге. Из-за обилия
листов в руках участника, ведущий баттла в шутку назвал его
лиственницей.

Источник:
https://www.youtube.com/watch?v=nyfXpR6jwcE

Автор:
Владислав Денисов

Вопрос 3:
Отвечая на вопрос о том, что означает это слово, Роман Павлюченко
предположил, что это место, где продаются куклы. О каком слове
английского происхождения идет речь?

Ответ:
Барбершоп.

Комментарий:
Роман подумал, что в барбершопе продаются куклы "Барби".

Источник:
https://www.youtube.com/watch?v=PB_ViEm2Kmk&t=19s

Автор:
Владислав Денисов

Вопрос 4:
В фильме Горана Дукича в баре играет песня группы "Joy Division", а
новенький в этом месте главный герой рассматривает посетителей и по
характерным признакам пытается угадать, кто как ушел. Кто попадает в
мир, в котором находится этот бар?

Ответ:
Самоубийцы.

Комментарий:
Фильм называется "Самоубийцы: история любви" и повествует о загробном
мире, в который попадают исключительно самоубийцы. Оказавшись в этом
мире, герой отправляется в бар и там встречает девушек, которые
предлагают ему игру: угадать, кто из присутствующих каким образом лишил
себя жизни. Даже музыка в этой сцене подобрана соответствующая: лидер
группы "Joy Division" Иен Кёртис повесился.

Источник:
   1. Х/ф "Самоубийцы: история любви" (2006), реж. Горан Дукич.
   2. https://ru.wikipedia.org/wiki/Кёртис,_Иэн

Автор:
Анастасия Стебалина

Вопрос 5:
   <раздатка>
   M = т + т + пт + т + т + т + пт
   </раздатка>
   В статье под названием "Квантовый круг для чайников" приводится
розданная вам формула, с помощью которой находят гамму. Напишите в своем
ответе то, что мы заменили в тексте предыдущего предложения.

Ответ:
И.

Зачет:
Р.

Комментарий:
Квинтовый круг (или, как еще говорят, квартовый круг) - это
геометрическое представление звуковой системы в виде последовательности
кварт или квинт. Благодаря такой формуле с помощью этого круга можно
найти мажорную гамму от любой ноты, где "т" и "пт" - это "тон" и
"полутон" соответственно.

Источник:
   1. http://ofeed.ru/kvintovyij-krug-dlya-chajnikov
   2. http://www.muz-lit.info/music_dictionary/музыкальные-термины-на-букву-к-рус/

Автор:
Владислав Денисов

Вопрос 6:
(pic: 20180311.jpg)
   Напишите фамилию человека, изображенного на данной карикатуре.

Ответ:
Каспаров.

Комментарий:
Это очередная пародия на картинку "Эволюция человека". В данном случае -
шахматная эволюция. Следующий за человеком шкаф - это суперкомпьютер
1990-х "Deep Blue", первый компьютер, победивший действующего чемпиона
мира по шахматам.

Источник:
http://magichess.uz/index.php?com=content&task=page&id=4743&lang=ru

Автор:
Роман Царегородцев

Вопрос 7:
Второго норвежского короля из династии Хорфагеров звали Эйрик Кровавая
Секира. А третий король Норвегии был прозван ТАКИМ. ТАКОЙ производится с
1998 года и сейчас принадлежит компании "Coca-Cola". Какое слово мы
заменили словом "ТАКОЙ"?

Ответ:
Добрый.

Комментарий:
После Кровавой Секиры кто угодно будет добрым. :-) Во второй части
вопроса речь идет о соке "Добрый".

Источник:
   1. https://ru.wikipedia.org/wiki/Список_королей_Норвегии
   2. https://ru.wikipedia.org/wiki/Мултон

Автор:
Данила Добровольский

Вопрос 8:
В направленном ЕМУ письме с угрозами некорректно были изображены жареные
кольца. Напишите ЕГО имя.

Ответ:
Пауль.

Комментарий:
Осьминог Пауль вызвал недовольство, когда предсказал проигрыш сборной
Германии. В виде жареных колец можно приготовить кальмара, но не
осьминога.

Источник:
Документальный фильм "Жизнь и злоключения Пауля, осьминога-оракула".

Автор:
Наталья Сычёва

Вопрос 9:
В произведениях про ИКСА, которые можно охарактеризовать как ИКС,
встречаются, например, Эксл Роуз и Иззи Стрэдлин. Что мы заменили ИКСОМ?

Ответ:
Слэш.

Комментарий:
Эксл Роуз и Слэш - действующие участники группы "Guns'N Roses", Иззи
Стрэдлин - бывший. Слэш - это еще и жанр фанфикшн-произведений
гомоэротического характера, так что они "встречаются" в смысле "состоят
в отношениях". И друг с другом, и со Слэшем, и вообще на что горазда
фантазия каждого конкретного автора.

Источник:
   1. https://archiveofourown.org/tags/Guns%20N'%20Roses/works
   2. https://ru.wikipedia.org/wiki/Слэш_(жанр)
   3. https://ru.wikipedia.org/wiki/Guns_N%E2%80%99_Roses

Автор:
Анастасия Стебалина

Вопрос 10:
Однажды Лекси Александр познакомилась с представителями фирмы "City
Boys" и после этого стала часто проводить время в пабах и электричках.
Напишите название фильма 2005 года, режиссером которого является Лекси
Александр, используя слова, начинающиеся на различные буквы.

Ответ:
"Хулиганы Зеленой улицы".

Зачет:
"Зеленая улица".

Комментарий:
В пабах и электричках обитают в том числе хулиганы. Футбольная фирма -
это группировка фанатов какого-либо клуба. "City Boys" - это футбольная
фирма клуба "Вальдхоф". "Хулиганы Зеленой улицы" - фильм о другой
фанатской фирме - футбольного клуба "Вест Хэм Юнайтед".

Источник:
   1. https://www.sports.ru/tribuna/blogs/bluewhitenavy/1469270.html
   2. http://rusfan.ru/posts/68065/futbolnie_firmi_firmi_futbolnih_huliganov
   3. https://en.wikipedia.org/wiki/Green_Street_(film)

Автор:
Владислав Денисов

Вопрос 11:
Первым инцидент с участием ИХ пережил Орвилл Райт в 1905 году. Для
предотвращения катастроф с ИХ участием используются собаки, пиротехника
и даже лазеры. Мы не просим назвать ИХ. Упомянув гидроним, назовите
событие 2009 года, виновниками которого стали ОНИ.

Ответ:
Чудо на Гудзоне.

Зачет:
Посадка на Гудзон и т.п. по слову "Гудзон".

Комментарий:
ОНИ - это птицы. "Чудо на Гудзоне" (англ. Sully) - американский
биографический драматический фильм о посадке самолета.

Источник:
   1. https://en.wikipedia.org/wiki/Bird_strike#History_of_incidents
   2. https://en.wikipedia.org/wiki/US_Airways_Flight_1549

Автор:
Ирина Абдрашитова

Вопрос 12:
Прослушайте ситуацию из КВН. Молодой человек предлагает девушке сыграть
в ИКСА. После того как та соглашается, он тут же заявляет: "Ты
победила". Один известный ИКС был по профессии ИКСОМ. Назовите его имя.

Ответ:
Гена.

Зачет:
Валера.

Комментарий:
Согласно шутке, не очень красивая девушка автоматически побеждает в игре
"Крокодил". Крокодил Гена, как и его сменщик крокодил Валера, работал в
зоопарке крокодилом.

Источник:
   1. КВН-2017. Высшая лига. Вторая 1/2 финала. "Плюшки имени Ярослава
Гашека". Приветствие.
https://www.youtube.com/watch?v=CW5tO7ursDU&t=1m45s
   2. https://ru.wikipedia.org/wiki/Крокодил_Гена

Автор:
Владислав Денисов

Вопрос 13:
Дуплет.
   1. Водить ИКСА - традиционная белорусская игра-хоровод, с помощью
которой кликали весну. Назовите ИКСА одним словом.
   2. В некоторых источниках утверждается, что Сергей Петрович Боткин
доказал: объем АЛЬФЫ зависит от душевного состояния. С этим фактом
связывают один из псевдонимов известного человека. Назовите АЛЬФУ одним
словом.

Ответ:
   1. Селезень.
   2. Селезенка.

Комментарий:
   1. Селезень - символ весны.
   2. Псевдоним Чехова - "Человек без селезенки".

Источник:
   1. https://ru.wikipedia.org/wiki/Селезень_(карагод)
   2. http://www.rg-rb.de/index.php?option=com_rg&task=item&id=6396&Itemid=0

Автор:
Данила Добровольский

Вопрос 14:
В одном из клипов певица Petite Meller проникается видами саванны,
африканскими танцами и общением с жирафами. Также в этом клипе можно
увидеть, как исполнительница ДЕЛАЕТ ЭТО в розовом платье. Памятник
герою, который установлен в одном датском городе, тоже ДЕЛАЕТ ЭТО. Мы не
просим вас ответить, что в тексте вопроса мы заменили словами "ДЕЛАТЬ
ЭТО". Назовите этого героя.

Ответ:
Стойкий оловянный солдатик.

Зачет:
Оловянный солдатик.

Комментарий:
ДЕЛАТЬ ЭТО - стоять на одной ноге. Певица в клипе подражает фламинго.
Памятник установлен в Оденсе - родном городе Ганса Христиана Андерсена.

Источник:
   1. https://www.wonderlandmagazine.com/2015/02/16/behind-scenes-petite-meller-baby-love/
   2. https://www.liveinternet.ru/community/3299606/post228921343/

Автор:
Владислав Денисов

Вопрос 15:
Сорокапятилетний спортсмен Нориаки Касаи уже принял участие в семи
зимних Олимпийских играх и планирует выступать, пока будет позволять
здоровье. Автор вопроса нашел в имени спортсмена топоним. Назовите его.

Ответ:
Нокиа.

Комментарий:
Такой же долговечный, как телефоны "Nokia". Нокиа - город в Финляндии.

Источник:
   1. https://ru.wikipedia.org/wiki/Касай,_Нориаки
   2. ЛОАВ.

Автор:
Анастасия Стебалина

Вопрос 16:
[Ведущему: не озвучивать кавычки.]
   Чтобы тело Ленина "не отсвечивало", священнослужитель Дмитрий Смирнов
предложил переместить его на АЛЬФУ. На одной юмористической картинке
известный отрицательный персонаж хвалит "АЛЬФУ". Какие три слова мы
заменили словом "АЛЬФА"?

Ответ:
Обратная сторона Луны.

Зачет:
Темная сторона Луны.

Комментарий:
По мнению протоиерея, тело Ленина нужно отправить куда подальше, а
именно - на обратную сторону Луны. На одной из картинок альбом "Pink
Floyd" "The Dark Side of the Moon" хвалит Дарт Вейдер.

Источник:
   1. https://www.1tv.ru/shows/pozner/vypuski/gost-protoierey-dmitriy-smirnov-pozner-vypusk-ot-20-03-2012
   2. https://www.gocomics.com/brevity/2012/04/06/

Автор:
Владислав Денисов

Вопрос 17:
Героиня одного произведения, описывая татуировку мужчины, говорит, что
она была "в виде якоря, обвитого ИМ", и предполагает, что изображение
является эмблемой яхт-клуба этого мужчины. На самом же деле татуировка
представляет собой бьющий сквозь НЕГО фонтан нефти. Изобразите ЕГО.

Ответ:
$.

Зачет:
С одной или двумя чертами.

Комментарий:
Девушка смотрела на татуировку в перевернутом виде, и ее предположение о
смысле рисунка было следствием ассоциации "материальный достаток -
яхта", тогда как на самом деле на руке мужчины был изображен источник (в
прямом смысле) его доходов.

Источник:
В.О. Пелевин. Шлем ужаса. Миф о Тесее и Минотавре.
http://pelevin.nov.ru/romans/pe-shlem/index.html

Автор:
Татьяна Хадыева

Вопрос 18:
Карл Саган пишет, что в античности БОГИ могли быть как злыми, так и
добрыми. С приходом христианства ситуация изменилась, и в сочинении "О
Граде Божьем" Блаженный Августин окончательно ОБОЖЕСТВЛЯЕТ БОГОВ. Какие
слова мы заменили словами "ОБОЖЕСТВЛЯЕТ БОГОВ"?

Ответ:
Демонизирует демонов.

Комментарий:
Изначально демоны могли быть и хорошими, Блаженный Августин убеждает,
что все они злые, т.е. демонизирует их.

Источник:
К. Саган. Мир, полный демонов. http://flibusta.is/b/360877/read

Автор:
Роман Царегородцев

Вопрос 19:
Герой советского мультфильма в одном устройстве использует заводной
автобус, который ездит по кругу. Мы не просим вас ответить, вместо чего
в этом устройстве такой автобус используется. Назовите это устройство,
название которого образовано от названия французской
компании-производителя.

Ответ:
Патефон.

Комментарий:
Вместо привычной иглы на механической ножке и двигателя, вращающего
виниловую пластинку, - заводной автобус, который ездит по ее дорожкам.
Название "патефон" произошло от названия французской компании "Пате".

Источник:
   1. Мультфильм "Его жена - курица" (1990).
https://www.youtube.com/watch?v=bhM3FaR7pdQ?t=2m13s
   2. https://ru.wikipedia.org/wiki/Патефон

Автор:
Владислав Денисов

Вопрос 20:
Согласно одной математической шутке, ИХ в жизни не одно, а два: в 6 и 28
лет. Назовите ИХ.

Ответ:
Совершеннолетия.

Комментарий:
В математике 6 и 28 - это совершенные числа. До следующего совершенного
числа (496) дожить человеку будет затруднительно.

Источник:
   1. https://www.youtube.com/watch?v=hkhaipY3JmU&t=56m11s
   2. https://ru.wikipedia.org/wiki/Совершенное_число

Автор:
Данила Добровольский

Вопрос 21:
Диетолог Эрика Ломбарди, рассуждая о пользе здорового питания, говорит:
"Чем длиннее ОНА, тем короче жизнь". Какое слово французского
происхождения мы заменили словом "ОНА"?

Ответ:
Этикетка.

Комментарий:
Чем короче этикетка, тем меньше в продукте добавок - а значит, он более
натуральный.

Источник:
https://www.sports.ru/tribuna/blogs/russiateam/1278214.html

Автор:
Владислав Денисов

Вопрос 22:
[Ведущему: не озвучивать кавычки.]
   Выпуск одного из российских ток-шоу, посвященный девушкам, готовым на
многое ради покорения богатого мужчины, называется так: "СДЕЛАЛИ ЭТО:
секреты охотниц за миллионерами." Существует версия, что выражение
"СДЕЛАТЬ ЭТО" появилось благодаря калашных дел мастерам. Какие два слова
мы заменили словами "СДЕЛАТЬ ЭТО"?

Ответ:
Раскатать губу.

Комментарий:
Многие девушки в погоне за красотой увеличивают размер своих губ. Есть
версия, что выражение "раскатать губу" появилось так: на калаче делали
надрез, называемый губой, засыпАли в него муку, а затем раскатывали. Эта
процедура помогала изделию сохранять свою форму при выпечке.

Источник:
   1. https://www.youtube.com/watch?v=fvY1Cum3FfU
   2. http://www.bolshoyvopros.ru/questions/2160232-raskatat-gubu-chto-znachit-eto-vyrazhenie-pochemu-govorjat-raskatal-gubu.html
   3. http://otzovik.com/review_4817191.html

Автор:
Владислав Денисов

Вопрос 23:
Юлия Меламед считает, что этот небольшой рассказ имел очень значительные
последствия для философии и естественных наук. Напишите название этого
рассказа.

Ответ:
"И грянул гром".

Комментарий:
Маленькие вещи влекут за собой большие события не только в тексте
рассказа.

Источник:
https://www.gazeta.ru/comments/column/melamed/10642139.shtml

Автор:
Роман Царегородцев

Вопрос 24:
На 83-й минуте футбольного матча "Манчестер Сити" - "Бернли" вратарь Ник
Поуп спас ворота гостей от удара Габриэля Жезуса. Твиттер "Бернли",
говоря об этом событии, в конце сообщения употребляет слово древнего
происхождения. Какое именно слово?

Ответ:
Аминь.

Зачет:
Amen.

Комментарий:
Слово "Pope" [пОуп] переводится с английского как "римский папа", а
фамилия бразильского нападающего "Сити" пишется как Jesus [джИзас] -
Иисус. Фраза "в конце сообщения" также является подсказкой, поскольку
восклицанием "Аминь!" часто заканчивают молитвы. Происхождение слова
"аминь" восходит к древнегреческому и древнееврейским языкам.

Источник:
   1. https://www.sports.ru/tribuna/blogs/greatness/1447270.html
   2. https://krylov.academic.ru/67/

Автор:
Владислав Денисов

Тур:
18 тур. "И кстати, Гарри" (Москва - Санкт-Петербург - Самара - Саратов)

Вопрос 1:
(pic: 20180312.jpg)
   Перед вами иероглифы, которые переводятся как "ДЕЛАТЬ ЭТО".
Заглавному герою известного фильма посоветовали не отвлекаться, когда он
ДЕЛАЕТ ЭТО. Ответьте, что мы заменили словами "ДЕЛАЕТ ЭТО", используя
слово с дефисом.

Ответ:
Играет в пинг-понг.

Комментарий:
Различные направления штрихов в иероглифах как бы имитируют
отскакивающий от стола мяч; в фильме "Форрест Гамп" заглавный герой
достиг успехов в этом виде спорта.

Источник:
   1. https://translate.google.ru/#zh-CN/en/%E6%89%93%E4%B9%92%E4%B9%93%E7%90%83
   2. Х/ф "Форрест Гамп" (1994), реж. Роберт Земекис.

Автор:
Елизавета Ежергина

Вопрос 2:
Внимание, в этом вопросе словом "ИКС" мы заменили два слова.
   В одном советском фильме прибывшие в Заполярье исследователи при
распаковке груза обнаружили ИКСА по имени Петр. Другие ИКСЫ зимой не
боятся людей. Назовите ИКСА двумя словами.

Ответ:
Арктический заяц.

Комментарий:
Петр Молибога приплыл в Арктику "зайцем". Несмотря на стереотип о
трусливости зайца, арктический заяц не боится человека - в основном,
из-за того, что на фоне снега его практически не видно. Зато он боится
арктического волка и белую сову.

Источник:
   1. https://ru.wikipedia.org/wiki/Семеро_смелых
   2. https://ru.wikipedia.org/wiki/Арктический_беляк

Автор:
Федор Жельне

Вопрос 3:
Софья Вдовиченко несколько парадоксально утверждает, что в некоторых
субкультурах английский играет роль ПРОПУСК. Заполните пропуск двумя
иностранными словами.

Ответ:
Lingua Franca.

Комментарий:
Lingua franca - язык или диалект, систематически используемый для
коммуникации между людьми, родными языками которых являются другие
языки. В переводе с итальянского - "франкский язык", что к английскому,
конечно, малоприменимо.

Источник:
   1. С.С. Вдовиченко. Речевой портрет языковой личности игрока в DotA2.
// "Научный диалог", 2016, N 2 (50). - С. 9-20.
   2. https://ru.wikipedia.org/wiki/Лингва_франка

Автор:
Анастасия Белова

Вопрос 4:
(pic: 20180313.jpg)
   Говоря о видоизменениях арабской буквы "айн", автор вопроса
процитировал популярную песню. Назовите приведенное в этой песне
определение любви.

Ответ:
Только то, что кажется.

Комментарий:
Некоторые отечественные востоковеды называют этот символ "узелком", что
логично в контексте вязи. В зависимости от положения в слове этот узелок
то завязывается, то развязывается. По мнению Алены Апиной, любовь - она
и есть только то, что кажется.

Источник:
   1. ЛОАВ.
   2. http://www.megalyrics.ru/lyric/aliena-apina/uzielki.htm

Автор:
Паша Казначеев

Вопрос 5:
Женщины рождаются с ограниченным количеством яйцеклеток, набор которых
формируется на четвертом месяце жизни эмбриона. Автор посвященной этому
статьи говорит, что формально каждый из нас побывал не только в БЕТЕ,
но, пусть и недолго, в АЛЬФЕ. Назовите АЛЬФУ двумя словами.

Ответ:
Бабушкина утроба.

Зачет:
По смыслу с упоминанием бабушки.

Комментарий:
Та самая яйцеклетка, из которой вы сформировались, уже существовала в
теле четырехмесячного эмбриона, который впоследствии стал вашей матерью.

Источник:
https://genedoe.wordpress.com/2010/09/29/in-your-grandmothers-womb-the-egg-that-made-you/

Автор:
Мария Иванова

Вопрос 6:
17 октября 2017 года умер канадский рок-музыкант Гордон Дауни. На
следующий день зрители местных телеканалов могли увидеть ИХ. Назовите ИХ
двумя словами, начинающимися на соседние буквы алфавита.

Ответ:
Слезы Трюдо.

Комментарий:
Джастин Трюдо широко известен как самый человечный политический лидер
современности, вот и после смерти любимого музыканта заплакал.

Источник:
https://www.independent.co.uk/arts-entertainment/music/news/justin-trudeau-crying-gord-downie-tribute-canada-tragically-hip-dead-dies-age-a8008246.html

Автор:
Паша Казначеев

Вопрос 7:
Евгений Евтушенко во время поездки в Каир сказал (цитата): "Пора сказать
"нет" философии кабычегоневышлизма". Услышав это, знакомый автора
вопроса задумался о том, чтобы сменить профессию. Ответьте двумя
словами: кем работал этот знакомый?

Ответ:
Синхронным переводчиком.

Комментарий:
Несмотря на то что эта фраза вполне может заставить сомневаться в том,
что ты всё в жизни делаешь правильно, знакомым двигали более тривиальные
соображения: перевести такую конструкцию на арабский с ходу не так-то
просто.

Источник:
   1. Лекция А.В. Германовича.
   2. https://www.thecairoreview.com/q-a/plight-of-an-arab-intellectual/

Автор:
Паша Казначеев

Вопрос 8:
ИКС в данном вопросе - замена.
   В различных вариациях ИКСА упоминаются город мэров и
библиографический каталог, содержащий ссылки на другие библиографические
каталоги. Назовите профессию, фигурирующую в самой популярной вариации
ИКСА.

Ответ:
Брадобрей.

Комментарий:
ИКС - парадокс Рассела, известный как парадокс брадобрея. Популярная
формулировка "Если брадобрей бреет всех, кто не бреется сам, то бреет ли
брадобрей себя?" в различных трактовках звучит как "Если все мэры живут
не в своем городе, а в отдельном городе мэров, то где живет мэр города
мэров?" и "Если в библиографическом каталоге указаны все каталоги, в
которых нет ссылки на самих себя, следует ли в него поместить ссылку на
сам этот каталог?".

Источник:
http://ru.math.wikia.com/wiki/Парадокс_Рассела

Автор:
Костя Шведов

Вопрос 9:
Министр сельского хозяйства одной небольшой страны недавно организовал
массовую молитву о дожде около НЕЕ. Назовите ЕЕ двумя словами.

Ответ:
Стена Плача.

Комментарий:
Последние пять лет в Израиле выдались засушливыми, и министр сельского
хозяйства Ури Ариэль решил, что все средства хороши, и созвал людей на
молитву. Возможно, Стена Плача выбрана из-за условной похожести дождя и
слезоотделения.

Источник:
https://www.timesofisrael.com/thousands-pray-at-western-wall-for-rain-to-break-drought/

Автор:
Паша Казначеев

Вопрос 10:
Санбенито - накидка, которую инквизиторы надевали на осужденных. По
словам Андрея Михайлова, изображенные на санбенито языки пламени могли
выглядеть по-разному и несли то же значение, которое раньше иногда нес
ОН. Назовите ЕГО двумя словами.

Ответ:
Большой палец.

Комментарий:
Поднимающиеся языки пламени означали, что еретик осужден на сожжение,
опускающиеся - что на него наложена епитимья. По мнению других
источников, еретику в санбенито с направленным вниз пламенем всё же
полагалась смерть, но менее мучительная. Существуют разные версии
относительно того, что означали различные положения большого пальца во
время гладиаторских боев, но участие он принимал точно.

Источник:
Вольтер. Избранные сочинения (примечания).
https://books.google.ru/books?id=-YWECwAAQBAJ&pg=PT258#v=onepage&q&f=false

Автор:
Паша Казначеев

Вопрос 11:
Одна из работающих с "AliExpress" компаний вкладывает в свои посылки
флаеры, в тексте которых говорится, что приключения персонажей одного
произведения не закончились. Назовите это произведение.

Ответ:
"Путешествие на Запад".

Комментарий:
Один из четырех классических романов китайской литературы "Путешествие
на запад" рассказывает о путешествии в Индию за священными сутрами.
Никакого отношения к деятельности интернет-магазинов это не имеет, но
китайские коммерсанты всё равно решили обыграть свое наследие, так как
большинство товаров отправляется на запад и северо-запад.

Источник:
   1. Флаер, приложенный к посылке, полученной автором вопроса.
   2. https://ru.wikipedia.org/wiki/Путешествие_на_Запад

Автор:
Паша Казначеев

Вопрос 12:
Томас Вентворт Рассел, также известный как Рассел-, долгое время был
начальником полиции Каира. Джон Бэгот Глабб, также известный как Глабб-,
возглавлял армию Иордании. Какие буквы мы дважды пропустили в тексте
этого вопроса?

Ответ:
Паша.

Комментарий:
Звучит очень непривычно, но некоторые европейцы тоже получали в
османской номенклатуре титул паши. Всегда мечтал написать вопрос со
своим именем в ответе.

Источник:
   1. https://en.wikipedia.org/wiki/Thomas_Wentworth_Russell
   2. https://en.wikipedia.org/wiki/John_Bagot_Glubb

Автор:
Паша Казначеев

Вопрос 13:
В одном из эпизодов фильма "Она" упоминается удушение кошачьим хвостом.
В другом эпизоде героиня восторгается мироощущением искусственного
интеллекта и, можно сказать, упоминает ИХ. Назовите ИХ двумя словами.

Ответ:
Оттенки серого.

Комментарий:
Удушение хвостом кошки фигурирует как фрагмент очень уж дикого полового
акта. Правда, акт телефонный. Фильм "Она" описывает любовь человека к
искусственному интеллекту, в нем задается вопрос о том, где граница
между полноценной личностью и способным к обучению и переживанию эмоций
искусственным интеллектом.

Источник:
Х/ф "Она" (2013), реж. Спайк Джонс.

Автор:
Паша Казначеев

Вопрос 14:
АЛЬФА рядом со студией современной художницы Нандиты Кумар вдохновила ее
на одну из работ, частью которой является сборник рецептов. Ответ на
вопрос, который, вероятно, не раз прозвучал в течение прошедших
нескольких часов, в контексте работы Кумар кажется очевидным. Какое
слово мы заменили АЛЬФОЙ?

Ответ:
Трава.

Комментарий:
(pic: 20180314.jpg)
   Наслаждаясь современным искусством (включая пакеты ЧГК для
марафонов), люди иногда задаются вопросом "Что курил автор?". Источник
вдохновения Нандиты Кумар - растущие рядом с ее студией сорняки, которые
вдохновили ее на создание инсталляции и сборника медицинских рецептов.

Источник:
http://www.livemint.com/Leisure/ZMaCi3D7yhPYR8s1S8qOAO/Goa-a-giver-of-many-gifts.html

Автор:
Паша Казначеев

Вопрос 15:
(pic: 20180315.jpg)
   В 2016 году была найдена эта написанная более ста лет назад картина,
сохранившаяся в идеальном состоянии. Назовите начальника ее автора.

Ответ:
Роберт Скотт.

Комментарий:
Эдвард Уилсон, участник экспедиции Скотта, был хорошим художником.
Картина отлично сохранилась в хижине в Антарктиде. Члены экспедиции
умерли от обморожения, птице на картине тоже плохо.

Источник:
https://www.stuff.co.nz/science/93617829/Kiwi-researchers-discover-almost-perfectly-preserved-118-year-old-painting-in-Antarctica

Автор:
Паша Казначеев

Вопрос 16:
В одном из выпусков мультсериала "Гриффины" герои попадают в страну, где
всё привычное заменено аналогом похуже. Так, вместо ПЕРВЫХ там
показывают ВТОРЫХ. Назовите ПЕРВЫХ и ВТОРЫХ в правильном порядке.

Ответ:
"Симпсоны", "Гриффины".

Комментарий:
Такая самоирония очень в духе "Гриффинов". Эта страна, кстати, Россия.

Источник:
Мультсериал "Гриффины", s16e07.

Автор:
Паша Казначеев

Вопрос 17:
Можно сказать, что Круз Веласкес умер ПРОПУСК, когда выпил желтую
жидкость, доказывая таможенникам, что это сок. Назовите короткое имя
того, кто спит под композицию "ПРОПУСК" в произведении 2004 года.

Ответ:
Пин.

Комментарий:
Наркокурьер отпил из бутылки с жидким метамфетамином и умер "от винта".
Под одноименную песню в одной из серий "Смешариков" спал пингвин.

Источник:
   1. https://www.independent.co.uk/news/world/americas/mexican-teenager-crystal-meth-death-us-border-officers-cruz-velazquez-apple-juice-cctv-footage-a7868411.html
   2. Мультсериал "Смешарики", серия "Полеты во сне и наяву".
https://www.youtube.com/watch?v=38a0f-fXZCc

Автор:
Саша Карчевский

Вопрос 18:
Сайт cosmo.ru определяет ЭТО как распространенную пластическую операцию
среди девушек, помешанных на современных стандартах красоты. Данное
определение послужило эпиграфом к одному из занятий Летней
математической школы в Кирове. Назовите ЭТО двумя словами.

Ответ:
Удаление ребер.

Комментарий:
Удаление ребер - широко используемая операция в теории графов.

Источник:
   1. https://www.cosmo.ru/beauty/star_beauty/bes-v-rebro-zvezdy-udalivshie-rebra-radi-tonkoy-talii/
   2. (pic: 20180316.jpg)

Автор:
Костя Шведов

Вопрос 19:
В песне, все доходы от продажи которой пойдут на благотворительность,
певица Франческа Бланшар говорит, что Мария оставила ее без дома и
утопила мечты. Гораздо больше подобных песен посвящены не Марии, а ЕЙ.
Назовите ЕЕ.

Ответ:
Катрина.

Комментарий:
Бланшар посвятила урагану "Мария", обрушившемуся на Пуэрто-Рико в этом
году, двусмысленную историю несчастной любви. Существуют десятки песен
об урагане "Катрина".

Источник:
https://francescablanchard.bandcamp.com/album/maria-single

Автор:
Паша Казначеев

Вопрос 20:
(pic: 20180317.jpg)
   На этой иллюстрации из игры "Magic: the Gathering" изображен умелый
ОН. Назовите ЕГО.

Ответ:
Ниндзя.

Комментарий:
По-настоящему умелого ниндзя мы никогда не увидим.

Источник:
http://gatherer.wizards.com/Pages/Card/Details.aspx?name=masterful%20ninja

Автор:
Паша Казначеев

Вопрос 21:
Дуплет.
   Надеемся, что ответ на первый вопрос поможет вам разобраться с
ответом на второй вопрос.
   1. Одним из самых популярных развлечений для жителей тайского города
Чингмая является Дом снега. Ответьте одним словом: посетив какую
достопримечательность города, можно заглянуть в этот дом?
   2. В одной шутке молодой человек просит Иисуса послать ему такую же
девушку, как и у другого участника вечеринки, но получает отказ со
ссылкой на известного уроженца Вены. Автор вопроса пришел к выводу, что
девушка - это ОН. Какие буквы мы пропустили в этом вопросе?

Ответ:
   1. Зоопарк.
   2. ферми.

Комментарий:
   1. Таиланд - жаркая страна, и павильон с искусственным снегом
находится в зоопарке, где местные жители, помимо наблюдения за
привыкшими к холодным температурам животными, могут поиграть в снежки и
просто увидеть снег.
   2. Иисус отказывает молодому человеку, ссылаясь на принцип Вольфганга
Паули, запрещающий двум фермионам находиться в одном и том же квантовом
состоянии.
   Гипотеза зоопарка, состоящая в том, что представители внеземной жизни
предпочитают не вмешиваться в жизнь на Земле и ограничиваются
наблюдением за ее развитием, является одним из разрешений парадокса
Ферми.

Источник:
   1. http://pattayatrip.ru/chiang-may/zoopark
   2. https://pikabu.ru/story/printsip_pauli_2121906
   3. https://ru.wikipedia.org/wiki/Гипотеза_зоопарка

Автор:
Костя Шведов

Вопрос 22:
В романе Роджера Желязны герои основывают колонию на другой планете
после гибели Земли. В 1979 году роман было решено экранизировать под
названием "ОН", но этого так и не произошло. На другом НЕМ, по разным
источникам, находилось от 35 до 60 человек. Назовите ЕГО.

Ответ:
"Арго".

Комментарий:
Сценарий фильма "Арго" к роману Желязны был выкуплен в 1980 году ЦРУ для
проведения операции "Канадская хитрость" по спасению заложников в
Тегеране, ставшей известной благодаря оскароносному фильму "Операция
"Арго"". Число аргонавтов, погнавшихся за Золотым руном, доподлинно
неизвестно.

Источник:
   1. https://ru.wikipedia.org/wiki/Князь_Света
   2. https://ru.wikipedia.org/wiki/Канадская_хитрость
   3. Х/ф "Операция "Арго"" (2012), реж. Бен Аффлек.
   4. https://ru.wikipedia.org/wiki/Аргонавты

Автор:
Костя Шведов

Вопрос 23:
Британского философа Алана Уоттса интересовала, помимо прочего,
категория трансцендентальности в восточных религиях. Назовите фильм 2013
года, в котором можно услышать персонажа по имени Алана Уоттс.

Ответ:
"Она".

Комментарий:
Важную роль в фильме играет саморазвивающийся искусственный интеллект. В
какой-то момент Саманта знакомится с созданным программистами
искусственным Аланом Уоттсом. Его, вероятно, сильно заинтересовала бы
идея интеллекта, не заключенного в физическое тело.

Источник:
Х/ф "Она" (2013), реж. Спайк Джонс.

Автор:
Паша Казначеев

Вопрос 24:
Для ИХ запоминания существует мнемоника из нескольких "F", где первой
соответствует "French". Назовите любую из НИХ двумя словами.

Ответ:
Первая база.

Зачет:
Вторая база; третья база; четвертая база.

Комментарий:
Очень распространенная в США реалия нам часто непонятна. Первая база в
контексте плотских отношений - это французский поцелуй.

Источник:
https://en.wikipedia.org/wiki/Baseball_metaphors_for_sex

Автор:
Паша Казначеев

Тур:
19 тур. "Гарася Педулаев и дореволюционный контражиотаж" (Кукисвумчорр -
Стерлитамак - Воронеж - Ивантеевка - Санкт-Петербург - Нижний Новгород)

Редактор:
Сергей Терентьев (Санкт-Петербург)

Инфо:
Редактор благодарит за помощь при подготовке пакета Антона Волосатова,
Сергея Лобачёва, Александра Нечаева, Марию Подрядчикову, Николая
Слюняева, Дмитрия Тарарыкова, Руслана Хаиткулова, Олега Холодова, Романа
Цуркана.

Вопрос 1:
Константин Долматов описал российскую политическую элиту палиндромом из
трех слов: второе слово - союз "да"; третье слово является фамилией.
Напишите этот палиндром полностью.

Ответ:
Воры да Кадыров.

Комментарий:
Автор вопроса на всякий случай извиняется.

Источник:
https://kuznec-d-k.livejournal.com/62086.html

Автор:
Антон Волосатов (Ивантеевка)

Вопрос 2:
Появление этого имени в советском фильме вряд ли случайно - ведь в
тюрьмах почти так же, по созвучию, называли стукачей. Напишите это имя.

Ответ:
Навуходоносор.

Комментарий:
На ухо доносит. В фильме "Джентльмены удачи" звучит ряд обидных прозвищ:
"Сарделька, сосиска, редиска, Навуходоносор, петух гамбургский!".

Источник:
Л.В. Беловинский. Энциклопедический словарь советской повседневной
жизни. https://culture.wikireading.ru/78011

Автор:
Сергей Терентьев (Санкт-Петербург)

Вопрос 3:
ИХ иногда обозначают как "СВ". Говоря о похвальной статистике, один
чиновник сравнил ИХ с подводными лодками. Назовите ИХ.

Ответ:
Столыпинские вагоны.

Зачет:
Вагонзаки; тюремные вагоны; вагоны для арестантов.

Комментарий:
Обычно СВ - это спальный вагон повышенной комфортности, чего о тюремной
транспортировке сказать, скорее всего, нельзя. Успешных побегов из таких
вагонов новейшая история действительно не знает.

Источник:
   1. https://www.segodnya.ua/investigations/Kak-v-Ukraine-perevozyat-zekov.html
   2. https://www.mk.ru/social/2016/04/04/v-odnom-vagone-s-zekami-chto-zhdet-osuzhdennykh-na-puti-v-koloniyu.html

Автор:
Антон Волосатов (Ивантеевка)

Вопрос 4:
На одной карикатуре помост висельника представляет собой ЕЕ, у которой
сидят два палача. Убийцей станет тот из палачей, который будет менее
аккуратен и удачлив. Назовите ЕЕ.

Ответ:
Дженга.

Комментарий:
Палачи по очереди вынимают элементы дженги, из которой и состоит помост.

Источник:
http://joyreactor.cc/post/3246731

Автор:
Сергей Терентьев (Санкт-Петербург)

Вопрос 5:
В 1677 году в Англии для большего реализма в НЕГО запихнули живых кошек.
Назовите ЕГО, использовав имя собственное.

Ответ:
Чучело [Гая] Фокса.

Комментарий:
Создавалось ощущение, что чучело Фокса кричит в агонии в момент
сожжения.

Источник:
https://en.wikipedia.org/wiki/Guy_Fawkes_Night

Автор:
Сергей Терентьев (Санкт-Петербург)

Вопрос 6:
В песне Луперкаля описывается тощий нарк, у которого даже ОНИ ДЕЛАЮТ
ЭТО. Какие два слова мы заменили словами "ОНИ ДЕЛАЮТ ЭТО"?

Ответ:
Ребра торчат.

Источник:
https://genius.com/And-luperkal-and-vadyara-blues-andrey-lyrics

Автор:
Сергей Лобачёв (Нижний Новгород)

Вопрос 7:
   <раздатка>
   Предавались бы неге в чертоге.
   Путь бы не был туманно-тернист.
   И Ахматовой ПРОПУСК
   Продвигал бы другой акмеист.
   </раздатка>
   Одно шуточное стихотворение описывает, как могли бы развиваться
события, если бы Черубина де Габриак не оказалась мистификацией и ее
встреча с Гумилёвым состоялась. В отрывке этого стихотворения мы сделали
пропуск и заменили три буквы в одном из слов. Заполните пропуск.

Ответ:
Бледные ноги.

Комментарий:
   Равнодушной толпою оплеван
   конквистадор, и предан огню.
   Не пошла Габриак с Гумилёвым -
   и сгубила его на корню.
   Плачет ангел. Разбиты скрижали.
   Что ж ты, дура, такое творишь?!
   Вышла б замуж. Детей нарожали
   бы. Нджамена. Кейптаун. Париж.
   Предавались бы неге в чертоге.
   Путь бы не был туманно-тернист.
   И Ахматовой бледные ноги
   Раздвигал бы другой акмеист.

Источник:
https://ru-matstihi.livejournal.com/484964.html

Автор:
Антон Волосатов (Ивантеевка)

Вопрос 8:
Один рецензент, говоря о сомнительных выводах, написал, что полученные
данные выглядят менее правдоподобно, чем ОН в часах с кукушкой. Назовите
ЕГО двумя словами, начинающимися на одну и ту же букву.

Ответ:
Птичий помет.

Зачет:
Помет птицы.

Источник:
Франс де Вааль. Достаточно ли мы умны, чтобы судить об уме животных?
https://books.google.ru/books?id=nTVlDQAAQBAJ&pg=PT88#v=onepage&q&f=false

Автор:
Сергей Терентьев (Санкт-Петербург)

Вопрос 9:
   <раздатка>
   ... [REDACTED] ...
   </раздатка>
   В названии одного из заданий последней части авторы намекают, что
наименее цензурный вариант все-таки верен. Восстановите закрытые от вас
два слова.

Ответ:
Big, gun.

Комментарий:
Не существует официальной расшифровки названия знаменитой пушки из
"Дума" BFG9000 [би-эф-джи девять тысяч] и ее модификаций, но наиболее
популярным является вариант "Big Fucking Gun" [биг фАкин ган]. Слово
"Redacted" [ридЭктэд] - синоним "Censored" [сЕнсод] и русскоязычной
купюры "вырезано цензурой".

Источник:
https://ru.wikipedia.org/wiki/BFG9000

Автор:
Антон Волосатов (Ивантеевка)

Вопрос 10:
По легенде, начало ЕМУ положили так называемые "Клубы ключей" -
специальные вечеринки, на которых все участники складывали ключи от
своих апартаментов в одну фуражку. Назовите ЕГО словом английского
происхождения.

Ответ:
Свинг.

Комментарий:
Затем девушки вытягивали случайный ключ и отправлялись проводить ночь с
его владельцем. Постоянно рискующие умереть в ходе задания летчики таким
образом заключали негласный договор, по которому оставшиеся в живых
должны позаботиться о возлюбленных своих погибших товарищей.

Источник:
   1. https://www.the-village.ru/village/city/ustory/233027-kink
   2. https://ru.wikipedia.org/wiki/Свинг_(секс)

Автор:
Сергей Терентьев (Санкт-Петербург)

Вопрос 11:
На карикатуре 1917 года ОНА переживает, что не сможет найти общий язык с
новой властью так же легко, как раньше, ведь в совете рабочих и
крестьянских депутатов больше 2000 человек! Назовите ЕЕ.

Ответ:
[Матильда] Кшесинская.

Зачет:
Матильда.

Комментарий:
Если император был в единственном числе, то членов временного
правительства было многовато.

Источник:
https://daily.afisha.ru/brain/6892-svoey-seme-on-byl-veren-istorik-o-romane-mezhdu-nikolaem-ii-i-matildoy/

Автор:
Руслан Хаиткулов (Москва)

Вопрос 12:
(pic: 20180318.jpg)
   По мнению одного интернет-пользователя, на розданном вам логотипе
вейп-шопа изображен ТАКОЙ ОН. Слово "ОН" - ТАКОГО происхождения. Что мы
заменили словами "ТАКОЙ ОН"?

Ответ:
Голландский штурвал.

Источник:
   1. https://twitter.com/andavior/status/828213483397226498/
   2. https://ru.wikipedia.org/wiki/Штурвал

Автор:
Мария Подрядчикова (Волгоград)

Вопрос 13:
Говорят, что в жизни каждого восточного поэта значимы две фигуры -
возлюбленная и покровитель. О женщинах в жизни Алишера Навои история
умалчивает, но мы знаем, что Султан-Хусейн сделал его великим визирем.
Автор одной книги пишет, что, несмотря на должность, Навои участвовал в
строительных работах, на которых совершал некое действие, т.е. был
ИКСОМ. Назовите ИКСА двухкоренным словом.

Ответ:
Глиномес.

Источник:
В.Н. Ерёмин. 100 великих поэтов.

Автор:
Руслан Хаиткулов (Москва)

Вопрос 14:
[Ведущему: сперва ошибочно прочитать фамилию Анненский с ударением на
третий слог.]
   Иннокентий Анненский женился на вдове намного старше себя и якобы
тяготился этим браком. Рассказывают, что однажды, помогая мужу во время
одного из сердечных приступов, жена случайно сломала его ЭТО, после чего
тот смог свободно задышать. Назовите ЭТО.

Ответ:
[Обручальное] кольцо.

Комментарий:
Кольцо долгое время не получалось снять с пальца, что, по мысли
рассказчиков, влияло на кровообращение. Вышло довольно символично.
Впрочем, в итоге Анненский всё же умер из-за своего слабого сердца.

Источник:
   1. http://annensky.lib.ru/names/dina/annnv_name.htm
   2. http://annensky.lib.ru/names/krivich/krivich2-1.htm

Автор:
Антон Волосатов (Ивантеевка)

Вопрос 15:
(pic: 20180319.jpg)
   Недавно ученые в очередной раз нашли ЭТО. У автора вопроса 2008 года
Александра Мальцева ЭТО было почти вдвое короче. Назовите ЭТО четырьмя
словами.

Ответ:
Самое большое простое число.

Комментарий:
Компьютер американского инженера Джонатана Пейса обнаружил пятидесятое
простое число Мерсенна длиной более 23 миллионов символов, которое на
данный момент является самым большим из известных простых чисел. В 2008
году самое большое из известных простых чисел состояло из 13 миллионов
цифр. На раздаточном материале - кадр из клипа на песню "Сестры" группы
"Самое большое простое число".

Источник:
   1. https://nplus1.ru/news/2018/01/05/spb4
   2. https://db.chgk.info/question/ufach08.1/3
   3. https://www.youtube.com/watch?v=uL0yiGrvA4Y

Автор:
Дмитрий Тарарыков (Дубровский)

Вопрос 16:
Согласно японской легенде, у НЕЕ сначала были ноги и панцирь. Но за то,
что ОНА не справилась с порученным ей заданием, владыка моря приказал ее
жестоко избить. Другая "ОНА" была создана в 2014 году. Назовите ЕЕ.

Ответ:
Медуза.

Комментарий:
По легенде, изначально у медузы были кости и панцирь, но за провинность
ее так избили, что и до сегодняшнего дня все медузы похожи на желе.
Интернет-издание "Meduza" было запущено в 2014 году.

Источник:
   1. Хэдленд Дэвис. Мифы и легенды Японии.
https://books.google.ru/books?id=J9fRw3xLznEC&pg=PT277#v=onepage&q&f=false
   2. https://ru.wikipedia.org/wiki/Meduza

Автор:
Сергей Терентьев (Санкт-Петербург)

Вопрос 17:
Этот человек учился в той же школе, что и его тезка. За маленький рост,
темные волосы и характерный нос друзья называли его "маленький еврей".
Позже он взял имя Рикардо Клементо. Назовите этого человека.

Ответ:
[Адольф] Эйхман.

Комментарий:
Адольф Эйхман - немецкий офицер, непосредственно отвечавший за массовое
уничтожение евреев. Он учился в школе, в которой незадолго до этого
учился Гитлер. После войны Эйхман скрылся в Южной Америке и взял себе
другое имя. Но агенты израильской разведки "Моссад" выследили его,
похитили, вывезли в Израиль, судили и казнили.

Источник:
   1. https://ru.wikipedia.org/wiki/Эйхман,_Адольф
   2. http://www.peoples.ru/state/criminal/killer/adolf_eichmann/index1.html

Автор:
Сергей Терентьев (Санкт-Петербург)

Вопрос 18:
Дуплет, два вопроса, по тридцать секунд обсуждения на каждый. Смотрите,
не перепутайте ответы!
   1. Чьей части тела Маяковский уподобил провалившуюся улицу?
   2. Какое прозвище носил наркоторговец-коллекционер из романа
Пелевина?

Ответ:
   1. Сифилитика.
   2. Филателист.

Комментарий:
   1. У классика "Улица провалилась, как нос сифилитика".
   2. Специалиста по маркам из романа "Generation П" логично называют
Филателистом.
   Героиня известного анекдота нередко путала эти два понятия.

Источник:
   1. В.В. Маяковский. А все-таки.
http://www.feb-web.ru/feb/mayakovsky/texts/ms0/ms1/ms1-062-.htm
   2. В.О. Пелевин. Generation П.
http://pelevin.nov.ru/romans/pe-genp/9.html
   3. https://www.anekdot.ru/id/-421900018/

Автор:
Антон Волосатов (Ивантеевка)

Вопрос 19:
В одной из глиняных табличек Ирвинг Финкель нашел слово со значением
"порно", что, по его мнению, является доказательством заимствования
одного сюжета в Библии. В одном из слов предыдущего предложения мы
пропустили две буквы. Напишите это слово целиком.

Ответ:
Попарно.

Комментарий:
Благодаря этому слову можно понять, что животные входили на вавилонский
ковчег попарно, точно так же, как и на Ноев ковчег, что было неизвестной
доселе вавилонской традицией.

Источник:
Ирвинг Финкель. Ковчег до Ноя: от Междуречья до Арарата.

Автор:
Сергей Терентьев (Санкт-Петербург)

Вопрос 20:
Переводчик книги Клаудио Раньери употребил выражение "окрестил Н'Голо
Канте осьминогом". Критик перевода, отмечая, что Канте - мусульманин,
приводит аналогию - это всё равно что сказать, что Гитлер написал ИКС
под названием "Mein Kampf" [майн кампф]. Назовите ИКС.

Ответ:
Талмуд.

Комментарий:
Талмуд - свод правил, регламентирующих религиозные, правовые отношения и
быт верующих евреев. Талмудом образно называют объемные книги.

Источник:
https://www.sports.ru/tribuna/blogs/slava22/1409312.html

Автор:
Сергей Терентьев (Санкт-Петербург), Роман Цуркан (Воронеж)

Вопрос 21:
У Дэвида Диксона была дочь от связи с рабыней. Для достижения некоего
результата он ДЕЛАЛ с дочерью ЭТО. Одна новоявленная мать совершала
аналогичное действие в связи с переизбытком. Что такое "ДЕЛАТЬ ЭТО"?

Ответ:
Купать в молоке.

Зачет:
Синонимичные ответы с упоминанием молока.

Комментарий:
Диксон считал, что купание в коровьем молоке может отбелить родившегося
чернокожего ребенка. У одной женщины после рождения выделялось так много
молока, что она купала своего ребенка в нем.

Источник:
   1. Элизабет Эбботт. История куртизанок.
   2. https://www.kleo.ru/items/news/2009/03/16/br_f.shtml

Автор:
Сергей Терентьев (Санкт-Петербург)

Вопрос 22:
   <раздатка>
   ДРОВОСЕК
   </раздатка>
   Перед вами часть менее используемого слова для обозначения понятия,
появившегося в XXI веке. Напишите четыре пропущенные на раздаточном
материале буквы.

Ответ:
СУАЛ.

Комментарий:
Слово-бумажник "ламберсексуал" составлено из слов "lumberjack"
[ламберджек] - лесоруб и "metrosexual" [метросексуал]. Менее
распространенное слово для обозначения этого понятия составлено из слов
"дровосек" и "метросексуал".

Источник:
https://ru.wikipedia.org/wiki/Ламберсексуал

Автор:
Сергей Терентьев (Санкт-Петербург)

Вопрос 23:
Сторонники одного из вариантов расположения ЕЕ говорят, что их вариант
лучше при использовании в доме на колесах, который часто передвигается.
Назовите ЕЕ двумя словами.

Ответ:
Туалетная бумага.

Комментарий:
Существуют сторонники двух вариантов расположения бумаги - бумага из
висит перед рулоном (к себе) или за ним (от себя). При варианте
расположения "от себя" вероятность разматывания рулона в доме на колесах
меньше.

Источник:
https://ru.wikipedia.org/wiki/Туалетная_бумага

Автор:
Сергей Терентьев (Санкт-Петербург)

Вопрос 24:
Александр Павлов пишет, что, когда в 2009 году была представлена "ОНА",
слух об этом как о самом неприятном зрелище стал быстро передаваться из
уст в уста. Назовите "ЕЕ" двумя словами.

Ответ:
"Человеческая многоножка".

Комментарий:
"Человеческая многоножка" считается одним из самых отвратительных
фильмов в истории кинематографа. После выхода этого фильма слухи о нем
быстро распространялись путем передачи информации из уст в уста, а не
так, как были сшиты его персонажи.

Источник:
Александр Павлов. Расскажите вашим детям. Сто одиннадцать опытов о
культовом кинематографе.

Автор:
Сергей Терентьев (Санкт-Петербург)

Тур:
20 тур. "Львята отца Симбы" (Северодвинск)

Редактор:
Валерия Комаровская

Вопрос 1:
Блок 1. Вопросы о кино.
   Начнем с "детского" вопроса.
   В вышедшем в 2017 году фильме "Человек-паук: Возвращение домой" у
человека-паука новый костюм, который специально для него спроектировал
Тони Старк. В силу возраста героя Старк заблокировал часть возможностей
костюма. К примеру, когда Питер Паркер в костюме, за ним ведется
постоянное видео- и аудионаблюдение, и протокол, ответственный за это,
называется "радионяня". Назовите одним словом протокол, ответственный за
блокировку боевых функций костюма и предназначенный обезопасить Питера
от самого себя.

Ответ:
Ходунки.

Источник:
Х/ф "Человек-паук: Возвращение домой" (2017), реж. Джон Уоттс.

Автор:
Алексей Нехлебаев

Вопрос 2:
Внимание, в вопросе есть замены.
   Однажды известный кинокритик Роджер Эберт попросил
режиссера-документалиста Эррола Морриса четко обозначить разницу между
документальным и игровым кино. На что тот ответил: "Ну это просто. Два
ИКСА". У американской группы "Glassjaw" [глАссджо] есть песня о
расставании под названием "Когда одна АЛЬФА становится двумя ИКСАМИ". Мы
не спрашиваем, что мы заменили АЛЬФОЙ. Ответьте, что было заменено
ИКСОМ.

Ответ:
Ноль.

Зачет:
0.

Комментарий:
Эррол Моррис имел в виду разницу в бюджетах документального и игрового
кино. АЛЬФА - восьмерка.

Источник:
   1. http://flavorwire.com/611411/errol-morris-on-creating-the-everything-bagel-of-his-new-netflix-series-wormwood
   2. https://www.discogs.com/Glassjaw-Everything-You-Ever-Wanted-To-Know-About-Silence/master/208380

Автор:
Сергей Никулин

Вопрос 3:
На одном меме, посвященном игре "Heroes of Might and Magic III",
изображен монитор компьютера с окном внутреннего вида города одного из
классов. Бюст известного человека выглядывает из-за монитора и советует
построить здание, позволяющее нанимать юнитов пятого уровня. В фильме
начала 1990-х годов бюст этого человека с помощью небольшого количества
гипса переделывают в бюст другой известной личности. Догадавшись, на
что, в основном, пошел гипс при переделке, назовите этот фильм.

Ответ:
"Бакенбарды".

Комментарий:
В фильме Юрия Мамина "Бакенбарды" из бюста Ленина делали бюст Пушкина
при помощи небольшого количества гипса, который пошел на бакенбарды.

Источник:
   1. http://joyreactor.cc/post/3024024
   2. https://ru.wikipedia.org/wiki/Бакенбарды_(фильм)

Автор:
Алексей Нехлебаев

Вопрос 4:
В вопросе есть замены.
   Антагониста Человека-паука, ученого, который в результате аварии
слился с механизмом, можно назвать ИМ. Многие считают, что на логотипе
известной по комиксам организации изображен ОН, хотя на самом деле -
ОНА. ОН и ОНА предпочитают одну и ту же среду. Назовите ЕГО и ЕЕ в любом
порядке.

Ответ:
Осьминог, гидра.

Зачет:
В любом порядке.

Комментарий:
Доктор Осьминог (Доктор Отто Гюнтер Октавиус) - высокоинтеллектуальный
злой ученый, заклятый враг Человека-паука и один из умнейших персонажей
вселенной Marvel. На его спине располагаются четыре мощных механических
щупальца. "Гидра" - террористическая организация в комиксах издательства
"Marvel Comics".

Источник:
   1. https://ru.wikipedia.org/wiki/Доктор_Осьминог
   2. https://ru.wikipedia.org/wiki/Гидра_(Marvel_Comics)

Автор:
Алексей Нехлебаев

Вопрос 5:
Блок 2. Вопросы о книгах.
   В книге "Простаки за границей" Марк Твен зачастую нелестно отзывается
о современных ему американских реалиях. Так, восхищаясь стойкостью
римского солдата во время извержения в Помпее, писатель сравнивает того
с полицейским: "Он был солдатом - и остался на посту, потому что
доблесть воина мешала ему обратиться в бегство. Будь он полицейским, он
тоже остался бы - потому что...". Закончите мысль Твена как можно
точнее.

Ответ:
"... спал бы крепким сном".

Зачет:
По смыслу.

Источник:
М. Твен. Простаки за границей. http://flibusta.is/b/389718/read

Автор:
Елена Ганькова

Вопрос 6:
Внимание, в вопросе есть замена.
   На встрече "Дилетантских чтений" в 2016 году Дмитрий Быков, вспомнив
теорию Михаила Эпштейна и заговорив о функциях материков, стран и, в
частности, России, добавил: "А кому не нравится такая стабильность,
поезжайте в другие части света". В предыдущем предложении мы заменили
одно слово. Восстановите его.

Ответ:
Тела.

Комментарий:
Михаил Эпштейн - философ и культуролог, большое внимание уделяющий
философии тела (см. одноименную книгу 2006 года). Развивая идеи
Эпштейна, Дмитрий Быков сказал, что мир - это организм, тело: Китай -
рабочие руки, США - мозг или сердце и т.п., а Россия - спина, которая
своей неподвижностью должна стабилизировать систему.

Источник:
   1. https://www.youtube.com/watch?v=hpIpjmYws_0&t=25m06s
   2. https://ru.wikipedia.org/wiki/Эпштейн,_Михаил_Наумович

Автор:
Елена Ганькова

Вопрос 7:
В серии книг "Плоский мир" Терри Пратчетта тролли - вымышленная раса,
кремниевая гуманоидная форма жизни. Один из троллей, Детрит,
устроившийся на службу в стражу, невероятно силен и, казалось бы, мог
быть идеальным стражником. Но на тренировке во время выполнения одного
из заданий он постоянно падает, так как не может рассчитать силу. Мы не
спрашиваем, в какую именно сторону он падает. Ответьте, чему именно
учатся в этот момент начинающие стражники.

Ответ:
Отдавать честь.

Зачет:
Козырять; брать под козырек.

Комментарий:
Тролль падает влево, так как, не рассчитав силы, бьет себя правой рукой
в висок. В принципе, вправо или влево - зависит от того, левша он или
правша. Это в книге не уточняется, поэтому для вопроса решающей роли не
играет.

Источник:
Т. Пратчетт. К оружию! К оружию! http://flibusta.is/b/75565/read

Автор:
Алексей Нехлебаев

Вопрос 8:
В вопросе есть замены.
   В серии книг Терри Пратчетта "Плоский мир" ОМЕГИ гильдии убийц делали
свое дело последними. Разбирая считалочку из другого произведения, в
которой также упоминаются ОМЕГИ, пользователь Живого Журнала устроил
настоящую экскурсию по одному из городов. Какому именно?

Ответ:
Лондон.

Комментарий:
ОМЕГИ - колокола (большая буква омега даже чем-то похожа на колокол). В
считалке из "1984" "Апельсинчики как мед, // В колокол Сент-Клемент
бьет..." упоминается множество церквей Лондона.

Источник:
   1. Т. Пратчетт. К оружию! К оружию! http://flibusta.is/b/75565/read
   2. https://markgrigorian.livejournal.com/857268.html

Автор:
Алексей Нехлебаев

Вопрос 9:
(pic: 20180320.jpg)
   Блок 3. Вопросы о котиках.
   Представленная вам картина британской художницы Сьюзен Герберт была
названа на сайте pikabu.ru неологизмом из девяти букв. Воспроизведите
его.

Ответ:
Котофелия.

Комментарий:
Сьюзен Герберт спародировала фрагмент известной картины Джона Эверетта
Милле "Офелия". Русскоязычный неологизм обыгрывает имя главной героини.

Источник:
   1. https://pikabu.ru/story/syuzen_gerbert_quotkotofeliyaquot_3954606
   2. http://www.dailymail.co.uk/news/article-3222637/From-Mona-Lisa-Lawrence-Arabia-famous-artworks-imagined-cats.html
   3. https://ru.wikipedia.org/wiki/Офелия_(картина_Милле)

Автор:
Елена Ганькова, Алексей Нехлебаев

Вопрос 10:
(pic: 20180321.jpg)
   Знакомый автора вопроса - Иван Петренко из города КостОполь
прокомментировал эту фотографию фразой: "Это же ОН!", и был совершенно
прав. Напишите абсолютно точно слово, которое мы заменили словом "ОН".

Ответ:
Кiт.

Комментарий:
КостОполь - украинский город, а на картинке кiт (кот по-украински), хоть
и похож частично на кита по-русски.

Источник:
ЛОАВ.

Автор:
Анна Дуплищева

Вопрос 11:
Прослушайте вторую и третью строки современного хокку:
   [...]
   Наружу выпала
   Пара драчливых котят
   Воспроизведите первую строчку, которая совпадает с названием
произведения 1963 года.

Ответ:
Колыбель для кошки.

Комментарий:
Роман Курта Воннегута.

Источник:
   1. https://www.stihi.ru/2016/09/26/4738/
   2. https://ru.wikipedia.org/wiki/Колыбель_для_кошки

Автор:
Елена Ганькова

Вопрос 12:
Блиц.
   У знакомой вам британской художницы Сьюзен Герберт есть серия кошек -
персонажей Шекспира. Этой серии и посвящен наш блиц.
   1. Назовите произведение, иллюстрация к которому представляет собой
серого кота, что-то нашептывающего на ухо черному.
   2. Назовите произведение, иллюстрация к которому изображает двух
практически одинаковых кота и кошку.
   3. Назовите коронованную особу, изображенную в виде кошки в окружении
множества нарядно одетых маленьких котят.

Ответ:
   1. "Отелло".
   2. "Двенадцатая ночь".
   3. Титания.

Комментарий:
   1. Серый кот Яго нашептывает на ухо черному коту - Отелло.
   2. На истории близнецов Виолы и Себастьяна построена "Двенадцатая
ночь" Шекспира.
   3. Титания - королева фей и эльфов в комедии "Сон в летнюю ночь".

Источник:
   1. http://rovdyrdreams.com/galereya-kartin-shekspirovskie-koshki-angliyskoy-hudozhnitsyi-syuzen-gerbert/
   2. https://ru.wikipedia.org/wiki/Отелло
   3. https://ru.wikipedia.org/wiki/Двенадцатая_ночь_(пьеса)
   4. https://ru.wikipedia.org/wiki/Титания_(персонаж)

Автор:
Елена Ганькова

Вопрос 13:
(pic: 20180322.jpg)
   Блок 4. Вопросы о картинах.
   В книге "30 величайших картин, которые потрясли мир" Жан-Луи Феррье
описывает "Портрет Карла V в сражении при Мюльберге" работы Тициана и
говорит о том, что картина очень строга, задние ноги лошади переводят
фигуру в галоп лишь условно, а впечатление движения создают только ОНИ.
Назовите ИХ двумя словами, лишь немного отличающимися друг от друга.

Ответ:
Помпоны попоны.

Источник:
Ж.-Л. Феррье. 30 величайших картин, которые потрясли мир. - М.: РИПОЛ
классик, 2005. - С. 90.

Автор:
Елена Ганькова

Вопрос 14:
В вопросе есть замена.
   При поиске кадров из мультфильма "Варежка" 1967 года автор вопроса
воспользовался системой "Яндекс", и в итоге - ссылкой на ТАКИЕ картинки.
Можно сказать, что одна из главных героинь упомянутого мультфильма -
ТАКАЯ. Что мы заменили словом "ТАКИЕ"?

Ответ:
Связанные.

Комментарий:
В Яндексе - связанные картинки. Главная героиня мультика "Варежка" -
варежка, которая, превращаясь в собачку, сохраняет свою вязаную фактуру.

Источник:
   1. https://yandex.ru/images/search?text=мультфильм%20варежка%20кадры
   2. https://ru.wikipedia.org/wiki/Варежка_(мультфильм)

Автор:
Елена Ганькова

Вопрос 15:
В клипе "Imitation of Life" группы "R.E.M." на протяжении почти четырех
минут показываются разные моменты вечеринки у бассейна, в которой
участвует порядка 40 человек. Создатели клипа хотели, чтобы у зрителя
сложилось ощущение, как будто он смотрит на картину. Поэтому клип
получился по-своему уникальным: для его создания были использованы всего
двадцать... Чего именно?

Ответ:
Секунд видео.

Зачет:
Секунд видеосъемки и т.п. по смыслу.

Комментарий:
20 секунд видео просто прокручивают туда-сюда, приближая отдельные
фрагменты, как при просмотре картины.

Источник:
https://arzamas.academy/micro/clip/16

Автор:
Алексей Нехлебаев

Вопрос 16:
В 2014 году студентка Христианского университета Оклахомы Амелия Хэмрик
изучила известное произведение искусства и записала "песню с задницы из
ада, которой исполнилось 500 лет". Андрей Зимоглядов охарактеризовал
часть этого произведения словами "Голые и смешные". Назовите его автора.

Ответ:
[Иероним] Босх.

Комментарий:
Центральная часть "Сада земных наслаждений" полна изображений обнаженных
людей в странных позах. А Амелия Хэмрик вдохновилась изображением
задницы с оттиснутыми на ней нотами на правой створке триптиха под
названием "Музыкальный ад".

Источник:
   1. https://cameralabs.org/11022-35-tajn-znamenitykh-kartin
   2. https://artchive.ru/publications/1619~Boskh_ego_znaet_glavnye_simvoly_Sada_zemnykh_naslazhdenij_krupnym_planom

Автор:
Елена Ганькова

Вопрос 17:
Блок 5. Вопросы о сериалах.
   В вопросе есть замены.
   В книге "Простаки за границей" Марк Твен описывает монастырь Святого
Саввы и пустынников, не покидавших его десятилетиями: "Они изгнали из
жизни всю ее прелесть и всю нежность, и, лишенная животворящих соков,
она превратилась в костлявую иссохшую карикатуру на самое себя. Уста их
никогда не знали ни поцелуя, ни песни; сердца не ведали ни ненависти, ни
любви... Это ОТЧАЯННЫЕ ДОМОХОЗЯЙКИ". В приведенной цитате мы заменили
два слова. Восстановите их.

Ответ:
Ходячие мертвецы.

Комментарий:
Название одного сериала заменено названием другого.

Источник:
М. Твен. Простаки за границей. http://flibusta.is/b/389718/read

Автор:
Елена Ганькова

Вопрос 18:
Сериал "Лучше звоните Солу" является спин-оффом сериала "Во все тяжкие"
и рассказывает, какая нелегкая жизнь привела Джеймса Макгила к карьере
продажного адвоката и пособника мафии под псевдонимом Сол Гудман. В
одной из серий подруга и коллега Макгила Ким узнаёт о том, что он
нарушил закон и у людей, собирающих против него улики, есть косвенные
доказательства. Встретившись с Макгилом, первым делом она требует у
Джеймса доллар. С какой целью?

Ответ:
Чтобы стать его адвокатом.

Зачет:
Гонорар за то, чтобы стать его адвокатом, и т.п. по смыслу.

Комментарий:
Доллар выступает гонораром и обозначает то, что между ними сложились
отношения адвокат-подзащитный. Адвокат не вправе разглашать информацию о
клиенте, и, следовательно, с этого момента Ким не смогла бы
свидетельствовать против друга.

Источник:
Телесериал "Лучше звоните Солу", s03e02, примерно 35-я минута.

Автор:
Алексей Нехлебаев

Вопрос 19:
В одном из сезонов сериала "Агенты Щ.И.Т." герои попадают в сложную
ситуацию: их сознание помещают в компьютерную имитацию, выход из
которой, как им сообщают, находится под одним из сооружений в парке
развлечений. Однако на месте нужного сооружения герои находят лишь
здание завода и решают, что возможности покинуть программу нет. Но одна
из героинь, использовав суперспособности, уподобляется библейскому
пророку и помогает друзьям обрести свободу. Назовите персонажа другого
фильма, который покинул наш мир при сходных обстоятельствах и, судя по
посланному им сигналу, проделал это успешно.

Ответ:
Терминатор.

Зачет:
Т-800.

Комментарий:
Выход находится на заводе на дне чана с расплавленным металлом. Героиня
раздвинула жидкий металл, почти как Моисей раздвинул море. Терминатор в
конце фильма "Терминатор-2: Судный день" тонет в жидком металле.

Источник:
   1. Телесериал "Агенты Щ.И.Т.", s03e20.
   2. https://ru.wikipedia.org/wiki/T-800_(робот)

Автор:
Алексей Нехлебаев

Вопрос 20:
Во время чукотско-корякских войн коряки с трудом оборонялись от
воинственных чукч и, используя подручные средства, превращали свои
яранги в крепости. После этих манипуляций ярангу можно было назвать так
же, как роман и картину XIX века. На этой картине изображены, в том
числе, ОНИ. В более позднем произведении прозвище одного из НИХ стало
боевым кличем. Напишите это прозвище.

Ответ:
Полумуж.

Комментарий:
На яранги лили воду, и получался практически ледяной дом. "Ледяной дом"
- это роман Ивана Лажечникова 1835 года, а также картина Валерия Якоби
1878 года, на которой изображены несколько карликов. Самый популярный
карлик Вестероса - Тирион Ланистер, которого прозвали полумужем.

Источник:
   1. http://www.p-w-w.org/index.php?topic=8807.0
   2. https://ru.wikipedia.org/wiki/Ледяной_дом
   3. Телесериал "Игра престолов", s01e09, примерно 44-я минута.
   4. https://ru.wikipedia.org/wiki/Тирион_Ланнистер

Автор:
Алексей Нехлебаев

Вопрос 21:
(pic: 20180323.jpg)
   Блок 6. Вопросы о СМИ.
   Во всемирно известных телевизионных "Битвах роботов" 1990-х - 2000-х
годов участвовали различные виды роботов. Перед вами несколько из них:
флиппер - робот с пневматическим опрокидывателем, возвращающийся в
исходное положение; робот - гидравлические ножницы, способный резать
металл. Назовите одним словом третьего робота.

Ответ:
Спиннер.

Комментарий:
Как видно, одна из отличительных особенностей третьего робота -
вращение. А раздаточный материал по форме отдаленно напоминает игрушку
спиннер.

Источник:
   1. Телепередача "Битвы роботов".
   2. https://ru.wikipedia.org/wiki/Битвы_роботов
   3. https://geektimes.ru/post/294857/

Автор:
Анна Дуплищева, Елена Ганькова

Вопрос 22:
[Ведущему: четко прочитать слово "понравилась".]
   Анекдот.
   Спортивный комментатор берет интервью у нашего лыжника, только что
сошедшего с дистанции:
   - Вы, конечно, не победили, но скажите: вам понравилась [ДВА СЛОВА
ПРОПУЩЕНО]?
   - [ОДНО СЛОВО ПРОПУЩЕНО] понравилась, поэтому и не победили.
   Восстановите любой из пропусков.

Ответ:
Сама гонка.

Зачет:
Самогонка.

Источник:
https://www.anekdot.ru/id/494176/

Автор:
Анна Дуплищева

Вопрос 23:
Герой современного детектива Сергей Бабкин - бывший оперативник и
частный детектив - обнаруживает неожиданный бытовой талант и удивляет
напарника, который спрашивает: "Может, зря ты опером стал, Серёга?...
Выигрывал бы конкурсы у НЕГО..." "Что за чувак? ... Египетского
разлива?" - недоумевает Сергей. Назовите того, с кем, по мнению
напарника, мог бы успешно соперничать Бабкин.

Ответ:
[Гордон] Рамзи.

Комментарий:
Детектив обладал выдающимися, по мнению напарника, кулинарными
способностями, но не знал, кто такой Гордон Рамзи - знаменитый
британский шеф-повар. Фамилия "Рамзи" напомнила ему лишь о Рамзесе -
имени целого ряда египетских фараонов.

Источник:
   1. Е. Михалкова. Нежные листья, ядовитые корни.
https://books.google.ru/books?id=wjKACgAAQBAJ&pg=PT182#v=onepage&q&f=false
   2. https://ru.wikipedia.org/wiki/Рамзи,_Гордон
   3. https://ru.wikipedia.org/wiki/Рамзес

Автор:
Елена Ганькова

Вопрос 24:
Закончить наш пакет мы хотим также "детским" вопросом.
   Во время первой игры детского КВН на канале "Карусель" ведущие
предположили: то ли очередные участники - фанаты известной московской
команды "Парапапарам", то ли очень прилежные ученики. Воспроизведите
название этой детской команды, состоящее из трех слов.

Ответ:
"Пора по партам".

Источник:
   1. Детский КВН - 2017, 1-я игра.
https://www.youtube.com/watch?v=wnr3CN5A1_4&t=24m27s
   2. https://ru.wikipedia.org/wiki/Парапапарам

Автор:
Елена Ганькова

Тур:
21 тур. "Кого бог послал" (Серпухов)

Редактор:
Гай Кузнецов

Вопрос 1:
Забавно, что АЛЬФА часто становится поставщиком новостей для желтой
прессы. Забавно, что в ЖЖ есть статья "АЛЬФА покрылась ржавчиной".
Назовите АЛЬФУ двумя словами.

Ответ:
Золотая молодежь.

Источник:
   1. https://www.newsru.com/russia/12jul2017/medal.html
   2. https://berni777.livejournal.com/584650.html

Автор:
Олег Холодов

Вопрос 2:
Герою Олега Дивова, спортсмену, в период расцвета его спортивной карьеры
всё время не хватает самой малости для победы. Прозвище, которое получил
этот герой, совпадает с названием произведения русской литературы.
Напишите это прозвище.

Ответ:
"Князь Серебряный".

Комментарий:
"Я из того периода вывез прозвище "Князь Серебряный" и паршивую
репутацию лыжника, который дальше второго места продвинуться хочет, но
не может".

Источник:
   1. О. Дивов. Толкование сновидений.
https://books.google.ru/books?id=xYieAAAAQBAJ&pg=PT54#v=onepage&q&f=false
   2. https://ru.wikipedia.org/wiki/Князь_Серебряный

Автор:
Павел Семёнов

Вопрос 3:
В 2015 году в Тарусе был открыт памятник поэту Николаю Заболоцкому.
Статья, посвященная этому событию, называлась "Первый из поэтов".
Эпитет, пропущенный в этом заголовке, входит в термин, предложенный
ленинградским поэтом Олегом Охапкиным. Напишите это понятие, состоящее
из двух слов.

Ответ:
"Бронзовый век".

Комментарий:
"Золотой век" русской поэзии - пушкинский, лермонтовский - у всех на
слуху. "Серебряный век", к которому Таруса имеет прямое отношение, тоже
всем известен. К "Бронзовому веку" относятся поэты, которые родились во
время войны - дети фронтовиков.

Источник:
https://svpressa.ru/culture/article/127398/

Автор:
Олег Холодов

Вопрос 4:
Интернет-магазин "Lakot" предлагает ИХ для выпускников всего по 50
рублей. По мнению Кристины Королёвой, игры в Ванкувере прошли для России
под знаком ИХ. Назовите ИХ двумя словами.

Ответ:
Деревянные медали.

Источник:
   1. http://lakot.ru/collection/medali-dlya-shkol
   2. https://www.championat.com/other/article-50300-den-vosmoj-pod-znakom-derevjannykh-medalej.html

Автор:
Олег Холодов

Вопрос 5:
Комик Энди Кауфман знаменит в том числе пародиями на Элвиса Пресли.
Назовите тремя словами группу, написавшую посвященную Энди Кауфману
песню.

Ответ:
"Король и Шут".

Комментарий:
Про Энди можно сказать, что он шут, пародировавший короля.

Источник:
   1. https://ru.wikipedia.org/wiki/Кауфман,_Энди
   2. https://ru.wikipedia.org/wiki/Театр_демона

Автор:
Александр Волков

Вопрос 6:
Выбранное название первоначально объяснялось любовью к группе "Byrds", и
только позже появились утверждения, что ОНИ символизируют Америку. А
какое заведение при ИХ упоминании приходит на ум в первую очередь?

Ответ:
Отель "Калифорния".

Комментарий:
Речь идет о группе "Eagles" - орлы. Sea eagle - это орлан, один из
американских символов.

Источник:
http://artrockcafe.ucoz.com/forum/20-29-1

Автор:
Гай Кузнецов

Вопрос 7:
Герой спектакля Театра Сатиры "Маленькие комедии большого дома"
описывает старинное канапе: "Раньше оно стояло в барской усадьбе. Так
что в его жилах течет дворянская кровь". Какое слово мы заменили в этом
вопросе?

Ответ:
Клопах.

Источник:
Спектакль Театра Сатиры "Маленькие комедии большого дома".

Автор:
Юрий Гришанов

Вопрос 8:
ЕЕ прозвище - "жалящая пчелка". ОНА родилась в 1977 году. Назовите ЕЕ
имя и фамилию.

Ответ:
Лейла Али.

Комментарий:
По крайней мере, половину прозвища унаследовала от отца.

Источник:
https://ru.wikipedia.org/wiki/Али,_Лейла

Автор:
Гай Кузнецов

Вопрос 9:
Несмотря на всю завесу секретности, окончательный уточненный показатель
106 стал известен уже в начале мая... Какого года?

Ответ:
1961.

Комментарий:
Речь идет о продолжительности полета Юрия Гагарина. Считается, что он
длился 108 минут, ан нет.

Источник:
http://megabook.ru/article/Гагарин%20Юрий%20Алексеевич

Автор:
Гай Кузнецов

Вопрос 10:
Расположенная на стыке Большого Казённого, Малого Казённого, Лялина и
Барашевского переулков Лялина площадь неофициально считается московским
аналогом ИХ. Назовите ИХ двумя словами.

Ответ:
Пять углов.

Источник:
http://caoinform.ru/miniatyurnaya-ploshhad-pyati-uglov/

Автор:
Гай Кузнецов

Вопрос 11:
В романе Антона Демченко "Небесный бродяга" жители Новгорода кучеров
называли "ваньками" - от имени Иван. Прозвище же таксистов восходит к
другому имени, которым иногда называют ЕЕ. Назовите ЕЕ двумя словами.

Ответ:
Электронная почта.

Комментарий:
Таксистов в Новгороде называли "емельками" - очевидно, от Емели, который
ездил на печи.

Источник:
А. Демченко. Небесный бродяга.
https://books.google.ru/books?id=gf8KDgAAQBAJ&pg=PT151#v=onepage&q&f=false

Автор:
Олег Холодов, Гай Кузнецов

Вопрос 12:
Дуплет.
   1. В каком положении находится суверенитет России, по мнению
Задорнова?
   2. В каком положении находится суверенитет Украины, по мнению
Задорнова?

Ответ:
   1. Висит.
   2. Лежит.

Комментарий:
В России - независимость, в Украине - незалежность.

Источник:
???

Автор:
Елена Батова

Вопрос 13:
Согласно одному анекдоту, владельцы собак ДЕЛАЮТ ЭТО чаще остальных
мужчин. ДЕЛАТЬ ЭТО вперед, согласно словарю русских поговорок, -
разузнавать о чем-то. Назовите героя, который любил ДЕЛАТЬ ЭТО в
одиночку и с друзьями.

Ответ:
Винни-Пух.

Комментарий:
Известно, что Винни-Пух любил играть в игру "Пустяки", суть которой
состояла в том, что игроки кидали палочки с моста в реку. Чья палочка
раньше выплывет с другой стороны моста, тот и победил.

Источник:
   1. Анекдот.
   2. https://dic.academic.ru/dic.nsf/proverbs/35295/
   3. https://arzamas.academy/materials/1212

Автор:
Олег Холодов

Вопрос 14:
Бен Ладен утверждал, что он не террорист, а просто влияет на ход истории
путем создания искусственных, запланированных кризисов - естественно, к
своей выгоде. По утверждению биографов, данную идею он позаимствовал,
прочитав знаменитый роман, который на арабском языке издавался под
названием "Аль-Каида". Назовите автора этого романа.

Ответ:
Айзек Азимов.

Комментарий:
Роман "Основание". "Аль-Каида" можно перевести как "основа".

Источник:
https://fantlab.ru/work7

Автор:
Юрий Гришанов

Вопрос 15:
Недавно организаторы решили устранить "несправедливость" и заново
пересмотрели итоги работы в период с середины 1950-х до середины 1990-х.
В результате был сделан вывод, что 12 раз их предшественники ошиблись.
Назовите любого из граждан СССР, которых этот пересмотр затронул
напрямую.

Ответ:
Лев Яшин.

Зачет:
Игорь Беланов.

Комментарий:
Французы пересмотрели результаты выборов лучшего игрока Европы, начиная
с 1956 года, когда была вручена первая премия, вплоть до 1995 года,
когда "Золотой мяч" получили возможность выиграть футболисты с любого
континента, при условии, что они играют в европейском чемпионате. Приз
Льва Ивановича виртуально отошел Пеле (всего у Пеле было бы семь
наград), Беланова "подсидел" Марадона.

Источник:
https://www.sovsport.ru/football/articles/946435-u-lva-jashina-otobrali-zolotoj-mjach-a-my-i-ne-zametili

Автор:
Алексей Паклин

Вопрос 16:
Послушайте сообщение: "Оксфордский словарь выбрал неологизм
"вспомнитьвсё", отражающий наиболее важные изменения, произошедшие в
мире в 2016 году, словом года". Предлагаем вам распространить данное
сообщение посредством ваших аккаунтов в соцсетях, блоге или ЖЖ. Какое
слово мы заменили в этом вопросе?

Ответ:
Постправда.

Зачет:
Послеправда; альтернативнаяправда (только слитно); постистина.

Источник:
   1. http://www.vokrugsveta.ru/news/263843
   2. https://en.oxforddictionaries.com/word-of-the-year/word-of-the-year-2016

Автор:
???

Вопрос 17:
Внимание, в вопросе есть замены.
   На следствии по делу декабристов Николая I крайне занимал вопрос,
почему во время бунта солдаты кричали: "Да здравствует Император и
АЛЬФА!". Какое якобы имя мы заменили АЛЬФОЙ?

Ответ:
Конституция.

Комментарий:
Константин был женат на польке, и никто точно не знал, как ее зовут.
Рылеев ради большего эффекта придумал и убедил солдат, что ее зовут
Конституция.

Источник:
https://www.youtube.com/watch?v=_jSZL2lVxiA

Автор:
Александр Волков

Вопрос 18:
В русском переводе мультфильма "В поисках Немо" можно услышать диалог
между двумя друзьями-рыбками:
   - Посмотри, какая она красавица, вот бы мне такую подружку...
   - Да, она даже была на НЕЙ.
   Назовите ЕЕ тремя словами, два из которых - англоязычное название.

Ответ:
Обложка "National Geographic".

Источник:
Мультфильм "В поисках Немо".

Автор:
Ася Баранова

Вопрос 19:
Википедия утверждает, что при лечении некоторых депрессий практикуют
отказ от ИКСА. У Джанни Родари есть рассказ, в котором робота за ИКС
приговорили к двум неделям тюрьмы. Назовите ИКС.

Ответ:
Сон.

Источник:
   1. https://ru.wikipedia.org/wiki/Депривация_сна
   2. Дж. Родари. Робот, которому захотелось спать.
http://flibusta.is/b/46498/read

Автор:
Павел Семёнов, Олег Холодов

Вопрос 20:
Выбор Марины Энн Хэнцис объясняется тем, что шкала сексуальной
ориентации, созданная Альфредом Кинси, выражается оттенками серого. По
другой версии, всё дело в ЕГО ненасытном стремлении к получению
удовольствий. Назовите ЕГО имя и фамилию.

Ответ:
Дориан Грей.

Комментарий:
Марина Энн Хэнцис - настоящее имя Саши Грей.

Источник:
https://ru.wikipedia.org/wiki/Саша_Грей

Автор:
Гай Кузнецов

Вопрос 21:
Дуплет.
   1. По замечанию Александра Цекало, на штрафы, собранные в
Белгородской области за нецензурную брань, можно построить ТАКУЮ
библиотеку. Какое слово мы заменили словом "ТАКАЯ"?
   2. На сайте магазина "IKEA" автор вопроса, как ни странно, не смог
найти ЭТАКУЮ кровать, хотя в ряде других фирм подобный заказ исполнят.
Какое слово мы заменили словом "ЭТАКАЯ"?

Ответ:
   1. Трехэтажная.
   2. Трехспальная.

Комментарий:
Шведская семья, шведский магазин...

Источник:
   1. https://www.1tv.ru/shows/prozhektorperishilton/vypuski/prozhektorperishilton-vypusk-ot-15-11-2008
(19:50)
   2. http://www.ikea.com/ru/ru/search/?query=трехспальная
   3. https://mo.tiu.ru/Trehspalnaya-krovat.html

Автор:
Гай Кузнецов, Александр Волков

Вопрос 22:
Соревнования в играх "Team Fortress 2", программа обучения детей
плаванию, дизель-генератор Atlas Copco модель QAS30, "Жигулёвская
кругосветка", фильм с участием Трита Уильямса и Ким Кэтролл. Уважаемые
знатоки! Дополните список еще одним событием.

Ответ:
Интеллектуальный марафон "36 часов NON STOP".

Зачет:
Марафон в Серпухове; "36 часов NON STOP".

Комментарий:
Соревнования идут 36 часов, Программа называлась "Плавать за 36 часов",
запас хода у генератора - 36 часов, Контрольное время кругосветки - 36
часов, фильм - "36 часов".

Источник:
   1. https://wiki.teamfortress.com/wiki/Tip_of_the_Hats
   2. http://www.swimming.khv.ru/news_full.php?nid=643
   3. http://www.elfast.ru/product_info.php?cPath=0_22_26&products_id=100
   4. http://news.samaratoday.ru/news/146316/
   5. https://www.kinopoisk.ru/film/94159/

Автор:
Елена Батова

Вопрос 23:
Рассказывают, что всем новым знакомым Анна Ахматова задавала три вопроса
об их предпочтениях: Кофе или чай? Кошка или собака? Сельдерей или
кешью? Какие два слова мы заменили в этом вопросе?

Ответ:
Пастернак и Мандельштам.

Комментарий:
Сельдерей - "глава" семейства, к которому относится и пастернак. Кешью и
миндаль - орехи ("Mandel" значит "миндаль").

Источник:
   1. Д.Л. Быков. Борис Пастернак. http://flibusta.is/b/260470/read
   2. https://www.liveinternet.ru/users/taris45/post423187949/

Автор:
Алексей Паклин

Вопрос 24:
Элвис Пресли щедро раздаривал друзьям драгоценности, при этом мужчинам
доставались изделия с гравировкой "TCB", что означало "Taking Care of
Business"; в женском же варианте фигурировала аббревиатура "TLC", где C
также расшифровывалось как "Care". А что означали буквы T и L?

Ответ:
Tender, Loving.

Комментарий:
Т.е. "Нежность, любовь, забота". На ответ может навести одна из самых
известных композиций Элвиса - "Love me tender".

Источник:
http://parallelnyj-mir.com/2/8228-kolco-presli.html

Автор:
Гай Кузнецов

Тур:
22 тур. "ЗаПущинная наука" (Пущино)

Редактор:
Данила Аладин

Вопрос 1:
В китайском языке в одном из вариантов обозначения России используются
три иероглифа: &#38470;&#35199;&#20122; - лу-си-я. Иероглифы означают:
АЛЬФА + запад + Азия. Антоним АЛЬФЫ входит в трилогию, за которую в
апреле 1980 года была присуждена Ленинская премия по литературе. Какие
два слова мы заменили АЛЬФОЙ?

Ответ:
Большая земля.

Комментарий:
"Малая земля", "Возрождение" и "Целина" - за эту трилогию в апреле 1980
года Брежневу была присуждена Ленинская премия по литературе.

Источник:
   1. https://magazeta.com/2009/11/new-eluosi/
   2. https://ru.wikipedia.org/wiki/Воспоминания_Брежнева

Автор:
Данила Аладин

Вопрос 2:
Википедия утверждает: то, что Аризона Донни Баркер была в курсе
преступлений, совершавшихся ее сыновьями, и, возможно, даже как-то
помогала им, не подлежит сомнению, но большинство специалистов по
криминальной истории США считают мифом ее руководство всеми операциями
банды. На сайте "ВикиФур" - русскоязычной фурри-энциклопедии
утверждается, что именно она послужила прототипом персонажа вселенной
Уолта Диснея. Назовите фамилию этого персонажа.

Ответ:
Гавс.

Зачет:
Beagle.

Комментарий:
Мамаша Гавс в мультсериале "Утиные истории" выступает в роли главаря
семейной преступной банды.

Источник:
   1. https://ru.wikipedia.org/wiki/Мамаша_Баркер
   2. http://ru.wikifur.com/wiki/Братья_Гавс

Автор:
???

Вопрос 3:
Дуплет.
   В серии "Марафонец" мультсериала "Смешарики" герой Бараш, страдая от
творческого кризиса, решил заняться бегом.
   1. Какое мебельное изделие появляется первым в этой серии?
   2. Бейсболку какого цвета носит Бараш в этой серии?

Ответ:
   1. Скамейка.
   2. Красного.

Источник:
Мультсериал "Смешарики", серия "Марафонец".
https://www.youtube.com/watch?v=hfynGYboSCE

Автор:
Данила Аладин

Вопрос 4:
В телесериале "Троцкий" описываются в том числе события Октябрьской
революции. Ответьте с точностью до минуты: когда Лейба Давыдович
Бронштейн отдал приказ об аресте Временного правительства?

Ответ:
7:40.

Источник:
Телесериал "Троцкий", 5-я серия.
https://www.youtube.com/watch?v=qIC6oxfBB4w&t=33m08s

Автор:
Данила Аладин

Вопрос 5:
Внимание, в вопросе одно слово заменили двумя.
   Герой американского сериала, описывая начальницу, говорит: "Страшная
баба! Когда Чайковский писал "Спящую красавицу", он, наверное, ее имел в
виду". Напишите замененное слово.

Ответ:
Щелкунчик.

Комментарий:
На английском "Щелкунчик" - "Nutcracker". Словом "Nuts" на сленге
обозначают тестикулы.

Источник:
Телесериал "Голова Германа", s01e18.

Автор:
Данила Аладин

Вопрос 6:
По одной из версий, этот киберспортивный термин, появившийся в
мультиплеере игры "Doom", где игрок противостоит не монстрам, а таким же
десантникам, как он сам, произошел от осколочной гранаты, т.к. от ее
осколков могли погибнуть и свои же солдаты. Часто это выдавалось за
несчастный случай или за боевые потери. Назовите этот термин.

Ответ:
Фраг.

Зачет:
Frag.

Комментарий:
Fragmentation grenade - осколочная граната. Произошел термин от
англоязычного глагола "to frag", означающего "поражать осколками".
Согласно сюжету игры "Doom", главный герой - космодесантник, сражающийся
с толпой монстров. В сетевом же режиме противниками являются такие же
десантники - видимо, поэтому очки, начисляемые за убийство противников,
было решено назвать фрагами.

Источник:
   1. https://ru.wikipedia.org/wiki/Фрэггинг
   2. https://ru.wikipedia.org/wiki/Фраг

Автор:
Данила Аладин

Вопрос 7:
Внимание, в вопросе есть замены.
   В видеоролике "Итальянцы пробуют русские лечебные средства"
итальянцы, узнав, из чего делается ОН, говорят: "Закат, ребята, не в
Америке, а в России!". Какое слово мы заменили словом "закат"?

Ответ:
Сумерки.

Комментарий:
Состав гематогена искренне поразил иностранцев.

Источник:
https://www.youtube.com/watch?v=ysmpP0l9by8&t=1m46s

Автор:
Данила Аладин

Вопрос 8:
Чтобы построить филогенетическое дерево некультивируемых микроорганизмов
- симбионтов позвоночных животных, биоинформатики использовали образцы
ДНК - с одной стороны, собранные в Московском зоопарке, а с другой
стороны, найденные в вечной мерзлоте Сибири. Назовите субстанцию из
Сибири, почтенный возраст которой должен был помочь в изучении эволюции
прокариот.

Ответ:
Г-но мамонта.

Источник:
???

Автор:
Евгения Крутинина

Вопрос 9:
(pic: 20180324.jpg)
   Не так давно одна американка выпустила свою линейку духов. Назовите
ее имя и фамилию, которые мы от вас скрыли.

Ответ:
Ким Кардашьян.

Комментарий:
Буква Ж - вовсе не буква Ж, а две буквы К. Хотя все знают, какая у Ким
Ж.

Источник:
https://pikabu.ru/story/logotip_dukhov_kim_kardashyan_3063368

Автор:
Данила Аладин

Вопрос 10:
(pic: 20180325.jpg)
   Перед вами изображение модификации танка Т-62М 1983 года. Ответьте
двумя словами, которые начинаются на одни и те же две буквы: какое
неофициальное название получили два противокумулятивных экрана на башне
этого танка?

Ответ:
Брови Брежнева.

Источник:
   1. https://militaryarms.ru/voennaya-texnika/tanki/t-62-2/
   2. http://btvt.narod.ru/5/t62m_mv/t62m_mv.htm

Автор:
Данила Аладин

Вопрос 11:
(pic: 20180326.jpg)
   Ответьте абсолютно точно: какое наречие мы от вас скрыли?

Ответ:
Периодически.

Комментарий:
Периодическая таблица Д.И. Менделеева.

Источник:
http://joyreactor.cc/post/3119098

Автор:
Данила Аладин

Вопрос 12:
(pic: 20180327.jpg)
   Блиц.
   1. В какой стране родился актер, скрытый над надписью "Паспорт РФ"?
   2. В какой стране родился актер, скрытый над надписью "Мед.книжка",
который в 2011 году был включен в Книгу рекордов Гиннесса как актер,
привлекающий к экранам телевизоров наибольшую аудиторию?
   3. В какой, несуществующей ныне, стране родилась актриса, скрытая над
надписью про шенгенскую мультивизу?

Ответ:
   1. Франция.
   2. Великобритания.
   3. СССР.

Комментарий:
   1. Жерар Депардье.
   2. Хью Лори.
   3. Мила Йовович (мультипаспорт).

Источник:
http://joyreactor.cc/post/1570346

Автор:
Данила Аладин

Вопрос 13:
Туалеты в одном из банков и трех ресторанах Женевы засорились купюрами
по 500 евро на общую сумму около 100 тысяч долларов. Представитель
прокуратуры Винсент Деруан заявил: "Пока нет никаких оснований думать,
что найденные деньги - ТАКИЕ". Какое слово мы заменили словом "ТАКИЕ"?

Ответ:
Грязные.

Источник:
https://ru.reuters.com/article/oddlyEnoughNews/idRUKCN1BU0YJ-ORUOE

Автор:
Данила Аладин

Вопрос 14:
В фильме "Тайна семи сестер" описан мир, где семейным парам разрешено
иметь только одного ребенка. Когда рождается семь сестер-близняшек,
родители решают сохранить всех и дают малышкам имена. Когда одну из них
СМЕРТЕЛЬНО ранят, ее сестра, успокаивая, говорит: "Ты - ПРОПУСК, ты
должна верить в лучшее". Какое слово мы пропустили?

Ответ:
Воскресенье.

Комментарий:
Сестрам дали имена в честь дней недели. Так девочки и появляются на
людях - каждая в свой день. В воскресенье Иисус воскрес.

Источник:
Х/ф "Тайна семи сестер" (2017), реж. Томми Виркола, 49-я минута.

Автор:
Данила Аладин

Вопрос 15:
Все, наверное, знают об изобретении хроматографии или об открытии
галлия. Назовите фамилию немецкого инженера, которому приписывали (в
рамках стратегии дезинформации) изобретение Nebelwerfer'а (в переводе с
немецкого - "туманомёта") - буксируемого реактивного миномёта времен
Второй мировой войны.

Ответ:
Небель.

Зачет:
Nebel.

Комментарий:
Как известно, хроматографию изобрел Цвет, с греческого "хрома"
переводится как цвет. А галлий открыл Лекок.

Источник:
   1. https://militaryarms.ru/voennaya-texnika/artilleriya/reaktivnyj-minomet-nebelwerfer-41-42/
   2. https://ru.wikipedia.org/wiki/Хроматография
   3. https://ru.wikipedia.org/wiki/Галлий

Автор:
Данила Аладин

Вопрос 16:
Главный герой фильма "Сделано в Америке" занимается торговлей оружием и
наркотиками и получает такой огромный доход, что не успевает отмывать
деньги. Генеральный прокурор штата, описывая его деятельность,
употребляет спортивный термин, впервые использованный в 1858 году.
Какой?

Ответ:
Хет-трик.

Источник:
   1. Х/ф "Сделано в Америке" (2017), реж. Даг Лайман, 82-я минута.
   2. https://en.wikipedia.org/wiki/Hat-trick

Автор:
Данила Аладин

Вопрос 17:
Стендап-комик Джим Джеффрис рассказывает о своем личном опыте
мастурбации: "Каждый день я <...> лежу с ноутбуком вот здесь, на уровне
груди, чтобы не видеть все те мерзости, что я с собой делаю, <...> и в 9
из 10 случаев я попадаю в ЦЕЛЬ". Какое слово мы заменили словом "ЦЕЛЬ"?

Ответ:
Яблочко.

Комментарий:
Видимо, автор пользуется продукцией компании "Apple".

Источник:
https://www.youtube.com/watch?v=Ft566m3Lfa0&t=21m27s

Автор:
Данила Аладин

Вопрос 18:
Действие компьютерной игры "Assassin's Creed Origins" происходит в
Египте. Если долго бродить по пустыне, то можно начать видеть миражи
оазисов, слышать звуки битвы, встретить кучу летающих насекомых. Также
можно встретить АЛЬФУ. Какое устойчивое выражение из двух слов мы
заменили АЛЬФОЙ?

Ответ:
Неопалимая купина.

Комментарий:
Библейские аллюзии пронизывают игру. Летающие насекомые - одна из казней
египетских - нашествие саранчи.

Источник:
https://www.youtube.com/watch?v=bsjbjSVOb0I

Автор:
Данила Аладин

Вопрос 19:
В вопросе мы заменили одно русское слово греческим и одну латинскую
букву греческой.
   В одном из заданий игры "World of Warcraft" вы противостоите злому
колдуну ИКСлему, который находится в высокой башне. Прежде чем
подняться, необходимо победить его ученицу - ЛЫКО. Напишите замененные
слово и букву.

Ответ:
Флоэма, Кси.

Комментарий:
Иксом мы заменили букву кси. Ксилема и флоэма - проводящие ткани у
растений.

Источник:
   1. https://ru.wowhead.com/npc=115903/флоэма
   2. https://ru.wikipedia.org/wiki/Ксилема
   3. https://ru.wikipedia.org/wiki/Флоэма

Автор:
Алексей Дубровский

Вопрос 20:
Этот уроженец города Рэд-Бэнк, штат Нью-Джерси, актер и режиссер, наш
современник, сыграл более 20 главных ролей, но за всё время на экране
произнес не более двух десятков фраз. Назовите хотя бы одну его роль в
кино.

Ответ:
Молчаливый Боб.

Зачет:
Кевин Смит (потому что периодически играл сам себя).

Источник:
https://en.wikipedia.org/wiki/Kevin_Smith

Автор:
Евгения Крутинина

Вопрос 21:
(pic: 20180328.jpg)
   Какое название созвездия мы от вас скрыли?

Ответ:
Рак.

Зачет:
Cancer.

Источник:
Мультсериал "Симпсоны", s29e05, 4-я минута.

Автор:
Мария Томашевская

Вопрос 22:
Внимание, в вопросе есть замена, при этом остатки рифмы и стихотворного
размера по возможности соблюдены.
   "На всё божья воля, помолимся мы,
   чтоб грех на душе не остался,
   а этот обыкновенный каменный хондрит
   к нам больше на Урал не возвращался".
   Какие два слова мы заменили словами "обыкновенный каменный хондрит"?

Ответ:
Челябинский метеорит.

Комментарий:
Что характерно, в предыдущем куплете поется о том, что все осколки
продали в Москву, чтоб он на Уральской земле не остался.

Источник:
???

Автор:
Евгения Крутинина

Вопрос 23:
С европейскими событиями 1934 и 1947 годов это явление никак не связано.
Среди причин исчезновения называют евроремонт, разрушение озонового слоя
Земли, внутренние войны в популяции, широкое распространение
генно-модифицированных продуктов и, наконец, распространение сотовых
телефонов. Назовите, так сказать, "главного героя", об исчезновении
которого идет речь.

Ответ:
Рыжий таракан.

Зачет:
Прусак.

Комментарий:
Даты - упразднение Пруссии де-факто и де-юре.

Источник:
   1. https://ru.wikipedia.org/wiki/Депопуляция_таракановых_в_странах_СНГ
   2. https://ru.wikipedia.org/wiki/Пруссия

Автор:
Евгения Крутинина

Вопрос 24:
В одной из песен группы "Хоть кого" масон шел по улице домой. И у первых
встречных осведомился об их ближайших родственниках. Учитывая, что папой
оказался Стакан Кагора, ответьте, кто мама.

Ответ:
Мама Епархия.

Источник:
https://www.youtube.com/watch?v=UgfvUTaTV4o

Автор:
Евгения Крутинина

Тур:
23 тур. "Мафия" (Астрахань)

Вопрос 1:
Дуплет.
   1. На карикатуре одиннадцатая, найдя двадцать четвертую и двадцать
пятую, радостно говорит им: "Вот вы где! Эгегей!". В ответ двадцать
пятая недоуменно интересуется, где двадцать шестая. Изобразите
одиннадцатую, двадцать четвертую и двадцать пятую. А в каком порядке,
решать вам.
   2. В аниме девушки, обозначая игривый образ, произносят: "ВОЗЬМИ".
Это связано с тем, что в японском языке "ВОЗЬМИ" является
звукоподражанием мяуканью кошки, аналогом русского "мяу". Что мы
заменили словом "ВОЗЬМИ"?

Ответ:
   1. Й [и краткое], X [экс], Y [вай].
   2. НЯ.

Зачет:
   1. В любом порядке.

Комментарий:
Одиннадцатая буква русского алфавита приняла соответствующие буквы
английского за похожие на них русские буквы. Возможно, у некоторых
команд на бланках для ответов написана точная характеристика качества
пакета, который Вам предстоит сыграть.

Источник:
   1. https://www.facebook.com/komixizm/photos/a.439543042780246/439543279446889/?type=3&theater
   2. https://ru.wikipedia.org/wiki/Ня

Автор:
Александр Борисенко (Астрахань)

Вопрос 2:
Согласно Йохану Хёйзинге, в античности ОНА обозначалась тремя способами
- пайдия (ребячество), атюрма (ерунда) и агон (состязание). В написанном
братьями произведении конца XIX века ОНА является ответом на вопрос. Кто
его задает?

Ответ:
Герман.

Комментарий:
ОНА - игра. Йохан Хёйзинга - автор трактата "Homo ludens" (человек
играющий). В опере "Пиковая дама" Петра и Модеста Чайковских Герман
поет: "Что наша жизнь? Игра!".

Источник:
   1. https://ru.wikipedia.org/wiki/Homo_ludens
   2. https://ru.wikipedia.org/wiki/Пиковая_дама_(опера)

Автор:
Александр Борисенко (Астрахань)

Вопрос 3:
В статье на сайте vokrug.tv, повествующей о съемках телеэпопеи "Хождение
по мукам", упоминается "Гоголь", который СДЕЛАЛ ЭТО. Брюс Уиллис СДЕЛАЛ
ЭТО в фильме "Двенадцать друзей Оушена". Какие три слова мы заменили
словами "СДЕЛАЛ ЭТО"?

Ответ:
Сыграл сам себя.

Комментарий:
"Гоголь" - это пароход, арендованный для съемок одной из сцен. Поскольку
это единственное судно, сохранившееся с начала XX века, название менять
не стали. Брюс Уиллис в фильме "Двенадцать друзей Оушена" исполнил
роль-камео.

Источник:
   1. https://www.vokrug.tv/article/show/akterov_seriala_hozhdenie_po_mukam_polivali_dozhdem_posypali_sheluhoi_i_brosali_v_gryaz_kak_i_gde_snimali_sagu_63247/
   2. https://ru.wikipedia.org/wiki/Двенадцать_друзей_Оушена

Автор:
Анастасия Подколзина (Астрахань)

Вопрос 4:
Учитель Сергей Волков, комментируя цитату министра образования о том,
что расходы на учебники резко сократятся, а на повышение квалификации
несколько возрастут, спрашивает: "Куда денется АЛЬФА?". АЛЬФА - это
название штата в Нигерии и поселка в Красноярском районе Астраханской
области. Что мы заменили АЛЬФОЙ?

Ответ:
Дельта.

Комментарий:
В математике дельта - обозначение конечной разности при изменении
какого-то параметра. Кроме того, дельта - это четвертая буква греческого
алфавита.

Источник:
   1. https://slovesnik.org/novosti/pochemu-druzhok-da-potomu.html
   2. https://ru.wikipedia.org/wiki/Дельта

Автор:
Евгений Бикмаев (Астрахань)

Вопрос 5:
Название русскоязычной статьи с рецептами алкогольных настоек на сливе
колючей одной буквой отличается от названия романа 1977 года. Напишите
название статьи.

Ответ:
"Пьющие в терновнике".

Комментарий:
Слива колючая - другое название терновника. Автор вопроса благодарит
свой алкоголизм за помощь в написании вопроса.

Источник:
   1. http://therumdiary.ru/napitki/nastojki/ternovye-nastojki.html
   2. https://ru.wikipedia.org/wiki/Тёрн
   3. https://ru.wikipedia.org/wiki/Поющие_в_терновнике

Автор:
Александр Борисенко (Астрахань)

Вопрос 6:
В 1831 году при написании картины о спасении заблудившегося в горах
путника Эдвин Лендсир в том числе изобразил ИКС, тем самым создав
несоответствующий действительности миф. Назовите ИКС двумя словами,
начинающимися на одну и ту же букву.

Ответ:
Бочонок бренди.

Комментарий:
Художник изобразил одного из сенбернаров, спасших путника, с бочонком на
шее. Хотя в действительности никаких емкостей со спиртным собакам на шею
не вешали.

Источник:
http://bustersmyth.ru/98

Автор:
Александр Борисенко (Астрахань)

Вопрос 7:
Будучи надетым на человека, умный он сообщает скорость движений, их
силу, количество сожженных калорий, затраченное время. Устройство
синхронизируется со смартфоном, и владелец может делиться данными в
соцсетях. В вопросе в слове французского происхождения мы пропустили
четыре буквы. Восстановите это слово.

Ответ:
Кондом.

Комментарий:
Давайте делиться данными!

Источник:
https://life.ru/t/технологии/980620/v_anghlii_nachinaiutsia_prodazhi_umnykh_prieziervativov

Автор:
Александр Борисенко (Астрахань)

Вопрос 8:
Анализируя реплики мужчин некой профессии, Уэльбек отмечает, что
французы убоги, поскольку используют мат, констатируя факт. Американцев
же он называет людьми истинно верующими. Назовите эту профессию словом с
двумя корнями.

Ответ:
Порноактеры.

Комментарий:
Французские порноактеры матерятся, констатируя факт получения
наслаждения. Американские же используют фразы типа "О Боже!".

Источник:
М. Уэльбек. Покорность. http://flibusta.is/b/426509/read

Автор:
Александр Кабин (Астрахань)

Вопрос 9:
Горная порода катлинит добывалась в каменоломнях плато "КотО-де-ПрерИ".
Для воюющих племен эти каменоломни были нейтральными территориями. Что
изготавливали из катлинита?

Ответ:
Трубки мира.

Комментарий:
После двух предыдущих вопросов нас потянуло задать вопрос о курении.

Источник:
   1. https://ru.wikipedia.org/wiki/Катлинит
   2. https://ru.wikipedia.org/wiki/Кото-де-Прери
   3. https://ru.wikipedia.org/wiki/Трубка_мира

Автор:
Александр Борисенко (Астрахань)

Вопрос 10:
В основе франкфуртских шахмат лежит правило, действовавшее в советском
мультфильме по мотивам азиатских сказок. Так конь в них, например, может
СДЕЛАТЬ ЭТО. В армии, чтобы СДЕЛАТЬ ЭТО, необходимы 100 дней. Какие два
слова на одну букву мы заменили словами "СДЕЛАТЬ ЭТО"?

Ответ:
Стать слоном.

Комментарий:
В мультфильме "Дракон" убивший дракона сам становился драконом. Во
франкфуртских шахматах фигура, взявшая другую фигуру, превращается в нее
(сохраняя свой цвет). Солдат-срочник после 100 дней службы становится
"слоном".

Источник:
   1. https://ru.wikipedia.org/wiki/Франкфуртские_шахматы
   2. https://ru.wikipedia.org/wiki/Дракон_(мультфильм)
   3. http://army-blog.ru/neustavnye-zvaniya/

Автор:
Александр Борисенко (Астрахань)

Вопрос 11:
Блиц, господа!
   1. Каким словом называется песня Олега Иванова, в 1970 году
победившая на Всесоюзном конкурсе молодых композиторов?
   2. Какими двумя словами называется басня Льва Толстого, одним из
действующих лиц которой является медведь?
   3. Как называется роман Ремарка, опубликованный в 1936 году?

Ответ:
   1. "Товарищ".
   2. "Два товарища".
   3. "Три товарища".

Источник:
   1. https://ru.wikipedia.org/wiki/Товарищ_(песня)
   2. Л.Н. Толстой. Два товарища.
https://rvb.ru/tolstoy/01text/vol_10/01text/0213.htm
   3. https://ru.wikipedia.org/wiki/Три_товарища

Автор:
Александр Борисенко (Астрахань)

Вопрос 12:
В 2016 году, налаживая двусторонние отношения, Россия и Турция заключили
соглашение о строительстве газопровода. Повествующий об этом новостной
сюжет был назван двумя существительными, в одном из которых "пропала"
буква. Напишите название сюжета.

Ответ:
"Труба мира".

Комментарий:
Заключение этого соглашения должно немного наладить напряженные
отношения Турции и России. Газопровод сложно назвать "трубкой", а слово
"пропала" - подсказка, наводящая на детскую загадку "А и Б сидели на
трубе...". И первый, и последний вопрос этого блока мы посвятили трубке
мира.

Источник:
https://www.1tv.ru/news/issue/2016-10-16/21:00#5

Автор:
Анастасия Подколзина (Астрахань)

Вопрос 13:
Символично, что талант этого исполнителя был замечен участником группы
"The Miracles" [зэ мИрэклз]. Можно сказать, что путь в шоу-бизнес для
НЕГО был открыт ИМ. Назовите этого исполнителя.

Ответ:
Стиви Уандер.

Комментарий:
И псевдоним исполнителя, и название группы переводятся как "чудо".

Источник:
http://echomp3.ru/500_rolling_stone/1_stevie_wonder/

Автор:
Александр Борисенко (Астрахань)

Вопрос 14:
Прочтя рукопись первого романа Берроуза, редактор выписал ему чек на 400
долларов - хорошие деньги за хорошее произведение. Мы не спрашиваем,
какое слово мы дважды заменили в вопросе. Назовите входящий в состав
этого слова безалкогольный напиток.

Ответ:
"Фанта".

Комментарий:
Фантастические деньги за фантастическое произведение.

Источник:
https://ru.wikipedia.org/wiki/Берроуз,_Эдгар_Райс

Автор:
Александр Борисенко (Астрахань)

Вопрос 15:
В этом вопросе все слова являются заменой. Какое имя упоминалось в
изначальном вопросе?

Ответ:
Тесей.

Комментарий:
Парадокс Тесея - парадокс, который можно сформулировать так: "Если все
составные части исходного объекта были заменены, остается ли объект тем
же объектом?". Так и мы заменили все слова в изначальном вопросе.

Источник:
https://ru.wikipedia.org/wiki/Корабль_Тесея

Автор:
Артем Миндрин, Сергей Падерин (Москва)

Вопрос 16:
А вот возьмите-ка следующий вопрос.
   Нередко современные российские дети видят в названии этого
стихотворения ненависть. А тут еще упоминание в тексте озверевшей толпы
и плевка в лицо. Дайте мне это название.

Ответ:
"Нате".

Комментарий:
У многих детей русское слово "нате" не входит в активно используемую и
знакомую им часть тезауруса. А вот английское "hate, HATE" им знакомо.
Стихотворение по своему содержанию действительно вполне себе пропитано
ненавистью. Предложение "возьмите вопрос" и просьба "дайте ... название"
- своего рода намеки на "нате".

Источник:
   1. Эфир радиостанции "Серебряный дождь", апрель 2017 г.
   2. В.В. Маяковский. Нате.
http://www.feb-web.ru/feb/mayakovsky/texts/ms0/ms1/ms1-056-.htm

Автор:
Александр Борисенко (Астрахань)

Вопрос 17:
Выбор Катара в качестве страны - хозяйки чемпионата мира по футболу 2022
года вызвал массу критики и дискуссий. В частности, дискуссии вызвал тот
факт, что ряд игр в Катаре пройдет в Лусаиле. На конец 2014 года Лусаил
был ИКСОМ. "ИКС" - произведение 2001 года. Назовите любого из его
авторов.

Ответ:
Регина Лисиц.

Зачет:
Игорь Корнелюк.

Комментарий:
Лусаил - строящийся плановый город, возведение которого началось в 2006
году. Можно сказать, что на конец 2014 года Лусаил был городом, которого
нет. "Город, которого нет" - песня Игоря Корнелюка, слова которой
написала Регина Лисиц.

Источник:
   1. http://champions.name/text/wc-2022-lusail.htm
   2. https://ru.wikipedia.org/wiki/Лисиц,_Регина_Климентьевна

Автор:
Игорь Гужвин (Астрахань)

Вопрос 18:
В интервью с Владимиром Познером журналист Юрий Рост так объяснил
разницу между одиночеством и уединением: "Одиночество - это ИКС снаружи,
а уединение - это ИКС изнутри". Коротким словом ответьте: что такое ИКС?

Ответ:
Ключ.

Комментарий:
Одиночество - вынужденное явление, уединение - добровольное.

Источник:
https://www.1tv.ru/shows/pozner/vypuski/gost-yuriy-rost-pozner-vypusk-ot-26-06-2017
(39:32)

Автор:
Евгений Бикмаев (Астрахань)

Вопрос 19:
Фаина Раневская родилась в високосном 1896 году. В следующем високосном
году родители отправили четырехлетнюю Фаину в Мариинскую гимназию для
девочек. В предыдущем предложении мы изменили одно из слов. Восстановите
его.

Ответ:
Восьмилетнюю.

Комментарий:
Следующим високосным годом после 1896 был 1904-й. 1900-й високосным не
был. Дело в том, что в григорианском календаре год является високосным в
двух случаях: либо он кратен 4, но при этом не кратен 100, либо кратен
400. Год не является високосным, если он не кратен 4, либо он кратен
100, но при этом не кратен 400. Последние годы столетий, оканчивающиеся
на два нуля, в трех случаях из четырех не являются високосными. Так,
1700, 1800 и 1900 годы не являются високосными, так как они кратны 100 и
не кратны 400. Годы 1600 и 2000 - високосные, так как они кратны 400.
Годы 2100, 2200 и 2300 - не високосные.

Источник:
   1. https://topwar.ru/54474-koroleva-epizoda-faina-georgievna-ranevskaya.html
   2. https://ru.wikipedia.org/wiki/Високосный_год

Автор:
Александр Борисенко (Астрахань)

Вопрос 20:
"Стишки-порошки" - это разновидность "стишков-пирожков", отличающаяся от
основного жанра тем, что вторая и четвертая строки в этих четверостишиях
рифмуются, а размер по слогам - 9, 8, 9, 2. Прослушайте порошок,
написанный, вероятно, человеком в летАх:
   зима бежишь с горы кататься
   наперекор своей судьбе
   и кажется что [ПРОПУСК]
   тебе
   Заполните пропуск двумя словами, либо тремя знаками.

Ответ:
Минус двадцать.

Зачет:
-20.

Комментарий:
-20 - не только зимняя погода, но и ощущение возраста. А мы играли 20-й
вопрос всего пакета, так что у нас тоже -20.

Источник:
https://vk.com/sandalporoshki?w=wall-31481258_93665

Автор:
Александр Кабин (Астрахань)

Вопрос 21:
В июле 2017 года в Москве прошел ливень, затопивший весь город. В
рассылке информационного сайта "Медуза" эта новость была озаглавлена
двумя словами, между которыми редакция намеренно поставила тире, а не
дефис. Воспроизведите этот заголовок.

Ответ:
Москва - река.

Комментарий:
Подобный заголовок означает, что Москва (город) стала рекой. Москва-река
- закрепившееся название реки Москвы.

Источник:
Рассылка "Медузы" от 14.07.2017 г.

Автор:
Евгений Бикмаев (Астрахань)

Вопрос 22:
(pic: 20180329.jpg)
   На фотографии, сделанной в аэропорту Дрездена в середине 1960-х, вы
видите списанный истребитель МиГ-15, установленный на платформу тягача.
Это был один из первых, но далеко не последних случаев использования на
аэродромах реактивной ТАКОЙ техники. Какое сложносоставное слово мы
заменили словом "ТАКОЙ"?

Ответ:
Снегоочистительной.

Зачет:
Снегоуборочной.

Комментарий:
Старые списанные истребители использовали в качестве снегоочистительной
техники.

Источник:
https://4kolesa.mirtesen.ru/blog/43346453727/Aerodromnaya-tehnika-proshlogo.-CHast-2

Автор:
Константин Артемьев (Астрахань)

Вопрос 23:
(pic: 20180330.jpg)
   Заполните пропуск в раздаточном материале двумя словами.

Ответ:
Bi son.

Комментарий:
"No, I'm bison" - "Нет, я бизон", или "No, I'm bi son" - "Нет, я би,
сын".

Источник:
(pic: 20180331.jpg)

Автор:
Артем Миндрин, Сергей Падерин (Москва)

Вопрос 24:
"Подними себе настроение", - призывает вывеска заведения, увиденная
автором вопроса. Мы не спрашиваем, какая буква заменена в вопросе двумя
другими. Напишите разновидность этого заведения.

Ответ:
Кальянная.

Комментарий:
В оригинале вывеска гласит: "Подыми себе настроение". А у нас прошел
последний вопрос тура - можете сходить подымить.

Источник:
ЛОАВ.

Автор:
Александр Борисенко (Астрахань)

Тур:
24 тур. "Гномы-конокрады" (Москва - Пермь)

Вопрос 1:
   <раздатка>
   Оценит десять книжек сразу
   (Он знает лишь названья книг).
   Он изо всех искусств постиг
   Одно - искусство общей фразы,
   Пусть нет и капли мысли в ней...
   Таков [...] наших дней
   </раздатка>
   Мы раздали вам последние шесть строк стихотворения. Восстановите
пропущенное слово.

Ответ:
Онегин.

Источник:
https://vk.com/old_mags?w=wall-110681027_19266

Автор:
Григорий Зырянов (Москва)

Вопрос 2:
Африканский город Ламу в XIX веке был центром торговли, куда стекалось
всё богатство Восточной Африки. Интересно, что первый британский консул
Ламу носил... Какую фамилию?

Ответ:
Хаггард.

Комментарий:
Джек Хаггард - брат Генри Хаггарда, автора романа "Копи царя Соломона".

Источник:
Журнал "Аэрофлот", январь 2018 г. - С. 178.

Автор:
Григорий Зырянов (Москва)

Вопрос 3:
Согласно рекламной статье в журнале, часы "Hamilton" можно увидеть более
чем в четырехстах кинокартинах на НИХ. Назовите ИХ двумя словами,
начинающимися на одну и ту же букву.

Ответ:
Запястья звезд.

Источник:
Журнал "Аэрофлот", январь 2018 г. - С. 168.

Автор:
Григорий Зырянов (Москва)

Вопрос 4:
В каком городе, согласно шутке, в кафе подают еще и спасательный круг?

Ответ:
Венеция.

Источник:
https://acomics.ru/~itchyfeet/71

Автор:
Григорий Зырянов (Москва)

Вопрос 5:
Не любящие американскую еду путешественники с Ближнего Востока сначала
не знают, что будут есть в Америке, но потом вспоминают, что путешествие
придется как раз на время ЕГО. Назовите ЕГО.

Ответ:
Рамадан.

Источник:
https://acomics.ru/~itchyfeet/37

Автор:
Григорий Зырянов (Москва)

Вопрос 6:
По сюжету праздничного выпуска английского сериала "Отбросы", главные
герои борются против могущественного лидера секты. В конце выпуска на
голову поверженного лидера падает висящий на стене ИКС, после чего его
последователи заявляют, что он опять спас их всех. Назовите ИКС двумя
словами.

Ответ:
Терновый венок.

Источник:
Телесериал "Отбросы", s02e07.

Автор:
Григорий Зырянов (Москва)

Вопрос 7:
Историк ФроЯнов утверждает, что в насыщенной социальными связями среде в
местах сгустков этих связей происходит ОНА городов. Известный швед
приурочил ЕЕ к нулю. Назовите ЕЕ.

Ответ:
Кристаллизация.

Источник:
   1. И.Я. Фроянов, А.Ю. Дворниченко. Города-государства Древней Руси.
http://flibusta.is/b/468845/read
   2. https://ru.wikipedia.org/wiki/Градус_Цельсия

Автор:
Григорий Зырянов (Москва)

Вопрос 8:
Историк ФроЯнов рассказывает, что мятеж 1113 года в Киеве, во время
которого были убиты несколько евреев-ростовщиков, был борьбой против
порабощения горожан. Какое слово мы заменили в тексте вопроса?

Ответ:
Закабаления.

Источник:
И.Я. Фроянов, А.Ю. Дворниченко. Города-государства Древней Руси.
http://flibusta.is/b/468845/read

Автор:
Григорий Зырянов (Москва)

Вопрос 9:
Героиня романа Андрея Лазарчука, действие которого происходит в
античности, клянется, что не изменит мужу даже с Антоном. И
действительно, по версии Лазарчука, ребенка она усыновила. Какие три
буквы мы заменили в тексте вопроса?

Ответ:
гел.

Комментарий:
"Не изменит мужу даже с ангелом" - прямая отсылка к непорочному зачатию.

Источник:
А.Г. Лазарчук. Мой старший брат Иешуа. http://flibusta.is/b/167503/read

Автор:
Григорий Зырянов (Москва)

Вопрос 10:
В одном фэнтезийном романе демиург создал только Солнце и Луну, из-за
чего ЕГО приходилось составлять сразу на всех. Назовите ЕГО греческим
словом.

Ответ:
Гороскоп.

Источник:
М.Г. Успенский. Невинная девушка с мешком золота.
https://books.google.ru/books?id=5iJIDwAAQBAJ&pg=PT127#v=onepage&q&f=false

Автор:
Григорий Зырянов (Москва)

Вопрос 11:
Солдаты Наполеона называли башкирскую конницу северными ИМИ. ОН на
протяжении более тысячи километров протекает по границе... Каких двух
государств?

Ответ:
Россия и Китай.

Комментарий:
ОН - Амур.

Источник:
   1. http://kuglib.ru/load/interesnoe/god_rossijskoj_istorii/chego_ne_znal_napoleon/38-1-0-422
   2. https://mkset.ru/news/culture/28-08-2015/severnymi-amurami-bashkir-prozvali-parizhskie-damy
   3. https://ru.wikipedia.org/wiki/Амур

Автор:
Григорий Зырянов (Москва)

Вопрос 12:
Проклятый дьяволом герой Роберта Артура, в очередной раз испытывая на
себе действие проклятия, говорит, что чувствует ЕГО. Назовите ЕГО двумя
словами, начинающимися на парные согласные.

Ответ:
Запах серы.

Источник:
Р. Артур. Сатана и Сэм Шэй. http://flibusta.is/b/485969/read

Автор:
Григорий Зырянов (Москва)

Вопрос 13:
Приехавший в Африку американец Роберт Сапольски сравнил царивший там
бесконтрольный капитализм и беззаконие с НИМ. Назовите ЕГО двумя
словами.

Ответ:
Дикий Запад.

Источник:
Р. Сапольски. Записки примата. Необычайная жизнь ученого среди павианов.
http://flibusta.is/b/504460/read

Автор:
Григорий Зырянов (Москва)

Вопрос 14:
Коренные жители Восточной Африки называли Роберта Сапольски ТАКИМ.
Однако это было связано не с его схожестью с определенной группой
народов, а с большой активностью Солнца. Что мы заменили словом "ТАКОЙ"?

Ответ:
Краснокожий.

Источник:
Р. Сапольски. Записки примата. Необычайная жизнь ученого среди павианов.
http://flibusta.is/b/504460/read

Автор:
Григорий Зырянов (Москва)

Вопрос 15:
В одном мультфильме ОНИ использовались в качестве охранных животных.
Блогер Дмитрий Сыендук пошутил, что местные жители должны хорошо
разбираться в генах, раз приручили столь опасных животных. Назовите ИХ
одним словом.

Ответ:
Крокодилы.

Источник:
https://www.youtube.com/watch?v=szmLr2nExIQ

Автор:
Григорий Зырянов (Москва)

Вопрос 16:
Персонаж романа "Двадцать лет спустя" заявляет, что желает научиться
астрономии. После этого он упоминает один из смертных грехов. Какую
букву мы пропустили в тексте вопроса?

Ответ:
Г.

Комментарий:
Герцог де Бофор желал научиться гастрономии.

Источник:
А. Дюма. Двадцать лет спустя. http://flibusta.is/b/168267/read

Автор:
Григорий Зырянов (Москва)

Вопрос 17:
В кафе "Батя-бургер" одно из блюд называется в честь главы европейского
государства. Назовите это государство.

Ответ:
Беларусь.

Комментарий:
Жареный картофель "Лукашенко".

Источник:
https://www.batyaburgers.ru/

Автор:
Григорий Зырянов (Москва)

Вопрос 18:
Аккум - это пустыня, появившаяся во второй половине XX века. Назовите
географический объект, на месте которого она располагается.

Ответ:
Аральское море.

Источник:
https://ru.wikipedia.org/wiki/Аралкум

Автор:
Григорий Зырянов (Москва)

Вопрос 19:
Песня группы "50 копеек" про один из периодов истории России называется
"Суровый РЭП". Какую букву мы заменили в предыдущем предложении?

Ответ:
Н.

Комментарий:
"Суровый НЭП".

Источник:
https://vk.com/topic-2310359_21982241

Автор:
Григорий Зырянов (Москва)

Вопрос 20:
В рассказе Джанни Родари жандарм обвиняет мальчика, одетого в зеленое
пальто, красный шарф и белый шерстяной берет, в политическом протесте и
говорит о НЕМ. Назовите ЕГО двумя словами.

Ответ:
Итальянский флаг.

Источник:
Дж. Родари. Бухгалтер и бора. http://flibusta.is/b/118147/read#t30

Автор:
Григорий Зырянов (Москва)

Вопрос 21:
"Состав" по-английски - "roster" [рОстер]. По мнению Бена Линдберга,
спортивные фанаты, которые в межсезонье лихорадочно пытаются угадать
изменения в составах команд, занимаются... Ответьте придуманным словом:
чем?

Ответ:
Ростербацией.

Зачет:
Rosterbation.

Источник:
Ben Lindbergh, Sam Miller. The Only Rule Is It Has to Work.
https://books.google.ru/books?id=Z-e2CgAAQBAJ&pg=PA9#v=onepage&q&f=false

Автор:
Александр Печёный (Москва)

Вопрос 22:
Дуплет.
   1. Древние вавилоняне верили, что человек умирал, когда его
регистрировал небесный ИКС. Кого мы заменили ИКСОМ?
   2. Сергей Николаев пишет, что в армии немного способов выделиться.
Например, некоторые выпрямляли АЛЬФЫ. Что мы заменили АЛЬФОЙ?

Ответ:
   1. Писец.
   2. Бляха.

Источник:
   1. Philip B. Meggs, Alston W. Purvis. Meggs' History of Graphic
Design.
   2. http://kefiijrw.com/army

Автор:
Александр Печёный (Москва)

Вопрос 23:
Пользователь Твиттера пошутил, что дизельпанк-проект Сергея Шнурова
называется "резиновый мужик". Какую букву в предыдущем предложении мы
пропустили?

Ответ:
Д.

Комментарий:
Дрезиновый.

Источник:
https://twitter.com/pod_veshestvami/status/954464626510581761/

Автор:
Михаил Миронов (Москва)

Вопрос 24:
Герой романа "Повесть о великом хадже" называет ЕГО синей линейкой с
белыми и черными точками, которую положили на желтую бумагу.
Строительство предшественника ЕГО было заброшено - вероятно, из-за того
что жрецы опасались наплыва греческих купцов. Назовите ЕГО.

Ответ:
Суэцкий канал.

Источник:
   1. Ф.А. Мухаммадиев. Путешествие на тот свет, или Повесть о великом
хадже. http://flibusta.is/b/260160/read
   2. https://ru.wikipedia.org/wiki/Канал_фараонов

Автор:
Григорий Зырянов (Москва)

Тур:
25 тур. "Зимняя меланхолия" (Москва - Калининград - Обнинск -
Санкт-Петербург - Салават)

Редактор:
Андрей Волыхов и Екатерина Лобкова (Москва)

Инфо:
Редакторы благодарят за тестирование и ценные замечания: Александру
Балабан и Тимофея Бокова, Татьяну Белецкую, Илью Иванова, Михаила
Иванова, Антона Мазнева, Егора Мухина, Антона Пинчука, Павла Семенюка,
Евгения Сидоровского, Игоря Тюнькина, Юрия Харькина и Сергея Шорина.

Вопрос 1:
Ася Казанцева рассказывает про флешмоб против гомеопатии, который должен
был начаться в одиннадцатом часу дня. А в какое точно время?

Ответ:
10:23.

Комментарий:
6*10^23 [шесть умножить на десять в двадцать третьей степени] - число
Авогадро. Гомеопаты оперируют разведениями куда бОльших порядков, а это
должно означать, что в их препаратах не остается ни одной молекулы
вещества.

Источник:
А. Казанцева. В Интернете кто-то неправ! Научные исследования спорных
вопросов. http://flibusta.is/b/489418/read

Автор:
Андрей Волыхов (Москва)

Вопрос 2:
[Ведущему: не указывать границы цитаты и не указывать, что в цитате есть
пропуск.]
   Участник группы "Буерак" рассказывает, что одна из его песен
описывает "жизнь человека, который книжек не читает, а <...> на заводе
каждый день с утра до ночи". На обложке мини-альбома, в который входит
эта песня, изображено "Оружие". Какое?

Ответ:
Булыжник.

Зачет:
"Булыжник - оружие пролетариата".

Комментарий:
Альбом называется "Пролетариат", а на обложке изображена известная
скульптура Ивана Шадра "Булыжник - оружие пролетариата".

Источник:
   1. https://daily.afisha.ru/archive/vozduh/music/buerak-ishchite-vdohnovenie-v-neozhidannyh-mestah/
   2. https://buerak.bandcamp.com/album/--6
   3. https://ru.wikipedia.org/wiki/Булыжник_%E2%80%94_оружие_пролетариата

Автор:
Рузель Халиуллин (Салават - Москва)

Вопрос 3:
Глядя на падающие с НЕЕ крупинки оксида железа, Уильям БасИнски понял,
что видит гибель своего произведения двадцатилетней давности.
Результатом стало создание альбома "Disintegration Loops" [дизинтегрЕйшн
лупс]. Назовите ЕЕ.

Ответ:
[Магнитофонная] лента.

Зачет:
[Магнитофонная] пленка.

Комментарий:
Магнитофонные записи начала восьмидесятых годов стали разрушаться прямо
во время оцифровки, и Басински сделал этот процесс ключевым при записи
своего альбома.

Источник:
https://ru.wikipedia.org/wiki/Басински,_Уильям

Автор:
Рузель Халиуллин (Салават - Москва), Екатерина Лобкова (Обнинск -
Москва)

Вопрос 4:
Для обложки альбома "Disintegration Loops" [дизинтегрЕйшн лупс] БасИнски
использовал кадры, снятые с крыши. Назовите точную дату съемки.

Ответ:
11 сентября 2001 года.

Комментарий:
Произведение, в котором воплотился конец его юности и американской
пасторали, композитор проиллюстрировал видом дымящегося Всемирного
торгового центра, который снимал с крыши своего дома в Бруклине.

Источник:
   1. https://en.wikipedia.org/wiki/The_Disintegration_Loops
   2. https://ru.wikipedia.org/wiki/Басински,_Уильям

Автор:
Екатерина Лобкова (Обнинск - Москва)

Вопрос 5:
В рассказе Юрия Нагибина Сомерсет Моэм называет себя Петербургским, хотя
точнее было бы Петроградским. Назовите человека, которого Моэм упоминает
перед этим.

Ответ:
Лоуренс [Аравийский].

Зачет:
Томас Эдвард Лоуренс.

Комментарий:
Моэм был агентом британской разведки и в 1917 году был послан в
Петроград с задачей не дать России выйти из войны. В те же годы
лейтенант Лоуренс был советником командира Арабской освободительной
армии, которая вместе с британскими войсками генерала Алленби сражалась
против турок. "Я спросил, была ли служба Моэма в "Интеллидженс сервис"
тоже игрой. "Чистой воды! - не задумываясь, ответил он. - Лоуренс -
Аравийский. Моэм - Петербургский".

Источник:
   1. Ю.М. Нагибин. Ненаписанный рассказ Сомерсета Моэма.
http://flibusta.is/b/164728/read
   2. https://ru.wikipedia.org/wiki/Моэм,_Уильям_Сомерсет

Автор:
Наталия Новыш (Санкт-Петербург)

Вопрос 6:
[Ведущему: четко прочитать слово "блАгосферА", чтобы команды не услышали
"блогосферы".]
   Лингвист Максим Кронгауз провел в центре "Благосфера" мастер-класс, в
названии которого упоминался Теодор ИКС. Какое слово английского
происхождения мы заменили ИКСОМ?

Ответ:
Фандрайзер.

Комментарий:
"Благосфера" - некоммерческая организация, и известный лингвист учил
благотворителей правильно просить денег. В частности, лучше давать
конкретную информацию, чем абстрактно говорить о служении добру, и стоит
осторожно употреблять заимствованные термины типа "краудфандинг" и
"фандрайзинг".

Источник:
https://philology.hse.ru/conflictology/news/201370087.html

Автор:
Екатерина Лобкова (Обнинск - Москва)

Вопрос 7:
[Ведущему: четко прочитать слова "Татьяны" и "Татианы", чтобы была
заметна разница.]
   В заголовке статьи лингвиста Татьяны Шмелёвой упоминаются день памяти
великомученицы Татианы и валютная единица. Какая?

Ответ:
Биткоин.

Зачет:
Биткойн, биток.

Комментарий:
Русский язык склонен избавляться от сочетания гласных. Например, таким
путем, который характерен для заимствованных христианских имен, - так
Татиана превратилась в Татьяну. Однако написание сравнительно новых
заимствований, таких как "биткоин" или "Таиланд", ориентировано на
оригинал.

Источник:
https://vnovgorode.ru/avtorskie-materialy/15211-russkij-yazyk-novgorodtsev-kak-pravilno-i-pochemu-tatyanin-den-i-bitkoin.html

Автор:
Екатерина Лобкова (Обнинск - Москва)

Вопрос 8:
Говоря о плохо работавшем устройстве ввода, автор вопроса заменил одну
букву в существующем английском слове. Воспроизведите получившийся
неологизм.

Ответ:
Toughpad.

Зачет:
Тафпад.

Источник:
ЛОАВ.

Автор:
Андрей Волыхов (Москва)

Вопрос 9:
У героини Алана Брэдли никак не получалось развести костер, пока она не
нашла емкость с керосином. Героиня говорит: "Я плеснула чуть-чуть на
дрова, и не успели бы вы сказать "ПРОПУСК", как чайник весело кипел".
Назовите пропущенное нами имя собственное.

Ответ:
Баден-Пауэлл.

Комментарий:
Роберт Баден-Пауэлл - основатель скаутского движения. Прежде чем
покинуть организацию девочек-скаутов, героиня поклялась, что больше не
окажется в ситуации, когда надо добыть огонь с помощью спички и шнурка
от туфли, и теперь она предпочитает поискать приличное горючее.

Источник:
А. Брэдли. Копченая селедка без горчицы.
https://books.google.ru/books?id=cpXTCgAAQBAJ&pg=PT31#v=onepage&q&f=false

Автор:
Мария Могильда (Калининград - Москва)

Вопрос 10:
Представляясь, хвастливый персонаж американского мультсериала Ричард
Стрэттон Четвертый говорит, что близкие друзья кратко зовут его... Как?

Ответ:
Айви.

Комментарий:
Как можно догадаться по имени, персонаж - аристократ и гордится своим
высоким положением. Видимо, он намекает на образование, полученное в
престижных университетах Лиги плюща (по-английски Ivy League [Айви
лиг]). При этом IV [четыре] римскими цифрами тоже можно прочитать как
"Айви".

Источник:
Мультсериал "Агент Арчер", s07e03.

Автор:
Кирилл Железнов (Калининград - Москва)

Вопрос 11:
Говоря о влиянии ЭТОГО на свой характер, Ричард Докинз вспоминает, как,
уже будучи студентом Оксфорда, забыл дома ручку и в итоге не смог
записать лекцию. В СССР и введение ЭТОГО, и упразднение были
мотивированы соображениями дисциплины. Назовите ЭТО.

Ответ:
Раздельное обучение [мальчиков и девочек].

Зачет:
Мужская школа; школа для мальчиков; школа с раздельным обучением.

Комментарий:
Докинз мог бы попросить ручку у соседа, но рядом сидела девушка, и
обратиться к ней он не решился, поскольку обучался в школе для мальчиков
и плохо умел общаться с противоположным полом. В 1943 году в Советском
Союзе ввели раздельное обучение, а в 1954 году отменили, причем сперва
предполагалось, что присутствие девочек отвлекает мальчиков от учебы, а
потом - что сдерживает хулиганов.

Источник:
   1. https://theoryandpractice.ru/posts/16467-spasibo-chto-nauchili-menya-dumat-richard-dokinz-o-svoey-uchebe-v-oksforde-i-idealnykh-lektorakh
   2. http://portalus.ru/modules/shkola/rus_readme.php?subaction=showfull&id=1193232434&archive=1196815145

Автор:
Екатерина Лобкова (Обнинск - Москва)

Вопрос 12:
В одном произведении говорится, что приходится жить с текущей крышей уже
который год и вообще ИХ никогда не заменят. Назовите ИХ тремя словами.

Ответ:
Дворцов заманчивые своды.

Источник:
https://vk.com/perawki?w=wall-28122932_70353

Автор:
Андрей Волыхов (Москва)

Вопрос 13:
В статьях о некой "принцессе" упоминаются высокая сложность, сильные
ощущения и риск инфекции, а также женская смелость. В "тезку" этой
"принцессы" когда-то разрешалось входить только в собственной обуви.
Назовите имя этой "принцессы".

Ответ:
Альбертина.

Комментарий:
Это вариант пирсинга - женский аналог мужского пирсинга гениталий,
известного как "Принц Альберт". Альбертиной также называется венский
художественный музей, открытый для приличной публики в 1822 году, хотя
вы могли подумать и о кёнигсбергском университете.

Источник:
   1. https://en.wikipedia.org/wiki/Princess_Albertina_piercing
   2. http://master-tattoo.ru/pirsing/zhenskij-intim-pirsing/printsessa-albertina
   3. https://ru.wikipedia.org/wiki/Галерея_Альбертина

Автор:
Андрей Волыхов (Москва)

Вопрос 14:
Говоря о фильме "МакбЕт", один российский критик недоумевает, зачем ему
рассказали, что в финале сын БАнко поднимает окровавленный ИКС. Назовите
фамилию этого критика.

Ответ:
Волобуев.

Комментарий:
Персонаж известного анекдота так и не смог произнести в одном
предложении слова "Волобуев" и "меч", вот и мы использовали замену.
Роман Волобуев пишет - вероятно, в шутку, - что думал, что Макбет
отобьется от врагов, пока не прочел, как маленький сын Банко поднимает
окровавленный меч Макбета.

Источник:
   1. https://twitter.com/romanvolobuev/status/669843045433651200/
   2. https://www.anekdot.ru/id/-712100014/ (источник вдохновения)

Автор:
Рузель Халиуллин (Салават - Москва)

Вопрос 15:
Герой фильма "Мстители: Эра АльтрОна", наблюдая, как противник плодит
свои копии, упоминает ПЕРВЫХ и ВТОРЫХ. Назовите ПЕРВЫХ и ВТОРЫХ
рифмующимися словами.

Ответ:
Кролики, католики.

Зачет:
В любом порядке.

Комментарий:
"Он размножается быстрее, чем кролики-католики", - говорит Ник Фьюри.
По-видимому, эта непочтительная шутка связана с запретом на контрацепцию
в католической церкви, хотя известно высказывание папы Франциска о том,
что католикам как раз подобает не плодиться по-кроличьи, а быть
ответственными родителями.

Источник:
   1. http://transcripts.wikia.com/wiki/Avengers:_Age_of_Ultron
   2. https://www.urbandictionary.com/define.php?term=catholic%20rabbit
   3. http://www.bbc.com/news/world-asia-30890989

Автор:
Наталия Новыш (Санкт-Петербург)

Вопрос 16:
Один сериал про работницу ресторана называется "АЛЬФА на кухне". Но, по
шуточному предположению автора вопроса, ему бы следовало называться
"АЛЬФА растет". Назовите слово, замененное АЛЬФОЙ.

Ответ:
Талия.

Комментарий:
Забавное имя для поварихи. Аллюзия на песню Семена Слепакова сделана
вполне намеренно.

Источник:
https://www.kinopoisk.ru/film/988863/

Автор:
Наталия Новыш (Санкт-Петербург)

Вопрос 17:
Говоря об ошибочности слова "дрова", представитель компании "On the
Bones" [он зе бОунз] упоминает тепло рук посетителей бара. Напишите в
исходном виде слово, в котором мы пропустили больше половины букв.

Ответ:
Сидроварение. Незачет: Сидровар; сидроварка.

Комментарий:
Пивовар говорит: "Первое тепло, которое получает сидр, - это тепло ваших
рук". Дрова дают тепло, не говоря уж о том, что в дрова можно напиться.
:-)

Источник:
Презентация компании "On the Bones".

Автор:
Наталия Новыш (Санкт-Петербург)

Вопрос 18:
   <раздатка>
   Euophrys omnisuperstes
   </раздатка>
   Ответьте максимально точно: где был обнаружен вид пауков Euophrys
omnisuperstes [эуОфрис омнисупЕрстес]?

Ответ:
На Эвересте.

Зачет:
На вершине Эвереста.

Комментарий:
В слове "omnisuperstes", которое означает "переживающий всё", видны два
распространенных корня, по которым можно предположить, что пауки также
проживают выше (super) всего и всех (omni). Пока пауков находили на
высотах до 6700 метров, но, возможно, они встречаются и выше.

Источник:
https://ru.wikipedia.org/wiki/Euophrys_omnisuperstes

Автор:
Екатерина Лобкова (Обнинск - Москва)

Вопрос 19:
Ощутив толчок, пара персонажей фантастического рассказа Аллена Стила
пускается бежать по намеченному маршруту. Внезапно палуба выравнивается,
и стюард приглашает пассажиров на таможенный контроль. Тогда эти
персонажи с ужасом понимают, что могут не вернуться домой. Чей бюст
стоит на палубе?

Ответ:
[Пауля фон] Гинденбурга.

Комментарий:
Целью путешественников во времени было запечатлеть крушение
"Гинденбурга" и спастись, добравшись до трапа и спрыгнув на землю с
возможно меньшей высоты. Но взрыва не произошло, дирижабль успешно
причалил к мачте, и персонажи приходят в ужас, поняв, что история
изменилась, и они могут не суметь вернуться в свое время.

Источник:
А. Стил. Куда мудрец боится и ступить...
http://flibusta.is/b/310638/read

Автор:
Екатерина Лобкова (Обнинск - Москва)

Вопрос 20:
Внимание, в вопросе есть замена.
   В пьесе Пола РУдника, написанной в конце XX века, друг знаменитого
актера Джона Берримора помогает молодому коллеге подготовиться к
исполнению роли Гамлета. Напишите слово, замененное в предыдущем
предложении.

Ответ:
Дух.

Зачет:
Тень; призрак.

Комментарий:
Джон Берримор прославился шекспировскими ролями в первой трети XX века и
к моменту действия пьесы уже давно умер. Впрочем, это знать
необязательно, ведь и самому Гамлету давал советы призрак отца.

Источник:
https://ru.wikipedia.org/wiki/Я_ненавижу_Гамлета

Автор:
Наталия Новыш (Санкт-Петербург)

Вопрос 21:
Рассматривая фотографии своих бывших коллег, герой серии романов Ю Несбё
упоминает далласский клуб полицейских. Какие два слова мы заменили в
предыдущем предложении?

Ответ:
Общество мертвых.

Комментарий:
Несбё известен детективными романами, а коллеги инспектора ХОле стали
бывшими, потому что погибли. Замена обусловлена измененными названиями
двух оскароносных фильмов.

Источник:
Ю Несбё. Снеговик. http://flibusta.is/b/193538/read

Автор:
Кирилл Железнов (Калининград - Москва)

Вопрос 22:
Курительная комната дирижабля "Гинденбург" была оборудована ИКСОМ.
Попасть в охраняемый поселок под названием ИКСЫ проще всего зимой. Какое
слово мы заменили ИКСОМ?

Ответ:
Шлюз.

Комментарий:
В курилке дирижабля поддерживалось избыточное давление и имелся
воздушный шлюз, чтобы водород не мог туда проникнуть. Шлюзы - поселок
работников ПерЕрвинского гидроузла, расположенный на острове в
Москве-реке. Свободный проход туда, как и на большинство
гидротехнических сооружений, перекрыт, но зимой можно пройти на остров
по льду.

Источник:
   1. https://ru.wikipedia.org/wiki/Гинденбург_(дирижабль)
   2. https://mikeseryakov.livejournal.com/98014.html

Автор:
Екатерина Лобкова (Обнинск - Москва)

Вопрос 23:
Во французской геральдике черный цвет обозначается словом, восходящим к
заимствованию со значением "ИКС". Горьковский "ИКС" появился в 1998
году. Назовите ИКСА.

Ответ:
Соболь.

Комментарий:
С начала XIII века насыщенный черный называли термином "sobelins"
[собелЕн] или "sabelins" [сабелЕн] в честь меха соболя, который
привозили в Западную Европу из России и Польши, что со временем
трансформировалось в "sable" [сабль]. С 1998 года Горьковский автозавод
выпускает микроавтобус "Соболь".

Источник:
   1. М. Пастуро. Черный. История цвета.
http://flibusta.is/b/492640/read
   2. https://ru.wikipedia.org/wiki/Соболь_(марка_автомобилей)

Автор:
Дмитрий Сахаров (Тутаев)

Вопрос 24:
Последняя запись в твиттере Честера Беннингтона посвящена борьбе с
загрязнением планеты мусором. По мнению автора вопроса, 20 июля 2017
года ПРОИЗОШЛО ЭТО. Ответьте тремя словами из произведения 1989 года:
что произошло?

Ответ:
Пластмассовый мир победил.

Комментарий:
В своем последнем твите фронтмен "Linkin Park" [лИнкин парк] просил
людей не превращать Землю в планету пластика, поэтому можно сказать, что
в день его самоубийства...

Источник:
   1. https://twitter.com/chesterbe
   2. Мнение автора вопроса.
   3. http://www.gr-oborona.ru/texts/1056898718.html
   4. http://www.gr-oborona.ru/pub/rock/alboms.html

Автор:
Рузель Халиуллин (Салават - Москва)

Тур:
26 тур. "Бесславные у елки" (Пущино)

Редактор:
Данила Аладин

Вопрос 1:
Внимание, в вопросе есть замены.
   В популярной онлайн-игре "World of Warcraft" существует задание
"Вступить в игру", в котором герою необходимо вернуть адмиралу Одиссию
морской компас и морскую карту, украденные Торговой компанией. "Вступить
в игру" обычно говорят, когда сделан первый шаг. Какие два слова мы
заменили словами "вступить в игру"?

Ответ:
Начало положено.

Комментарий:
Когда сделан первый шаг, начало положено. Задание "Начало положено"
является одним из первых заданий на острове Лазурной дымки.

Источник:
https://ru.wowhead.com/quest=9506/начало-положено

Вопрос 2:
Прослушайте отрывок из песни Мирона Фёдорова "Неваляшка":
   Наш творец то ли хлопал ушами, то ли толком не шарил.
   И мы родились не в тот век, в холодной державе, не на том полушарии.
   Помним каждое слово, ИКСЫ того, за что не светят АЛЬФЫ.
   АЛЬФЫ по традиции вручаются лучшим ИКСАМ. Назовите ИКС одним словом и
АЛЬФУ двумя словами.

Ответ:
Знаток, хрустальная сова.

Комментарий:
Лучшие знатоки ЧГК награждаются хрустальными совами.

Источник:
https://genius.com/Oxxxymiron-nevalyashka-lyrics

Вопрос 3:
На одном из интернет-мемов изображены четыре портрета профессора Снегга
и стрелки, направленные к ним из общего начала. Что именно символизирует
это изображение?

Ответ:
Компас.

Источник:
https://pikabu.ru/story/velikiy_sneyp_5308050

Вопрос 4:
В одном из видео Youtube-канала "ТОП-5 ДАЙ 5" рассказывается о десяти
знаменитых людях, умерших в 2017 году. В эту десятку вошли Хью Хефнер,
Малкольм Янг, Михаил Задорнов, Честер Беннингтон, Lil Peep, Олег
Яковлев, Дэвид Рокфеллер, Дмитрий Хворостовский, Чак Берри и Роман
Сащенко, известный в России под прозвищем Рома ИКС. "ИКС" - это еще и
фильм Стивена Содерберга, вышедший 15 мая 1999 года. Назовите ИКС.

Ответ:
Англичанин.

Комментарий:
Роман Сащенко, он же Рома Англичанин, - один из дуэта ЛСП. "Англичанин"
- фильм Стивена Содерберга.

Источник:
   1. https://www.youtube.com/watch?v=Y_rqi03gW1Q
   2. https://www.kinopoisk.ru/film/4169/

Вопрос 5:
В одной из заставок сериала "Симпсоны" очень наглядно и смешно
представлена ОНА. От внимательных зрителей не ускользнул даже бармен Мо,
которого настиг обратный процесс. Ходят слухи, что данную серию из-за
заставки хотели запретить в ряде стран, дабы не оскорблять чувства
верующих. Назовите ЕЕ.

Ответ:
Эволюция.

Комментарий:
В заставке показано, как Гомер Симпсон эволюционирует из клетки в
человека и идет в свой дом смотреть телевизор с женой и детьми. Бармен
Мо идет в обратную сторону "из человека в крысу".

Источник:
https://www.youtube.com/watch?v=HZnw9JlAufk

Вопрос 6:
В одной английской шутке Гарри Поттер на английском говорит с одним
человеком. Вот перевод этого диалога:
   - Ты белый.
   - Нет, я черный.
   - Перестань шутить.
   - Я серьезно.
   Ответьте абсолютно точно: с кем говорит Гарри Поттер?

Ответ:
Сириус Блэк.

Комментарий:
   - You're white.
   - No, I'm Black.
   - Lol. Stop joking.
   - I'm Sirius.
   Фамилия Блэк означает "черный", а имя Сириус созвучно с "serious"
(серьезно).

Источник:
http://joyreactor.cc/post/181397

Вопрос 7:
В одной забавной рекламе фильма "Пятый элемент" на телеканале СТС
показывают главного героя фильма с разных ракурсов, повторяя "ИКС ВВЕРХ"
или "ИКС ВНИЗ". Данная шутка может восприниматься только русскоязычной
аудиторией. Какие имя и два антонима мы заменили словами "ИКС ВВЕРХ" и
"ИКС ВНИЗ"?

Ответ:
Брюс вылез, Брюс залез.

Комментарий:
Шутка основана на созвучии глагола "вылез" и фамилии Брюса Уиллиса,
сыгравшего в этом фильме главную роль.

Источник:
Реклама на телеканале СТС. https://www.youtube.com/watch?v=uul9LBqOdNk

Вопрос 8:
В одной забавной рекламе на телеканале СТС рекламируется фильм с
Николасом Кейджем. В самом рекламном ролике показаны разные музыкальные
жанры - рэп, поп и т.д. Какой фильм рекламировался в этом ролике?

Ответ:
Пророк.

Комментарий:
В рекламном ролике диктор говорил: "Этот фильм не про рэп. Этот фильм не
про поп. Потому что это про рок!".

Источник:
Реклама на телеканале СТС. https://www.youtube.com/watch?v=jJiEXmg9af4

Вопрос 9:
На одном из баттлов Versus BPM сошлись Алфавит и Хип-хоп одинокой
старухи. Оба участника не имеют речевых дефектов, однако второй уязвляет
первого видоизмененной детской скороговоркой на тему болезни своего
оппонента. Чем именно болен Алфавит?

Ответ:
Рак.

Комментарий:
Сунул Рики руку в друга, видит Рики: в друге рак!

Источник:
https://www.youtube.com/watch?v=nyfXpR6jwcE&t=12m03s

Вопрос 10:
В современных школах детям ЕЕ объясняют так: "Возьмем два разделенных
клапаном сосуда, один с газом, другой без. Далее откроем клапан, и
молекулы газа распределятся по обоим сосудам. Пройдет N тысяч лет, и
каждая молекула вернется на свое исходное место". Представители
гуманитарных дисциплин, услышав о НЕЙ, говорят, что жизнь всё расставит
по своим местам. В вопросах о НЕЙ по своим местам всё расставил
петербуржец в 2003 году. Назовите ЕЕ.

Ответ:
Теорема Пуанкаре.

Комментарий:
Теорема Пуанкаре - доказанная математическая гипотеза о том, что всякое
односвязное компактное трехмерное многообразие без края гомеоморфно
трехмерной сфере. Она была сформулирована в 1904 году математиком Анри
Пуанкаре, доказана Григорием Перельманом в серии статей 2002-2003 годов
и подтверждена математическим сообществом в 2006 году, став первой и
единственной на данный момент (2018 год) решенной задачей тысячелетия.

Источник:
https://ru.wikipedia.org/wiki/Гипотеза_Пуанкаре

Вопрос 11:
На экзамене по математическому анализу неподготовленный студент просит у
преподавателя последний шанс на "тройку", на что тот с улыбкой
предлагает ему СДЕЛАТЬ ЭТО. СДЕЛАТЬ ЭТО удалось профессору Саймону
Флеггу с помощью дьявола в одной советской короткометражке 1972 года.
Какие три слова мы заменили словами "СДЕЛАТЬ ЭТО"?

Ответ:
Доказать теорему Ферма.

Комментарий:
В рассказе Артура Порджеса "Саймон Флегг и дьявол" профессор Саймон
Флегг обращается за доказательством теоремы к дьяволу. По этому рассказу
снят игровой научно-популярный фильм "Математик и чёрт" (СССР, 1972,
производство Центрнаучфильм, творческое объединение "Радуга", режиссер
Семен Райтбурт).

Источник:
   1. Архив цитат Московского технологического университета МИРЭА.
   2. https://www.youtube.com/watch?v=7y0bxYz94XU

Вопрос 12:
В альтернативной студенческой озвучке фильма "Властелин Колец" в одной
из сцен Саруман создает существо, обладающее огромной силой и журналом
посещаемости занятий, которое не будет знать жалости и актуальных
новостей факультета и будет служить деканату. Формула создания этого
существа очень проста - хвост тритона, глаз лягушки и МНОГО ИКСОВ.
Говорят, что это существо еще на первом курсе отказывается от друзей в
пользу МНОГИХ ИКСОВ. Какие два слова мы заменили словами "МНОГО ИКСОВ"?

Ответ:
Сто рублей.

Комментарий:
Хвост тритона, глаз лягушки, сто рублей к стипендии, и получился
староста!

Источник:
https://www.youtube.com/watch?v=VIkCxxNAONM&t=40s

Вопрос 13:
Внимание, в вопросе есть замены.
   В народе говорят, что девушки на техническом факультете - как ЗУБРЫ в
СЕРПУХОВЕ, хоть заповедники отстраивай! Кстати, в СЕРПУХОВЕ за убийство
ЗУБРА предусмотрено очень суровое наказание. Какие слова мы заменили
словами "ЗУБРЫ" и "СЕРПУХОВ"?

Ответ:
Панды, Китай.

Комментарий:
За убийство панды в Китае наказывают смертной казнью.

Источник:
   1. https://www.youtube.com/watch?v=VIkCxxNAONM&t=5s
   2. https://ria.ru/science/20170926/1505567599.html

Вопрос 14:
На одном из интернет-мемов изображены четыре науки в виде разноцветных
человечков. Первая говорит: "Извлеки корень картофеля из земли". Вторая
продолжает: "Нарежь под правильным углом". Третья завершает: "Посыпь
хлоридом натрия". И четвертая подводит итог: "Ваша картошка готова,
спасибо за заказ". Назовите четвертую науку, днем рождения которой
считается 27 апреля 1839 года.

Ответ:
Социология.

Источник:
   1. https://pikabu.ru/story/novost_466_studentyi_uluchshili_vkus_kartoshki_poderevenski_s_pomoshchyu_geometrii_5645365
   2. https://ru.wikipedia.org/wiki/Социология

Вопрос 15:
В статье "Десять способов СДЕЛАТЬ ЭТО" фигурируют Орели-Антуан де Тунан,
Иоанн Люксембургский, Ахмет Зогу и другие. В детстве многие опрометчиво
мечтают СДЕЛАТЬ ЭТО. Какие два слова мы заменили словами "СДЕЛАТЬ ЭТО"?

Ответ:
Стать королем.

Комментарий:
Орели-Антуан де Тунан правил племенем индейцев-арауканов в Чили под
именем короля Антуана I. Иоанн Люксембургский почти двадцать лет был
чешским королем. Ахмет Зогу был албанским президентом, а затем стал
королем Албании.

Источник:
https://wowfacts.net/kak-stat-korolem-10-sposobov/

Вопрос 16:
В книге Майкла Стэкпола "Вол'джин. Тени Орды" автор утверждает, что ОНИ
- старейшая раса Азерота, и именно от НИХ произошли ночные эльфы. Мы же
считаем, что первые ОНИ появились вместе с первыми форумами. Назовите
ИХ.

Ответ:
Тролли.

Комментарий:
Речь идет о троллях в Интернете.

Источник:
М. Стэкпол. Вол'джин. Тени Орды. http://flibusta.is/b/364120/read

Вопрос 17:
В 1920 году В.И. Ленин одобрил проект строительства ЕЕ. Работать ОНА
начала 19 марта 1922 года. Другую ЕЕ создал Стивен Кинг в 1982 году.
Третью ЕЕ, появившуюся несколько ранее, создатель называл просто
300-метровой. Какое слово мы заменили словом "ОНА"?

Ответ:
Башня.

Комментарий:
Речь идет об Эйфелевой башне, "Темной башне" Стивена Кинга и Шуховской
башне.
   z-checkdb: Цикл "Темная башня" в 1982 году только начал создаваться,
а закончен он был в 2004 году, когда вышел одноименный роман, см.
авторский источник (Евгений Рубашкин).

Источник:
   1. https://ru.wikipedia.org/wiki/Шуховская_башня
   2. https://ru.wikipedia.org/wiki/Тёмная_Башня_(цикл)
   3. https://ru.wikipedia.org/wiki/Эйфелева_башня

Вопрос 18:
В романе Валентина Пикуля "Баязет" поручик Карабанов интересуется у
маркитанта гарнизона, сколько должен весить солдатский ИКС. На это
прапорщик Латышев говорит, что ИКС должен быть примерно с ладонь. ИКС -
это еще и некоторые промежуточные детали в механизмах и узлах машин,
разновидность зажима проводов в электротехнике. Также ИКСОМ называют
человека, не склонного к яркому проявлению эмоций. Назовите ИКС.

Ответ:
Сухарь.

Комментарий:
Солдатский сухарь в царской армии должен был быть размером с ладонь.
Также сухарем называют промежуточные детали в механизмах и узлах машин,
разновидность зажима проводов в электротехнике и неэмоционального
человека.

Источник:
   1. В.С. Пикуль. Баязет. http://flibusta.is/b/76743/read
   2. https://ru.wikipedia.org/wiki/Сухарь

Вопрос 19:
В статье Википедии под названием "ОНА" сказано, что на НЕЕ отозвались
стихами Твардовский, Симонов, Берггольц и Исаковский. Также "ОНА" - это
экранизированный комикс 2010 года Фабьена Нури и Тьерри Робена. Назовите
ЕЕ двумя словами, начинающимися на одну и ту же букву.

Ответ:
Смерть Сталина.

Комментарий:
На смерть Сталина отозвались стихами советские литераторы: Твардовский,
Симонов, Берггольц и Исаковский. Комикс экранизировали в 2017 году.

Источник:
   1. https://ru.wikipedia.org/wiki/Смерть_Сталина
   2. https://ru.wikipedia.org/wiki/Смерть_Сталина_(значения)

Вопрос 20:
Во французском фильме "Долгая помолвка" описаны трагические дни Первой
мировой войны. Незадолго до мобилизации молодой парень Манек, проводя
время со своей невестой Матильдой, постоянно повторяет ей три слова. На
войне он, попав под обстрел немецкого самолета, прощаясь с жизнью,
вырезает на стволе дерева три первые буквы тех самых слов. Этими тремя
буквами также было названо одно акционерное общество, появившееся в
России в 1989 году. Напишите эти три буквы.

Ответ:
МММ.

Комментарий:
МММ - "Манек - муж Матильды", "Матильда - мечта Манека". АО "МММ" -
крупнейшая в истории России финансовая пирамида, организованная
россиянином Сергеем Мавроди. Изначально структура "МММ" была создана в
1989 году и до 1 февраля 1994 года вела только финансовую и торговую
деятельность.

Источник:
   1. Х/ф "Долгая помолвка" (2004), реж. Жан-Пьер Жене.
   2. https://ru.wikipedia.org/wiki/МММ

Вопрос 21:
По словам мастера айкидо Макото Ито, любое боевое искусство начинается с
НЕЕ. В одной японской притче мастер чайной церемонии поссорился с
самураем и был вызван им на поединок, отложенный на неделю. Убоясь
смерти, мастер чайной церемонии пришел к мастеру кендо с просьбой
обучить его. Всю неделю мастер учил его ЕЙ. Придя в назначенное время,
самурай увидел ЕЕ мастера чайных церемоний, испугался и принес тому
извинения, отказавшись от поединка. Назовите ЕЕ.

Ответ:
Стойка.

Комментарий:
Безупречная стойка отражает уверенность, бесстрашие и мужество мастера.
Любое будо начинается с правильной стойки.

Источник:
Семинар Макото Ито, г. Реутов, ноябрь 2017 г. (личный опыт автора
вопроса).

Вопрос 22:
По словам академика Алехновича, ИКС является непростым испытанием для
всех первокурсников МГТУ им. Баумана. В одной известной антиутопии ИКСОМ
называется космический корабль, который должен нести просветление в
другие миры. Назовите ИКС.

Ответ:
Интеграл.

Комментарий:
Обычно интегрирование нелегко дается студентам первого курса. В
антиутопии Замятина "Мы" главный герой занимается строительством
корабля, который понесет просветление, - "Интегралом".

Источник:
   1. Личный опыт автора вопроса.
   2. Е.И. Замятин. Мы. http://flibusta.is/b/314962/read

Вопрос 23:
Помимо своей политической деятельности, ОН - географ и геоморфолог,
исследователь тектонического строения Сибири и Средней Азии и
ледникового периода, предсказавший существование Земли Франца-Иосифа.
Архитекторы Лихтенберг и Душкин при создании проекта станции "Дворец
советов" обращались к архитектуре Древнего Египта - подземным безоконным
постройкам, освещаемым лишь масляными плашками вверху колонн. Годами
позже эту станцию переименовали в ЕГО честь. Назовите ЕГО.

Ответ:
[Петр] Кропоткин.

Комментарий:
Речь идет о станции московского метро "Дворец советов", которая в наши
дни называется "Кропоткинская".

Источник:
   1. https://ru.wikipedia.org/wiki/Кропоткин,_Пётр_Алексеевич
   2. https://ru.wikipedia.org/wiki/Кропоткинская_(станция_метро)

Вопрос 24:
Согласно словарю Ефремовой, "Бывай!" говорят при прощании, расставании
на длительное время или навсегда, или при выражении утраты, лишения,
исчезновения чего-либо. Также "Бывай!" - это песня композитора Вячеслава
Добрынина на слова Леонида Дербенёва, написанная в 1975 году. В чьем
исполнении эта песня получила наибольшую известность?

Ответ:
Лев Лещенко.

Комментарий:
Команда "Бесславные у елки" прощается с вами и благодарит за внимание,
уделенное нашим вопросам.

Источник:
   1. https://dic.academic.ru/dic.nsf/efremova/278901/
   2. https://ru.wikipedia.org/wiki/Прощай_(песня)

Тур:
27 тур. "Pro Forma" (Москва)

Вопрос 1:
Начнем с простого вопроса. АЛЬФА в нем заменяет другое слово.
   Самая дорогая АЛЬФА в мире обходится более чем в 100 миллионов
долларов в год - из-за упущенной выгоды. А что на ней написано?

Ответ:
"Мне повезет".

Зачет:
"I'm Feeling Lucky".

Комментарий:
АЛЬФА - это кнопка. На кнопке "Мне повезет", которая переадресует
пользователя в обход рекламы прямо на сайт, стоящий первым в поисковой
выдаче, Google теряет порядка 100-150 миллионов долларов в год. Удачи!

Источник:
http://seooki.ua/blog/mne-povezet/

Автор:
Сергей Вознесенский

Вопрос 2:
Герой Сигизмунда Кржижановского собирал необычную коллекцию. Одними из
наименее ценных экземпляров в ней были ИКСЫ сторожа или неудачливой
проститутки. Запишите ИКС двумя символами.

Ответ:
??

Комментарий:
Этот герой зарисовывал зевки. Их у представителей ночных профессий
бывает достаточно. В шахматной нотации зевок обозначается двумя
вопросительными знаками.

Источник:
   1. С.Д. Кржижановский. Материалы к биографии Горгиса Катафалаки.
http://az.lib.ru/k/krzhizhanowskij_s_d/text_0270.shtml
   2. https://ru.wikipedia.org/wiki/Зевок_(шахматы)

Автор:
Андрей Алдашев

Вопрос 3:
Описывая одну из опасностей, подстерегающих участников рассчитанной на
триста лет космической экспедиции, автор фантастического произведения
упоминает две известные итальянские фамилии. Какие?

Ответ:
Капулетти, Монтекки.

Зачет:
В любом порядке.

Комментарий:
За время полета успеют смениться много поколений, поэтому ссора кого-то
из первых путешественников может перерасти в кровную вражду их потомков,
что поставит под угрозу успех экспедиции. Слово "фамилия" здесь
использовано в значении "семейство".

Источник:
Б.Ф. Лапин. Первый шаг. http://flibusta.is/b/31558/read

Автор:
Андрей Алдашев

Вопрос 4:
Герою Ю Несбё не сразу удается поразить синего змея. Какой предмет он
при этом держит в руке?

Ответ:
Шприц.

Комментарий:
Со змеем сравнивается вена наркомана.

Источник:
Ю Несбё. Призрак. http://flibusta.is/b/306888/read

Автор:
Андрей Алдашев

Вопрос 5:
В русском написании имени известного древнего правителя есть ПЕРВОЕ и
ТРЕТИЙ. Назовите ВТОРОЕ.

Ответ:
Горло.

Комментарий:
Врача отоларинголога нередко называют "ухо-горло-нос". В русском
написании имени царя Навуходоносора есть "ухо" и "нос".

Источник:
А надо?

Автор:
Андрей Алдашев

Вопрос 6:
Московские неонацисты заставили встретившегося им азиата перед
видеокамерой отречься от ислама. Причиной того, что тот исполнил
требование скинхедов, были не побои и угрозы, а ИКС. Назовите ИКС словом
с удвоенной согласной.

Ответ:
Буддизм.

Комментарий:
Видимо, избитый был бурятом или тувинцем, так что не боялся, что Аллах
покарает его.

Источник:
https://www.kommersant.ru/doc/2804707

Автор:
Андрей Алдашев

Вопрос 7:
Морячок из повести Константина Бадигина раскрывает тайну пробравшихся на
корабль шпионов, и те выбрасывают его за борт. Позже один из них
отвечает заподозрившему неладное капитану примерно так же, как Путин.
Какие две буквы мы заменили тремя в предыдущем предложении?

Ответ:
Ка.

Комментарий:
Капитан спросил, не видел ли тот морячка. Шпион ответил, что капитан
должен сам за этим следить, а он не сторож его матросам. Впрочем,
морячок не утонул.

Источник:
   1. К.С. Бадигин. Секрет государственной важности.
http://flibusta.is/b/137620/read
   2. https://dic.academic.ru/dic.nsf/dic_wingwords/2337/

Автор:
Андрей Алдашев

Вопрос 8:
[Ведущему: кавычки в вопросе не озвучивать.]
   В романе американского писателя отмечается, что "Украина" - та еще
дыра по сравнению с "Россией", где у героя был вид на жительство. Какое
слово мы заменили в тексте вопроса?

Ответ:
Кремль.

Зачет:
Мавзолей.

Комментарий:
Речь идет о московских гостиницах. Более современная "Россия" западным
фасадом выходила на Красную площадь.

Источник:
М.К. Смит. Парк Горького. http://flibusta.is/b/80161/read

Автор:
Андрей Алдашев

Вопрос 9:
До знакомства с неким небольшим списком автор вопроса не догадывался,
что после города Афины может идти спарта. Что мы пропустили в предыдущем
предложении?

Ответ:
Заглавную [букву] Ы.

Зачет:
Прописную [букву] Ы; Ы.

Комментарий:
Речь идет об игре в города. Несколько населенных пунктов, чьи названия
начинаются на "Ы", существуют, и среди них турецкий город Ыспарта. Герои
известного фильма используют букву Ы, "чтоб никто не догадался".

Источник:
https://traditio.wiki/Список_городов_на_букву_Ы

Автор:
Андрей Алдашев

Вопрос 10:
Как ни странно, воры в законе Джони Бурдгунич, Ахмед Шалинский, Ило
Калининградский и Вова Зюзя СДЕЛАЛИ ЭТО. Какие четыре слова мы заменили
словами "СДЕЛАЛИ ЭТО"?

Ответ:
Родились в год Петуха.

Комментарий:
Все они 1981 года рождения. По восточному календарю год Петуха еще
продолжается.

Источник:
http://www.primecrime.ru/chronicles/1981/birth/

Автор:
Андрей Алдашев

Вопрос 11:
Причиной, по которой около года назад в Петербурге не состоялось
праздничное мероприятие для учеников английской спецшколы, стала ОНА.
Для героя известного фильма и ОНА не стала решением проблемы. Назовите
ЕЕ двумя словами, начинающимися на одну и ту же букву.

Ответ:
Смерть сурка.

Комментарий:
Предсказывавший погоду сурок Фрол из петербургского зоопарка умер, а
другого ручного зверька не нашлось.

Источник:
http://www.rosbalt.ru/piter/2017/02/01/1588212.html

Автор:
Андрей Алдашев

Вопрос 12:
На своих селфи одна пользовательница Инстаграма, в адресе которой есть
цифры 1916 [девятнадцать шестнадцать], по мнению автора вопроса, нередко
напоминает зомби. Напишите краткую форму ее имени.

Ответ:
Ира.

Зачет:
Ирка.

Комментарий:
Год основания Ирландской Республиканской Армии совпадает с номером
школы, который указала в своем адресе в Инстаграме московская школьница
Ира Нечитаева. Этот вопрос мы посвящаем памяти Долорес О'Риордан.

Источник:
https://www.instagram.com/irka1916/

Автор:
Андрей Алдашев

Вопрос 13:
Внимание, в вопросе есть замена.
   В 2017 году на сайте Примечания.ру появилась ироническая редакция
статьи под заголовком: "В Крымских горах спасли женщину от
самоубийства". Какое сложное слово мы заменили в тексте заголовка?

Ответ:
Роскомнадзора.

Комментарий:
Местный новостной ресурс опубликовал информацию об операции по спасению
женщины, пытавшейся покончить с собой. На одно из слов в заголовке
среагировал поисковый бот Роскомнадзора, и вскоре в редакцию пришло
уведомление о том, что статья "содержит информацию, распространение
которой в Российской Федерации запрещено". Редакция подумала-подумала и
зароскомнадзорила в статье всё, что могла.

Источник:
https://primechaniya.ru/home/news/noyabr_2017/spasateli_snyali_so_skal_krymchanku-_pytavshuyusya_pokonchit_s_soboj/

Автор:
Сергей Вознесенский

Вопрос 14:
В известном английском романе экран сравнивается с мутным ПЕРВЫМ.
Интересно, что ПЕРВОЕ по-английски - это в том числе ТАКОЕ ВТОРОЕ.
Назовите по-английски ТАКОЕ ВТОРОЕ.

Ответ:
[The] Looking glass.

Комментарий:
Роман, о котором идет речь, - "1984". Как известно, телеэкраны в романе
на самом деле являются еще и телекамерами. Словосочетание "The looking
glass" (т.е. зеркало) можно перевести не только как "смотровое", но и
как "смотрящее" стекло, что в контексте романа выглядит весьма
символично.

Источник:
Дж. Оруэлл. 1984. http://flibusta.is/b/229751/read

Автор:
Сергей Вознесенский

Вопрос 15:
В романе "Фимбулвинтер. Пленники бирюзы" описан проект разветвленной
сети порталов, соединяющей множество миров. Этот проект носит название
"ИКС". Фамилия какого поэта, согласно одной гипотезе, родственна слову
"ИКС"?

Ответ:
Есенин.

Комментарий:
Согласно скандинавским легендам, основой мироздания является исполинский
ясень Иггдрасиль, по ветвям которого разбросаны Девять Миров.

Источник:
   1. Ю. Лебединская, И. Вереснев, Н. Немытов, Е. Красносельская, О.
Силин, Ю. Скуркис. Фимбулвинтер. Пленники бирюзы.
http://flibusta.is/b/351465/read
   2. https://geno.ru/article/122/

Автор:
Сергей Вознесенский

Вопрос 16:
За три месяца до предполагаемой публикации ОН написал пятисотстраничный
черновик первого тома, но к работе над вторым томом приступить уже не
успевал. Однако выход из кошмарной ситуации все-таки был найден.
Назовите ЕГО фамилию.

Ответ:
Менделеев.

Комментарий:
В итоге он забил на дедлайн и лег спать. Дальше вы знаете.

Источник:
С. Кин. Исчезающая ложка, или Удивительные истории из жизни
периодической таблицы Менделеева.
https://books.google.ru/books?id=LFsmCwAAQBAJ&pg=PT51#v=onepage&q&f=false

Автор:
Сергей Вознесенский

Вопрос 17:
   <раздатка>
   Задне*******й район
   </раздатка>
   В каком российском областном центре есть район, в названии которого
мы скрыли несколько букв?

Ответ:
Смоленск.

Комментарий:
Заднепровский район включает в себя всю часть города, расположенную по
ту сторону Днепра от исторического центра.

Источник:
https://ru.wikipedia.org/wiki/Административное_деление_Смоленска

Автор:
Андрей Алдашев

Вопрос 18:
   <раздатка>
   colon is obligatory prerequisite to be a colonisator
   </раздатка>
   Словосочетание "obligatory prerequisite" [облИгатори прерЕквизит]
переводится на русский язык как "обязательное условие". Какие слова мы
заменили словами "colon" [кОлон] и "colonisator" [колонизЭйтор]?

Ответ:
Ass, assenisator.

Комментарий:
Второй пункт инструкции "Как стать ассенизатором" гласит: "Убедитесь в
наличии задницы. Наличие задницы - обязательное условие для
ассенизатора". Логично - с нее, как-никак, вся работа и начинается.
Возможно, вам могло помочь знание слова "колоноскопия", но взять через
это вопрос, пожалуй, трудновато.

Источник:
https://banga.tv3.lt/lt/2forum.showPosts/340881.121-=(2954284059

Автор:
Сергей Вознесенский

Вопрос 19:
Программа для автоматизации управления учебным заведением, разработанная
в 2009 году, была призвана повысить возможности руководителя. Напишите
ее название, использовав дефис.

Ответ:
"E-rector".

Зачет:
В узнаваемой русской транслитерации.

Источник:
https://www.facebook.com/erectorerp/

Автор:
Андрей Алдашев

Вопрос 20:
Алексей Швед - несомненный лидер баскетбольного клуба "Химки". Когда в
одной встрече на последних минутах заключительной четверти он совершает
насколько удачных действий, в трансляции "Матч-ТВ" упоминается последний
богатырь. Что мы заменили в тексте вопроса?

Ответ:
Время первых.

Комментарий:
Швед - несомненный лидер команды, и от его игры в решающий момент
зависит итоговый результат. Кроме того, Швед играет под первым номером.
Название одного отечественного фильма 2017 года мы заменили названием
другого.

Источник:
http://news.sportbox.ru/Vidy_sporta/Basketbol/vtb-league/spbvideo_NI751755_translation_Jedinaja_liga_VTB_1_2_finala_Zenit___Khimki

Автор:
Андрей Алдашев

Вопрос 21:
Известный человек, записывая средневековую историю, не расслышал слово
"vair" [вэйр], означающее "беличья шкурка", и использовал в тексте слово
"verre" [верр] - "стекло". Назовите этого человека.

Ответ:
[Шарль] Перро.

Комментарий:
Так меховая туфелька Золушки превратилась в хрустальную. Существует
устойчивое мнение, что "меховая туфелька" - изящный эвфемизм.

Источник:
М. Пастуро. Зеленый. История цвета. http://flibusta.is/b/512461/read

Автор:
Сергей Вознесенский

Вопрос 22:
Chuo [чУо] - это японские высокоскоростные поезда, отличающиеся
исключительно строгим следованием расписанию. Какими тремя буквами
оканчивается русскоязычный неологизм, образованный от слова "Чуо"?

Ответ:
-цид.

Комментарий:
Высокая скорость и точность прибытия - подарок для уставшего от жизни
самурая.

Источник:
https://www.urbandictionary.com/define.php?term=Chuocide

Автор:
Сергей Вознесенский

Вопрос 23:
Надеемся, что этот вопрос не станет гробом.
   На шуточной картинке под названием "Право первой ночи" друг
спрашивает у жениха: "А где твоя-то?" Тот грустно отвечает: "ПРОПУСК".
Заполните пропуск тремя словами.

Ответ:
Моя у барона.

Комментарий:
Оооооо.

Источник:
Поле экспериментов автора вопроса.

Автор:
Сергей Вознесенский

Вопрос 24:
(pic: 20180332.jpg)
   Подходить к столу апелляционного жюри сразу после этого вопроса не
рекомендуется.
   Розданное вам изображение - это не редакторский брак: просто вы
смотрите на персонажа ПРОПУСК. Заполните пропуск, использовав дефис.

Ответ:
Из-под лупы.

Зачет:
Из-за лупы.

Комментарий:
А с той стороны, кстати, Шерлок. Держитесь подальше от апелляционного
жюри. Спасибо за внимание.

Источник:
???

Автор:
Сергей Вознесенский


FreeBSD-CVSweb <freebsd-cvsweb@FreeBSD.org>